You are on page 1of 217

Mc lc

1 Dy s v cc bi ton v dy s 4
1.1 Gii thiu . . . . . . . . . . . . . . . . . . . . . . . . . . . . . . . . 4
1.2 nh ngha v cc nh l c bn . . . . . . . . . . . . . . . . . . . 5
1.3 Mt s phng php gii bi ton v dy s . . . . . . . . . . . . . 8
1.3.1 Dy s thc: mt s dng dy s c bit . . . . . . . . . . 8
1.3.2 Dy s nguyn . . . . . . . . . . . . . . . . . . . . . . . . . 12
1.3.3 Dy s v phng trnh . . . . . . . . . . . . . . . . . . . . 17
1.3.4 Mt vi th thut khc . . . . . . . . . . . . . . . . . . . . 18
1.4 Mt s phng php xy dng h thng bi tp . . . . . . . . . . . 23
1.4.1 Xy dng dy hi t bng phng trnh . . . . . . . . . . . 23
1.4.2 Xy dng dy truy hi t cp nghim ca phng trnh bc 2 24
1.4.3 Xy dng cc dy s nguyn t li gii cc phng trnh
nghim nguyn . . . . . . . . . . . . . . . . . . . . . . . . . 25
1.4.4 Xy dng dy s l nghim ca mt h phng trnh ph
thuc bin n . . . . . . . . . . . . . . . . . . . . . . . . . . 26
1.5 L thuyt dy s di con mt ton cao cp . . . . . . . . . . . . . 27
1.5.1 Ri rc ha cc khi nim v nh l ca l thuyt hm
bin s thc . . . . . . . . . . . . . . . . . . . . . . . . . . . 27
1.5.2 Phng php hm sinh v bi ton tm s hng tng qut . 29
1.5.3 i s tuyn tnh v phng trnh sai phn . . . . . . . . . 30
1.5.4 S dng xp x trong d on kt qu . . . . . . . . . . . . 31
1.6 Bi tp . . . . . . . . . . . . . . . . . . . . . . . . . . . . . . . . . 32
2 Phng trnh sai phn 41
2.1 Sai phn . . . . . . . . . . . . . . . . . . . . . . . . . . . . . . . . . 41
2.1.1 nh ngha . . . . . . . . . . . . . . . . . . . . . . . . . . . 41
2.1.2 Tnh cht . . . . . . . . . . . . . . . . . . . . . . . . . . . . 41
2.2 Phng trnh sai phn tuyn tnh . . . . . . . . . . . . . . . . . . . 43
2.2.1 Mt s khi nim chung v phng trnh sai phn . . . . . 43
2.3 Phng trnh sai phn tuyn tnh bc nht . . . . . . . . . . . . . 44
1
MC LC 2
2.3.1 nh ngha . . . . . . . . . . . . . . . . . . . . . . . . . . . 44
2.3.2 Phng php gii . . . . . . . . . . . . . . . . . . . . . . . . 44
2.3.3 Phng php tm nghim ring ca phng trnh sai phn
tuyn tnh cp 1 khng thun nht khi v phi f(n) c
dng c bit . . . . . . . . . . . . . . . . . . . . . . . . . . 45
2.3.4 Bi tp . . . . . . . . . . . . . . . . . . . . . . . . . . . . . 47
2.4 Phng trnh sai phn tuyn tnh cp 2 . . . . . . . . . . . . . . . 47
2.4.1 nh ngha . . . . . . . . . . . . . . . . . . . . . . . . . . . 47
2.4.2 Cch gii . . . . . . . . . . . . . . . . . . . . . . . . . . . . 48
2.5 Phng trnh sai phn tuyn tnh cp 3 . . . . . . . . . . . . . . . 55
2.5.1 nh ngha . . . . . . . . . . . . . . . . . . . . . . . . . . . 55
2.5.2 Phng php gii . . . . . . . . . . . . . . . . . . . . . . . . 55
2.5.3 V d . . . . . . . . . . . . . . . . . . . . . . . . . . . . . . 56
2.5.4 Phng trnh sai phn tuyn tnh cp k . . . . . . . . . . . 58
3 Xc nh s hng tng qut ca mt dy s 60
3.1 Tm s hng tng qut ca dy (dng a thc) khi bit cc s
hng u tin . . . . . . . . . . . . . . . . . . . . . . . . . . . . . . 61
3.2 Cng thc truy hi l mt biu thc tuyn tnh . . . . . . . . . . . 63
3.2.1 V d . . . . . . . . . . . . . . . . . . . . . . . . . . . . . . 64
3.3 Cng thc truy hi l mt h biu thc tuyn tnh . . . . . . . . . 70
3.3.1 V d . . . . . . . . . . . . . . . . . . . . . . . . . . . . . . 70
3.4 Cng thc truy hi l biu thc tuyn tnh vi h s bin thin . . 72
3.5 Cng thc truy hi dng phn tuyn tnh vi h s hng . . . . . . 78
3.6 H thc truy hi phi tuyn . . . . . . . . . . . . . . . . . . . . . . 81
3.6.1 Quy trnh tuyn tnh ho mt phng trnh sai phn . . . . 82
3.6.2 V d . . . . . . . . . . . . . . . . . . . . . . . . . . . . . . 83
3.6.3 Mt s v d khc . . . . . . . . . . . . . . . . . . . . . . . 87
3.6.4 Bi tp. . . . . . . . . . . . . . . . . . . . . . . . . . . . . . 96
4 Phng trnh hm sai phn bc hai 99
4.1 Hm tun hon v phn tun hon cng tnh . . . . . . . . . . . . 99
4.2 Phng trnh hm sai phn bc hai vi hm tun hon v phn
tun hon . . . . . . . . . . . . . . . . . . . . . . . . . . . . . . . . 100
4.3 Phng trnh vi hm s tun hon, phn tun hon nhn tnh . . 108
4.3.1 nh ngha . . . . . . . . . . . . . . . . . . . . . . . . . . . 109
4.3.2 Mt s bi ton . . . . . . . . . . . . . . . . . . . . . . . . . 109
4.3.3 Mt s v d p dng . . . . . . . . . . . . . . . . . . . . . 125
MC LC 3
5 Dy s sinh bi hm s 128
5.1 Hm s chuyn i php tnh s hc v i s . . . . . . . . . . . . 128
5.2 V cc dy s xc nh bi dy cc phng trnh . . . . . . . . . . 135
5.3 nh l v ba mnh tng ng . . . . . . . . . . . . . . . . . 141
5.4 Mt s bi ton v c lng tng v tch . . . . . . . . . . . . . . 142
5.5 Bi tp . . . . . . . . . . . . . . . . . . . . . . . . . . . . . . . . . 144
6 Mt s lp hm chuyn i cc cp s 145
6.1 Cp s cng, cp s nhn v cp s iu ho . . . . . . . . . . . . 145
6.2 Dy s tun hon . . . . . . . . . . . . . . . . . . . . . . . . . . . . 146
6.3 Hm s chuyn i cp s cng . . . . . . . . . . . . . . . . . . . . 152
6.4 Hm s chuyn i cp s cng vo cp s nhn . . . . . . . . . . . 154
6.5 Hm s chuyn i cp s nhn vo cp s cng . . . . . . . . . . . 155
6.6 Hm s chuyn i cp s nhn vo cp s iu ho . . . . . . . . 156
7 Mt s lp hm chuyn i cc cp s trong tp ri rc 158
7.1 Hm s chuyn i cp s cng thnh cp s cng . . . . . . . . . 158
7.2 Hm s chuyn i cp s nhn thnh cp s nhn . . . . . . . . . 161
8 Mt s bi ton xc nh dy s trong lp dy tun hon cng
tnh v nhn tnh. 167
8.1 Mt s bi ton xc nh dy s trong lp dy tun hon cng tnh167
8.2 Hm s xc nh trn tp cc s nguyn . . . . . . . . . . . . . . . 170
8.2.1 Hm s chuyn i cc php tnh s hc . . . . . . . . . . 170
8.2.2 Hm s chuyn tip cc i lng trung bnh . . . . . . . . 172
8.2.3 Phng trnh trong hm s vi cp bin t do . . . . . . . 177
8.2.4 Mt s dng ton lin quan n dy truy hi . . . . . . . . 180
8.3 Hm s xc nh trn tp cc s hu t . . . . . . . . . . . . . . . 184
8.4 Phng trnh trong hm s vi cp bin t do . . . . . . . . . . . . 191
8.5 S dng gii hn gii phng trnh hm . . . . . . . . . . . . . 198
Ti liu tham kho . . . . . . . . . . . . . . . . . . . . . . . . . . . . . 217
Chng 1
Dy s v cc bi ton v dy
s
1.1 Gii thiu
Chn ti v dy s, chng ti t trc mnh mt nhim v v cng kh
khn, bi y l mt lnh vc rt kh v rt rng, s dng nhiu kin thc khc
nhau ca ton hc. Hn th, trc c kh nhiu cun sch chuyn kho v
ti ny. D vy, chng ti vn mun c gng ng gp mt s kinh nghim v
ghi nhn ca mnh thu lm c trong qu trnh ging dy nhng nm qua.
Tp ti liu ny khng phi l mt gio trnh v dy s, li cng khng phi
l mt cm nang hng dn gii cc bi ton dy s. Tp ti liu ny ng hn
ht l nhng cp nht ca tc gi v nhng phng php gii cc bi ton dy
s cng vi nhng nhn nh i khi mang y tnh ch quan ca tc gi. V vy,
hy coi y l mt ti liu m. Hy tip tc trin khai, lin h v c kt kinh
nghim, ghi nhn nhng ci hay v gp cho nhng ci cha hay, thm ch cha
chnh xc.
Trong ti liu ny, khng phi tt c cc vn ca dy s u c cp
ti. V d phn dy s v bt ng thc ch c ni n rt s si, cc bi ton
dy s m thc cht l cc bi ton v ng d cng khng c xt ti... Hai
mng ln m tp ti liu ny ch n nht l bi ton tm s hng tng qut
ca mt dy s v bi ton tm gii hn dy s.
Trong tp ti liu ny, cc vn v cc bi ton c mc kh d khc
nhau. C nhng bi c bn, c nhng bi kh hn v c nhng bi rt kh. V
vy, cn phi la chn vn vi mc thch hp (v d c mt s vn v
bi ton ch ng phi mc k thi chn i tuyn hoc quc t).
Vit tp ti liu ny, tc gi s dng rt nhiu ngun ti liu khc nhau,
tuy nhin ch c mt s bi c ghi ngun gc, mt s bi khng th xc nh c.
4
1.2. nh ngha v cc nh l c bn 5
Tc gi cng s dng cc bi ging ca cc thy Phan c Chnh, Nguyn
Vn Mu, L nh Thnh, ng Hng Thng, Nguyn Minh c... trong bi vit
ca mnh.
Cui cng, tp ti liu ny khng khi c nhng nhm ln v thiu st, tc
gi rt mong nhn c s gp ca tt c cc thy c gio. V rt mong rng,
vi n lc chung ca tt c chng ta, tp ti liu s tip tc c hon thin v
b sung.
1.2 nh ngha v cc nh l c bn
nh ngha 1.1. Dy s l mt hm s t N vo mt tp hp s (N, Q, R, C)
hay mt tp con no ca cc tp hp trn). Cc s hng ca dy s thng
c k hiu l u
n
, v
n
, x
n
, y
n
thay v u(n), v(n), x(n), v(n). Bn thn dy s c
k hiu l x
n
.
V dy s l mt trng hp c bit ca hm s nn n cng c cc tnh
cht ca mt hm s.
nh ngha 1.2. Dy s x
n
c gi l dy tng (gim) nu vi mi n ta c
x
n+1
x
n
(x
n+1
x
n
). Dy s tng hoc dy s gim c gi chung l dy n
iu.
Dy s xn c gi l b chn trn nu tn ti s thc M sao cho vi mi n
ta c x
n
M.
Dy s x
n
c gi l b chn di nu tn ti s thc m sao cho vi mi n
ta c x
n
m.
Mt dy s va b chn trn, va b chn di c gi l dy b chn.
Dy s x
n
c gi l tun hon vi chu k k nu x
n+k
= x
n
vi mi n N. Dy
s tun hon vi chu k 1 gi l dy hng.
nh ngha 1.3. Ta ni dy s x
n
c gii hn hu hn a khi n dn n v
cng nu vi mi > 0, tn ti s t nhin N
0
(ph thuc vo dy s x
n
v )
sao cho vi mi n > N
0
ta c [x
n
a[ nh hn .
lim
n
x
n
= a > 0N
0
N : n > N
0
[xn a[ <
Ta ni dy s x
n
dn n v cng khi n dn n v cng nu vi mi s
thc dng M ln tu , tn ti s t nhin N
0
(ph thuc vo dy s x
n
v M)
sao cho vi mi n > N
0
ta c [x
n
[ ln hn M.
lim
n
x
n
= M > 0N
0
N : n > N
0
[x[ > M.
Dy s c gii hn hu hn c gi l dy hi t. Dy s khng c gii hn
hoc dn n v cng khi n dn n v cng gi l dy phn k.
1.2. nh ngha v cc nh l c bn 6
nh l 1.1 (Tng, hiu, tch, thng cc dy hi t). Nu x
n
, y
n
l cc
dy hi t v c gii hn tng ng l a, b th cc dy s x
n
+ y
n
, x
n
y
n
,
x
n
y
n
v x
n
/y
n
cng hi t v c gii hn tng ng l a + b, a b, a.b, a/b.
(Trong trng hp dy s thng, ta gi s y
n
v b khc khng)
nh l 1.2 (Chuyn qua gii hn trong bt ng thc). Cho dy s x
n
c
gii hn hu hn l, nu N
0
N : n > N
0
ta c a x
n
b th a x
n
b.
nh l 1.3 (nh l kp). Cho ba dy s x
n
, y
n
, z
n
trong x
n
v z
n
c
cng gii hn hu hn 1, v N
0
N : n > N
0
ta c x
n
y
n
z
n
. Khi y
n
cng c gii hn l 1.
nh l 1.4 (Dy n iu). Mt dy tng v b chn trn hay mt dy gim
v b chn di th hi t. Ni ngn gn hn, mt dy s n iu v b chn th
hi t.
nh l 1.5 (V dy cc on thng lng nhau). Cho hai dy s thc a
n
, b
n

sao cho
a) n N, a
n
b
n
;
b) nN, [a
n+1
, b
n+1
] [a
n
, b
n
];
c) b
n
a
n
0 khi n .
Khi tn ti duy nht s thc l sao cho [a
n
, b
n
] = 1.
nh l 1.6 (Bolzano Veierstrass). T mt dy b chn lun c th trch ra mt
dy con hi t.
nh ngha 1.4. Dy x
n
c gi l dy Cauchy nu > 0N
0
N: m, n >
N
0
[x
m
x
n
[ < .
nh ngha 1.5 (Tiu chun Cauchy). Dy s x
n
c gii hn hu hn khi v
ch khi n l dy Cauchy.
Cp s cng. Dy s x
n
c gi l mt cp s cng khi v ch khi tn
ti d sao cho
n N, x
n+1
= x
n
+d.
d c gi l cng sai ca cp s cng, x
0
l s hng u, x
n
l s hng th n.
Ta c cc cng thc c bn sau:
x
n
= x
0
+nd
S
n
= x
0
+x
1
+ + x
n1
= nx
0
+n(n 1)d/2
= n(x
0
+x
n1
)/2
1.2. nh ngha v cc nh l c bn 7
Cp s nhn. Dy s x
n
c gi l mt cp s nhn khi v ch khi tn ti
q sao cho
n N, x
n+1
= qx
n
.
d c gi l cng bi ca cp s nhn, x
0
l s hng u, x
n
l s hng th n.
Ta c cc cng thc c bn sau:
x
n
= q
n
x
0
S
n
= x
0
+x
1
+ +x
n1
= (q
n
1)x
0
/(q 1)
Nu [q[ < 1 th x
n
c gi l cp s nhn li v hn. Tng ca cp s nhn
li v hn c tnh theo cng thc
S = x
0
/(1 q)
Dy Fibonacci. Dy s Fibonacci l dy s c nh ngha bi
f
0
= 0, f
1
= 1, n N, f
n+2
= f
n+1
+f
n
.
Dy s Fibonacci c rt nhiu tnh cht th v v xut hin mt cch t nhin
trong nhiu lnh vc khc nhau. Chng ta c cng thc sau y tm s hng
tng qut ca dy s Fibonacci:
Cng thc Binet.
f
n
=
_
1+

5
2
_
n

_
1

5
2
_
n

5
.
Ni chung, cc dy s xc nh bi cng thc truy hi f
n+2
= f
n+1
+ f
n
(vi
f
0
, f
1
bt k) c gi l dy Fibonacci m rng.
Dy Farey. Dy Farey F
n
vi mi s nguyn dng n l tp hp cc phn s
ti gin dng a/b vi 0 a b n v (a, b) = 1 xp theo th t tng dn.
V d 1.1.
F
5
= 0/1, 1/5, 1/4, 1/3, 2/5, 1/2, 3/5, 2/3, 3/4, 4/5, 1/1.
Ngoi tr F
1
, F
n
c s l cc phn t v 1/2 lun nm gia. Gi p/q, p

/q

v
p

/q

l cc s hng lin tip trong dy Farey th


pq

qp

= 1, v p

/q

= (p + p

)/(q +q

).
S cc s hng N(n) trong dy Farey c tnh theo cng thc
N(n) = 1 +
n

k=1
(k) = 1 +(n).
1.3. Mt s phng php gii bi ton v dy s 8
1.3 Mt s phng php gii bi ton v dy s
Phng php gii cc bi ton dy s rt a dng nh chnh yu cu ca
chng. c th l mt tnh cht s hc, mt tnh cht i s hay mt tnh cht
gii tch. Di y chng ta s xem xt nhng phng php c bn nht.
Tuy nhin, c th a ra hai nguyn l chung gii cc bi ton dy s l
- ng ngi vit ra cc s hng u tin ca dy s
- ng ngi tng qut ha bi ton
1.3.1 Dy s thc: mt s dng dy s c bit
Dy s dng x
n+1
= f(x
n
)
y l dng dy s thng gp nht trong cc bi ton v gii hn dy s.
Dy s ny s hon ton xc nh khi bit f v gi tr ban u x
0
. Do vy s hi
t ca dy s s ph thuc vo tnh cht ca hm s f(x) v x
0
. Mt c im
quan trng khc ca dy s dng ny l nu a l gii hn ca dy s th a phi l
nghim ca phng trnh x = f(x). Chng ta c mt s kt qu c bn nh sau:
nh ngha 1.6. Hm s f : D D c gi l mt hm s co trn D nu tn
ti s thc q, 0 < q < 1 sao cho [f(x) f(y)[ q[x y[ vi mi x, y thuc D.
nh l 1.7. Nu f(x) l mt hm s co trn D th dy s x
n
xc nh bi
x
0
= a D, x
n+1
= f(x
n
) hi t. Gii hn ca dy s l nghim duy nht trn
D ca phng trnh x = f(x).
Chng minh. Vi mi n > m th p dng nh ngha hm s co, ta c
[x
n
x
m
[ = [f(x
n1
) f(x
m1
)[ q[x
n1
x
m1
[ q
m
[x
nm
x
0
[ (1.1)
T y [x
n
x
0
[ [x
n
x
n1
[ + +[x
1
x
0
[ (q
n1
+ + 1)[x
1
x
0
[, suy
ra x
n
b chn. Xt > 0. T (1.1), do q < 1 v [x
nm
x
0
[ b chn nn ta suy
ra tn ti N sao cho q
N
[x
nm
x
0
[ < . Suy ra x
n
l dy Cauchy v do hi
t.
V d 1.2 (Vit Nam, 2000). Cho dy s x
n
xc nh nh sau
x
0
= 0, x
n+1
=
_
c

c + x
n
.
Tm tt c cc gi tr ca c vi mi gi tr x
0
(0, c), x
n
xc nh vi mi n
v tn ti gii hn hu hn lim
n
x
n
.
1.3. Mt s phng php gii bi ton v dy s 9
Gii. x
1
tn ti th ta th c

c +x
n
0 vi mi x
0
(0, c) hay c(c1) x
0
vi mi x
0
(0, c), suy ra c 2. Vi c 2 th 0 < x
1
<

c. Nu 0 < x
n
<

c
th c

c +x
n
> c 2

c, suy ra x
n+1
tn ti v ta cng c 0 < x
n+1
<

c.
t f(x) =
_
c

c +x th f

(x) =
1
4

x +x
_
c

c + x.
Vi mi x (0,

c) ta c (c + x)(c

c + x) > c(c
_
c +

c) 2(2
_
2 +

2) >
1
4
. T suy ra [f

(x)[ q < 1 vi mi x (0,

c), tc f(x) l
hm s co trn (0,

c), suy ra dy s cho hi t. Vy tt c cc gi tr c cn


tm l c 2.
Mt trng hp na cng c th xt c s hi t ca dy s x
n
l trng
hp f n iu. C th l
Nu f l hm s tng trn D th x
n
s l dy n iu. Dy s ny tng
hay gim tu theo v tr ca x
0
so vi x
1
.
Nu f l hm gim trn D th cc dy con x
2p
, x
2p+1
l cc dy n iu
(v ngc chiu nhau).
V d 1.3 (V ch sinh vin Moskva, 1982). Cho dy s x
n
xc nh bi
x
0
= 1982, x
n+1
= 1/(4 3x
n
). Hy tm lim
n
x
n
Gii. Tnh ton trc tip ta thy 0 < x
2
< 1, x
3
> x
2
. V f(x) = 1/(4 3x) l
mt hm s tng t [0, 1] vo [0, 1] nn t y, x
n

n2
l mt dy s tng v b
chn trn bi 1 do c gii hn. Gi s gii hn l a th ta c a = 1/(4 3a)
hay a = 1 (gi tr a = 1/3 loi do dy tng).
Cu hi: Vi nhng gi tr no ca x
0
th dy s xc nh vi mi x v c gii
hn? Khi no th gii hn l 1? Khi no th gii hn l 1/3?
Trong trng hp f l hm gim, ta c th chng minh dy hi t bng cch
chng minh hai dy con trn cng hi t v mt gii hn.
Tuy nhin, kh khn nht l gp cc hm s khng n iu. Trong trng
hp ny, ta phi xt tng khong n iu ca n v s hi t ca hm s s ty
thuc vo gi tr ban u.
V d 1.4. Tm tt c cc gi tr ca a dy s x
n
xc nh bi x
0
= a, x
n+1
=
2 x
2
n
c gii hn hu hn.
Gii. Hm s f(x) = 2 x
2
tng trn (, 0) v gim trn (0, +). Phng
trnh f(x) = x c hai nghim l x = 2 v x = 1. l nhng d kin quan
trng trong li gii bi ton ny.
u tin, ta nhn xt rng nu a < 2 th do f : (, 2) (, 2) v
l hm tng, x
1
= 2 a
2
< x
0
nn dy s x
n
gim. Nu dy x
n
b chn di
th n hi t v nghim ca phng trnh x = 2 x
2
, iu ny mu thun v dy
gim v x
0
< 2. Vy x
n
khng b chn di, tc khng c gii hn hu hn.
Nu a > 2 th x
1
< 2 v ta cng suy x
n
khng c gii hn hu hn.
1.3. Mt s phng php gii bi ton v dy s 10
Vi a = 2, 1 th dy s c gii hn. Xt x
0
[2, 2]. Ta chng minh dy s
c gii hn hu hn khi v ch khi tn ti n sao cho x
n
= 2 hoc x
n
= 1. Tht
vy, gi s x
n
c gii hn hu hn l b v x
n
/ 2, 1 vi mi n. Khi b = 2
hoc b = 1. Gi s b = 2 th tn ti N
0
sao cho x
n
nm trong ln cn 2 vi
mi n N
0
. Nhng nu x
n
= 2+ th x
n+1
= 2+4
2
> x
n
, suy ra dy x
n
tng k t N
0
v khng th dn v 2. Nu b = 1 k t n N
0
no x
n
thuc
ln cn 1. Xt
x
n+2
x
n
= 2 (2 x
2
n
)
2
x
n
= (2 x
n
x
2
n
)(x
2
n
x
n
1)
Ti ln cn 1 th x
2
n
x
n
1 < 0. V nu x
n
< 1 th x
n+1
> 1 (v ngc li
x
n
> 1 th x
n+1
< 1 - chng ta ang xt trong ln cn im 1!) nn c th gi
s x
n
> 1. Khi 2 x
n
x
2
n
< 0 suy ra x
n+2
> x
n
. Tip tc nh vy, suy ra
1 < x
n
< x
n+2
< < x
n+2k
< mu thun vi gi thit b = 1. Vy iu gi
s l 2, tc l dy s ch c gii hn khi tn ti n sao cho x
n
= 2 hoc x
n
= 1.
Sau khi thu c kt qu ny, ta s dng hm ngc f
1
(x) =

2 x
xy dng tt c cc gi tr a tha mn iu kin u bi.
Trong v d trn, ta s dng gi thit tn ti gii hn thu gn min D,
t mt hm c bin thin phc tp tr thnh mt hm n iu.
Dy s dng x
n+1
= x
n
(x
n
)

v nh l trung bnh Cesaro


y l trng hp c bit ca dy s dng x
n+1
= f(x
n
). Tuy nhin, vi
dy s dng ny vn hi t ca x
n
thng khng c t ra (v qu n gin
v gii hn ch c th l 0 hoc ). y, ta s c mt yu cu cao hn l tm
bc tim cn ca x
n
, c th l tm b sao cho x
n
= O(n

). Vi cc dy s c dng
ny, nh l trung bnh Cesaro s t ra rt hu hiu.
nh l 1.8 (Trung bnh Cesaro). Nu dy s x
n
c gii hn hu hn l a
th dy s cc trung bnh x
1
+ x
2
+ + x
n
)/n cng c gii hn l a.
nh l ny c th pht biu di dng tng ng nh sau: Nu limn (x
n+1

x
n
) = a th lim
n
x
n
/n = a.
Ta chng minh nh l cch pht biu 2. R rng ch cn chng minh cho
trng hp a = 0. V lim
n
(x
n+1
x
n
) = 0 nn vi mi > 0 tn ti, N
0
sao
cho vi mi n N
0
ta c [x
n+1
x
n
[ < . Khi , vi mi n > N
0
[x
n
/n[ [[x
N
0
[ +[x
N
0
+1
x
N0
[ + +[x
n
x
n1
[]/n < [x
N
0
[/n + (n N
0
)/n.
Gi c nh N
0
, ta c th tm c N
1
> N
0
sao cho [x
N0
[/N
1
< . Khi vi
mi n > N
1
ta s c [x
n
/n[ < 2. Vy lim
n
x
n
/n = 0.
nh l trung bnh Cesaro c nhiu ng dng quan trng trong vic tm gii
hn dy s v c th pht biu cho cc trung bnh khc nh trung bnh nhn,
1.3. Mt s phng php gii bi ton v dy s 11
trung bnh iu ha, trung bnh ly tha. Tuy nhin, y ta ch khai thc cch
pht biu 2 ca nh l p dng cho cc dy s c dng x
n+1
= x
n
(x
n
)

.
tm s sao cho x
n
/n

c gii hn hu hn, theo nh l trung bnh Cesaro, ta


ch cn tm g sao cho x

n+1
x

n
c gii hn hu hn a. Khi , lim
n
x

n
/n = a,
suy ra limx
n
/n

1
= a

1
, tc l = 1/.
V d 1.5. Cho dy s x
n
c xc nh bi x
0
= 1/2, x
n+1
= x
n
x
2
n
. Chng
minh rng lim
n
nx
n
= 1.
Gii. Trong bi ny, = 1 do ta s th vi = 1. D dng chng minh
c lim
n
x
n
= 0. Ta c
1/x
n+1
1/x
n
= (x
n
x
n+1
)/x
n+1
x
n
= x
2
n
/(x
n
x
2
n
)x
n
= 1/(1 x
n
) 1.
T p dng nh l trung bnh Cesaro, suy ra lim1/nx
n
= 1, suy ta limnx
n
=
1.
V d 1.6. Cho dy s x
n
c xc nh bi x
0
= 1, x
n+1
= sin(x
n
). Chng
minh rng lim

nx
n
=

3.
Gii. Dy s cho khng c dng x
n+1
= x
n
(x
n
)

(?) nhng kt lun ca


bi ton gi cho chng ta n nh l trung bnh Cesaro. V = 1 nn ta s
th vi = 2. D dng chng minh c rng limx
n
= 0. Xt
1/x
2
n
1/x
2
n
= [x
2
n
sin
2
(x
n
)]/x
2
n
sin
2
(x
n
) 1/3
(Dng quy tc LHopitale)
T , theo nh l trung bnh Cesaro lim1/nx
2
n
= 1/3, suy ra limlim

n.x
n
=

3.
Nh vy, ta c th tm nu bit . Trong trng hp khng bit th ta
phi d on.
V d 1.7 (Chn i tuyn Vit Nam, 1993). Dy s a
n
c xc nh bi
a
1
= 1 v a
n+1
= a
n
+ 1/

a
n
. Hy tm tt c cc s thc dy s (a
n
)

/n
c gii hn hu hn khc 0.
Gii. Trc ht ta chng minh a
n
dn ti v cng khi n dn ti v cng. Tht
vy, ta c a
2
n+1
= a
2
n
+ 2

a
n
+ 1/a
n
> a
2
n
+ 2. Suy ra a
2
n+1
> 1 + 2n suy ra
(pcm). Tr li bi ton, xt
a
3/2
n+1
a
3/2
n
= (a
n
+ 1/

a
n
)
3/2
a
3/2
n
= (1 + 1/a
3/2
n
)
3/2
/(1/a
3/2
n
)
t x = 1/a
3/2
n
th x 0 khi n . Do lim
n
(a
3/2
n+1
a
3/2
n
) =
lim
x0
(1 + x)
3/2
/x = 3/2 (Quy tc LHopitale) T suy ra lim a
3/2
n
/n = 3/2.
1.3. Mt s phng php gii bi ton v dy s 12
Vi > 3/2 suy ra gii hn bng , vi < 3/2 suy ra gii hn bng 0. Vy
= 3/2 l gi tr duy nht tho mn yu cu bi ton.
Cu hi:
1) Lm sao c th d on c gi tr ?
2) v c mi quan h g?
1.3.2 Dy s nguyn
Dy s nguyn l mt phn quan trng trong l thuyt dy s. Ngoi cc vn
chung nh tm s hng tng qut ca dy s, tm cng thc tnh tng n s
hng u tin... cc bi ton v dy s nguyn cn quan tm n tnh cht s
hc ca dy s nh chia ht, ng d, nguyn t, chnh phng, nguyn t cng
nhau... Cc bi ton v dy s nguyn rt a dng. Trong nhiu trng hp, dy
s ch l ci b ngoi, cn bn cht bi ton l mt bi ton s hc. Trong cc
phn di y, chng ta s t cp n nhng bi ton nh vy m chuyn
chng vo phn bi tp.
Nguyn l Dirichlet v dy s nguyn
Nguyn l Dirichlet l mt nguyn l ht sc n gin nhng li v cng hu
hiu trong cc bi ton chng minh, c bit l chng minh s tn ti ca mt
i tng tho mn mt iu kin no . S dng nguyn l ny, ngi ta
chng minh c nhiu kt qu rt mnh, v d nh nh l Fermat-Euler v
tng hai bnh phng, nh l Weil v phn b u... y ta nu ra hai kt
qu lin quan n dy s:
nh l 1.9 (Weil, v phn b u). Nu l s v t th dy n
n=1
phn
b u trn khong (0, 1).
nh l 1.10 (V s tun hon ca cc s d). Cho dy s nguyn x
n

xc nh bi cng thc truy hi x


n+k
= a
1
x
n+k1
+ + a
k
x
n
v k s hng u
tin nguyn. Khi , vi mi s nguyn dng N, dy s d ca x
n
khi chia cho
N s tun hon.
Tip theo ta xt mt vi v d v vic s dng nguyn l Dirichlet trong cc
bi ton dy s.
V d 1.8. Chng minh rng nu 1 a
1
, a
2
, ..., a
n+1
2n th tn ti i < j sao
cho a
i
[ a
j
.
Gii. Mi s a
i
c th vit di dng a
i
= 2
s
i
r
i
vi r
i
l s l. Cc s r
i
ch c
th nhn n gi tr t 1, 3, ..., 2n1. V c n + 1 s nn theo nguyn l Dirichlet,
tn ti i < j sao cho r
i
= r
j
v tng ng ta c a
i
[ a
j
.
1.3. Mt s phng php gii bi ton v dy s 13
V d 1.9 (Tp chi AMM). Xt n s nguyn dng a
1
< a
2
< < a
n
2n
sao cho [a
i
, a
j
] > 2n vi mi i ,= j. Chng minh rng a
1
> 2n/3.
Gii. Nu a
1
2n/3, ta xt n +1 s 2a
1
, 3a
1
, a
2
, . . . , a
n
. Cc s ny u khng
ln hn 2n v khng c s no l bi ca s no. iu ny mu thun vi kt qa
bi ton trn.
V d 1.10. (Canada, 2000) Cho A = (a
1
, a
2
, ..., a
n
) l dy cc s nguyn thuc
on [1000, 1000]. Gi s tng cc s hng ca A bng 1. Chng minh rng tn
ti mt dy con (cha t nht 1 phn t) ca A c tng bng 0.
Gii. Ta c th gi s trong A khng c phn t no bng 0, v nu ngc
li th bi ton hin nhin. Ta sp xp dy A thnh dy B = (b
1
, ..., b
2000
) bng
cch chn dn t cc s hng ca dy A theo quy tc sau: b
1
> 0, b
2
< 0. Vi
mi i 3 chn b
i
l s c du ngc vi du ca tng s
i1
= b
1
+ + b
i1
(v sao lun thc hin c?). Bng cch xy dng nh th, ta c 2000 s
s
1
, s
2
, ..., s
2000
nm trong on [999, 1000]. Nu trong s s
i
c mt s bng 0
th bi ton ng. Trong trng hp ngc li, theo nguyn l Dirichlet tn ti
i < j sao cho s
i
= s
j
. Khi b
i+1
+ +b
j
= 0.
H m c s v dy s nguyn
H m c s c th dng xy dng nhiu dy s c tnh cht rt th
v. Nhn trn phng din ca mt c s khc, c th rt kh nhn ra quy lut,
nhng nu chn ng c s th bi ton tr nn v cng n gin.
Xin nhc li l vi b l mt s nguyn dng ln hn hay bng 2 th mi s
nguyn dng N u c th biu din mt cch duy nht di dng
N = a
1
...a
k
(b) = a
1
b
k1
+ +a
k
vi 1 a
1
b 1, 0 a
2
, . . . , ak b 1.
l nh ngha h m c s dng c bn nht. Tuy nhin, c th ly mt dy
s nguyn bt k (c tr tuyt i tng nghim ngt) lm h m c s v d h
m c s (2), h m c s Fibonacci (3 = 4 2 + 1, 17 = 13 + 3 + 1...)
Cc h m thng s dng nht l h m c s 2 v c s 3. Di y ta xt
mt vi v d:
V d 1.11 (IMO 1983). Chng minh hoc ph nh mnh sau: T tp hp
105 s nguyn dng u tin lun c th chn ra mt tp con gm 1983 s sao
cho khng c ba s no lp thnh mt cp s cng.
Gii. Ta chng minh mnh tng qut: T 3n s t nhin u tin lun c
th chn ra 2n s sao cho khng c ba s no lp thnh mt cp s cng. Tht
vy, xt trong h m c s 3 tp hp tt c cc s c n ch s. Chn cc s
m trong biu din tam phn ca n ch cha ch s 2 v ch s 0. Khi c 2n
s nh vy v khng c ba s no trong chng lp thnh mt cp s cng.
1.3. Mt s phng php gii bi ton v dy s 14
V d 1.12 (Singapore 1995). Cho dy s f
n
xc nh bi f
1
= 1, f
2n
= f
n
v f
2n+1
= f
2n+1
.
(i) Tnh M = maxf
1
, ..., f
1994

(ii) Tm tt c cc gi tr n, 1 n 1994 sao cho f


n
= M.
Gii. Kinh nghim mt cht ta thy ngay f
n
chnh l tng cc ch s ca n
trong h m nh phn. T y do 1994 < 2048 = 211 suy ra M = 10.
V d 1.13. Dy s f
n
c xc nh bi f
1
= 1, f
2n
= 3f
n
, f
2n+1
= f
2n+1
.
Hy tnh f
100
.
Gii. f
n
c xc nh nh sau: Xt biu din nh phn ca n ri tnh gi
tr ca s nh phn ny trong h tam phn. V 100 = 26 + 25 + 22 nn f
100
=
36 + 35 + 32 = 981.
V d 1.14. Dy s a
n
c xc nh bi 0 a
0
< 1, a
n
= 2a
n1
nu 2a
n1
<
1 v a
n
= 2a
n1
1 nu 2a
n1
1. Hi c bao nhiu gi tr a
0
a
5
= a
0
.
Gii. Phn tch: Khi tnh a
n
theo a
n1
ta c th la chn mt trong hai cng
thc. Tt nhin, vi a
0
chn ri th tt c cc bc tip theo u xc nh mt
cch duy nht. Tuy nhin, ta c th chn a
0
nh th no sau cc cng
thc tnh theo ng kch bn cho. C 2
5
= 32 kch bn nh vy. V d vi kch
bn (1, 1, 2, 1, 2) ta c x
1
= 2x
0
, x
2
= 2x
1
= 4x
0
, x
3
= 2x
2
1 = 8x
0
1, x
4
=
2x
3
= 16x
0
2, x
5
= 2x
4
1 = 32x
0
3.
Gii phng trnh x
0
= x
5
ta c x
0
= 3/31. Tt nhin, c c mt li
gii hon chnh, ta cn phi lp lun cht ch thy rng cc x
0
thu c l
khc nhau v vi mi x
0
thu c, dy s s "i" ng nh kch bn nh.
Tuy nhin, phn tch ny gi chng ta hng n h nh phn. V ta c li gii
p mt sau:
Nu a
0
= 0, d
1
d
2
d
3
. . . l biu din nh phn ca a
0
th a
1
= 0, d
2
d
3
d
4
. . . Tht
vy, nu 2a
0
< 1 th d
1
= 0 v a
1
= 2a
0
= 0, d
2
d
3
d
4
. . . cn nu 2a
0
1 th d
1
= 1
v a
1
= 2a
0
1 = 0, d
2
d
3
d
4
. . .
Hon ton tng t, a2 = 0, d
3
d
4
d
5
. . . , . . . , a
5
= 0, d
6
d
7
d
8
. . . Nh vy a
5
= a
0
khi v ch khi a
0
l phn s nh phn tun hon chu k 5. C 2
5
= 32 chu k tun
hon nh vy, trong chu k 11111 cho chng ta a
0
= 1 (loi). Vy tt c c
31 gi tr a
0
tha mn yu cu bi. l 0, (00000), 0, (00001), . . . , (0, 11110).
Tnh sang h thp phn l cc gi tr 0, 1/31, 2/31, . . . , 30/31.
S phc v dy s nguyn
S phc c nhng ng dng rt quan trng trong ton hc ni chung v trong
l thuyt dy s ni chung. Nh s phc, chng ta c th thy c mi quan h
gia hm lng gic v hm m. Nh s phc, mi a thc bc n u c n
1.3. Mt s phng php gii bi ton v dy s 15
nghim v v vy nh l Vit mi pht huy c tc dng. Di y ta xt mt
s v d v ng dng ca s phc trong cc bi ton tnh tng v dy truy hi.
V d 1.15. Vi s nguyn dng n, hy tnh
A(n) = C
0
n
+ C
3
n
+ + C
3[n/3]
n
.
Gii. C th t B(n) = C
1
n
+C
4
n
+ +C(n) = C
2
n
+C
5
n
+ ri s dng cc
cng thc
A(n) +B(n) = B(n + 1), B(n) + C(n) = C(n + 1), C(n) + A(n) = A(n + 1)
tm cng thc tnh A(n). Tuy nhin da theo cch tnh C
0
n
+C
2
n
+ +C
2
n
[n/2]
bng cch thay x = 1, y = 1 v x = 1, y = 1 vo cng thc nh thc Newton, ta
c cch gii khc kh p nh sau: Gi l s tha mn phng trnh
2
++1 = 0.
Do
3
= 1 nn ta c
(1 + 1)
n
= A(n) + B(n) + C(n)
(1 +)
n
= A(n) + B(n) +
2
C(n)
(1 +
2
)
n
= A(n) +
2
B(n) + C(n)
T y suy ra 3A(n) = 2
n
+ (1 + )
n
+ (1 +
2
)
n
. T y, dng cng thc
Moivre ta tm c
A(n) = [2n + 2 cos(np/3)]/3.
V d 1.16. Tnh tng S
n
(x) = C
0
n
+ C
1
n
cos x + + C
n
n
cos nx.
Gii. t Tn(x) = 0 + C
1
n
sin x + + C
n
n
sin nx th S
n
(x) + iT
n
(x) = C
0
n
+
C
1
n
(cos x+i sinx)+ +C
n
n
(cos x+i sinx)
n
= (1+cos x+i sinx)
n
= 2[cos(x/2)[cos(x/2)+
i sin(x/2)]]
n
= 2
n
cos
n
(x/2)[cos(nx/2) + i sin(nx/2)].
T suy ra S
n
(x) = 2
n
cos
n
(x/2) cos(nx/2).
V d 1.17 (AMM). Cho dy s u
n
xc nh bi u
0
= 3, u
1
= 0, u
2
=
2, u
n+3
= u
n+1
+u
n
. Chng minh rng u
p
lun chia ht cho p nu p l s nguyn
t.
Gii. Phng trnh c trng ca dy s c dng x
3
x 1 = 0. Nu phng
trnh c trng ny c nghim nguyn th ta c th s dng nh l nh Fermat
chng minh kt lun ca bi ton. Tuy nhin, cc nghim ny khng nguyn,
thm ch phng trnh ch c 1 nghim thc. Ta phi cu cu n s tr gip ca
s phc.
Gi u, v, wl ba nghim ca phng trnh th u+v+w = 0, uv+vw+wu = 1,
suy ra u
2
+ v
2
+ w
2
= (u +v + w)
2
2(uv +vw +wu) = 2. T ta c th kt
lun
u
n
= u
n
+ v
n
+ w
n
1.3. Mt s phng php gii bi ton v dy s 16
Vi p l s nguyn t l th u
p
= (v +w)
p
= v
p
w
p

p1
i=1
C
i
p
v
i
w
pi
.
Tng t v
p
= w
p
u
p

i = 1
p1
C
i
p
w
i
u
pi
, w
p
= u
p
v
p

p1
i=1
C
i
p
u
i
v
pi
.
T suy ra 3(u
p
+v
p
+w
p
) =

p1
i=1
C
i
p
(v
i
w
pi
+ w
i
u
pi
+ u
i
v
pi
)
By gi, ch rng C
i
p
chia ht cho p vi 1 i p 1
i
(v p l s nguyn t)
v (v
i
w
pi
+ w
i
u
pi
+ u
i
v
pi
) l s nguyn (biu thc i xng i vi u, v, w)
nn v phi l mt s nguyn chia ht cho p. Vy vi p nguyn t, p > 3 bi ton
c chng minh. Cui cng ch u
2
= 2, u
3
= 3 ta c bi ton ng vi mi
p.
Dy s dng [n]
Dy s dng x
n
= [n] c nhiu tnh cht s hc th v. Nu a > 1 th
[n

]
n1
l dy cc s nguyn dng phn bit, c s bin thin gn ging mt
cp s cng nhng li khng phi l mt cp s cng. Dy s ny c bit th v
khi a l s v t bc hai. Ta c mt kt qa quen thuc sau y
nh l 1.11. Nu a, b l cc s v t dng tho mn iu kin 1/a + 1/b = 1
th hai dy s x
n
= [n], y
n
= [n], n = 1, 2, 3, ... lp thnh mt phn hoch ca
tp hp cc s nguyn dng.
Chng minh. Xt hai dy s , 2, 3, ...v , 2, 3, ... Khng mt s hng
no trong cc s hng trn l s nguyn. Vi mi s nguyn dng N, c [N/] s
hng ca dy th nht nm bn tri N v [N/] s hng ca dy th hai. Nhng
N/ + N/ = N, v , l cc s v t, phn l ca cc s N/ v N/ l cc
s dng c tng bng 1 (do ng thc trn). Suy ra c [N/] + [N/] = N 1
s hng ca c hai dy nm bn tri N. V bn tri N + 1 c N s hng ca c
hai dy nn gia N v N +1 c ng mt s hng ca mt trong hai dy, t
suy ra iu phi chng minh.
Cu hi: C th pht biu v chng minh nh l o nh th no?
Hai dy s trn vt ht tp hp cc s nguyn dng. iu ny cho chng ta
mt hng suy ngh: nu hai dy s vt ht tp hp cc s nguyn dng th c
kh nng chng s c dng trn. V nhiu bi ton c xy dng theo hng
ny. Chng ta xt mt v d
V d 1.18 (AMM). Gi s f
n
v g
n
l hai dy s nguyn dng c xc
nh nh sau
1) f
1
= 1
2) g
n
= na 1 f
n
, trong a l s nguyn ln hn 4,
3) f
n+1
l s nguyn dng nh nht khc cc s f
1
, f
2
, ..., f
n
, g
1
, g
2
, ..., g
n
.
Chng minh rng tn ti cc hng s , sao cho f
n
= [n], g
n
= [n] vi
mi n = 1, 2, 3, ...
1.3. Mt s phng php gii bi ton v dy s 17
Gii. Theo cch xy dng f
n
v g
n
lp thnh mt phn hoch ca N

. Gi
s ta tm c a, b tha mn iu kin u bi, khi , ta phi c 1/+1/ = 1.
Ngoi ra, khi n ln th na 1 = f
n
+ g
n
n + n, suy ra + = a. Vy
, phi l nghim ca phng trnh x
2
ax +a = 0.
Xt phng trnh x
2
ax +a = 0 c hai nghim < . V a > 4, , l cc
s v t. Dy s f
n
v g
n
c xc nh mt cch duy nht, do chng
minh khng nh ca bi ton, ta ch cn chng minh [n] v [n] tha mn
cc iu kin 1), 2), 3).
R rng [a] = 1, [n] = [n(a )] = n + [n)] = na [n] 1 (do n v
t).
Gi s [n] = [m] = k, t n = k +r, m = k + s vi 0 < r, s < 1 th
n +m = k(1/ + 1/) + r/ +s/ = k + r/ + s/,
iu ny khng th xy ra v 0 < r/ + s/ < 1. Nh vy vi mi m, n ta c
[n] ,= [m].
Tip theo,
[(n + 1)] [n] + 1, [(n + 1)] [n] + 2 > [n] + 1.
Cui cng gi s k l mt s nguyn bt k v n = [(k + 1)/]. Nu n > k/
th k < n < (k + 1)/ = k + 1 v [n] = k. Nu n < k/ th (k n) >
k k/ = k(1 1/) = k, (k n) < k ((k +1)/1) = k +1, suy ra
[(k n)] = k.
T cc nhn xt trn ta suy ra mi s nguyn dng k c mt trong dy s
ng mt ln v hai dy s [n] v [n] tha mn iu kin 3) (pcm)
Ghi ch: Trong li gii trn, ta khng dng n kt qu ca nh l trn
v cng chnh l mt cch chng minh khc cho nh l.
Cc bi ton v dy s dng [n] thng lin quan n phn hoch v cc
dy s gn tuyn tnh (x
m+n
x
m
+x
n
). Xin xem thm mt s v d trong phn
bi tp.
1.3.3 Dy s v phng trnh
Dy s c mi quan h rt cht ch vi phng trnh. iu ny c th thy
rt r qua hai v d c bn: phng trnh sai phn tuyn tnh c gii bng vic
xt nghim ca phng trnh c trng, gii hn ca dy s cng thng c
gii ra t mt phng trnh. V vn ny, xin c thm cc mc tng ng
trong bi ny. y l mt trong nhng ni dung quan trng nht trong phn dy
s.
1.3. Mt s phng php gii bi ton v dy s 18
1.3.4 Mt vi th thut khc
Sp xp li th t
Sp xp li th t l mt th thut thng c p dng trong cc bi ton
lin quan n bt ng thc trong dy s. Vic sp xp li th t cc s trn
ng thng dn n cc tnh cht c bit m mt dy s bt k khng c,
chng hn nu a < b < c th [c a[ = [c b[ + [b a[. Cng nh cc nguyn l
c bn khc, nguyn l n gin ny t ra kh hu hiu trong nhiu trng hp.
V d 1.19 (Vit Nam 1998). Tn ti hay khng mt dy s thc x
n
tha
mn iu kin
1) [x
n
[ 0, 666 vi mi n = 1, 2, 3, ...
2) [x
m
x
n
[ 1/n(n + 1) + 1/m(m+ 1) vi mi s nguyn dng m,n.
Gii. Gi s tn ti dy s nh vy. Vi mi s nguyn dng N, ta sp xp li
cc s x
1
, ..., x
N
theo th t tng dn
x
i1
x
i2
x
iN
Khi [x
iN
x
i1
[ = [x
iN
x
iN1
[+ +[x
i2
x
i1
[1/iN(iN+1)+1/i
N1
(i
N1
+
1)+ +1/i
2
(i
2
+1)+1/i
1
(i
1
+1) = 2

1/i
k
(i
k
+1)1/i
N
(i
N
+1)1/i
1
(i
1
+1) =
A(N).
V i
1
, i
2
, ..., i
N
ch l mt hon v ca 1, 2, ..., N nn ta c
A(N) = 2

1/k(k + 1) 1/i
N
(i
N
+ 1) 1/i
1
(i
1
+ 1)
= 2(1 1/(N + 1)) 1/i
N
(i
N
+ 1) 1/i
1
(i
1
+ 1)
2(1 1/(N + 1)) 1/1.2 1/2.3 = 4/3 2/(N + 1)
By gi ch rng [x
iN
x
i1
[ 2x0, 666 < 4/3. Chn N ln sao cho 4/3
2/(N + 1) > 2x0, 666, ta suy ra mu thun. Vy khng tn ti dy s tha mn
yu cu bi.
V d 1.20 (Lin X 1986). Gi s a
1
, a
2
, ..., a
n
l cc s dng tu . Chng
minh bt ng thc
1/a
1
+ 2/(a
1
+a
2
) + + n/(a
1
+ +a
n
) < 4(1/a
1
+ 1/a
2
+ + 1/a
n
)
Gii. V phi khng thay i nu ta thay i th t ca a
i
do ta ch cn
(v phi) chng minh bt ng thc ng cho trng hp tng bn tri ln
nht. iu ny xy ra khi a
i
c sp theo th t tng dn. Tht vy, gi s
0 < b
1
b
2
... b
n
l cc s a
i
c sp xp li. Khi r rng vi mi k ta
c b
1
+ + b
k
a
1
+ +a
k
v
1/a
1
+2/(a
1
+a
2
)+ +n/(a
1
+ +a
n
) 1/b
1
+2/(b
1
+b
2
)+ +n/(b
1
+ +b
n
)
1.3. Mt s phng php gii bi ton v dy s 19
Vi mi k, ghp cc s hng ca tng bn phi thnh cp ta c nh gi sau
(2k1)/(b
1
+ +b
2k1
)+2k/(b
1
+ +b
2k1
) < (2k1)/kb
k
+2k/(k+1)b
k
< 4/b
k
T suy ra bt ng thc cn chng minh.
Php th lng gic
Nhiu dy s i s vi cng thc phc tp c th tr thnh cc dy s n
gin nh php th lng gic. Th thut ny c bit hiu quan trong cc bi
ton chng minh mt dy s l tun hon hay khng tun hon. p dng
c th thut ny, iu cn thit l bit cc cng thc lng gic v mt cht
nhy cm ton hc.
V d 1.21 (Vit Nam, 1990). Cho x
n
l dy s tha mn iu kin [x
1
[ < 1,
x
n+1
= (x
n
+
_
3 3x
2
n
)/2 (n 1)
a) x
1
phi tha mn iu kin g tt c cc s hng ca dy s u dng?
b) Dy s trn c tun hon khng?
iu kin [x
1
[ < 1 v dng ca hm s gi ngay cho chng ta php t
x
1
= cos vi thuc (0, ) khi x
2
= (cos + 3 sin)/2 = cos( 2/3).
T suy ra x
n+1
= cos( 2n/3). T y c th d dng tr li cc cu hi
ca bi.
V d 1.22 (KVANT). Cho dy s u
n
xc nh bi: u
1
= 2, u
n+1
= (2 +
u
n
)/(1 2u
n
).
a) Chng minh rng u
n
,= 0 vi mi n nguyn dng
b) Chng minh dy khng tun hon
Gii. t = arctan2, tan = 2. Khi nu u
n
= tanx th u
n+1
= tan(+ x),
suy ra u
n
= tan(n). S dng cng thc tan 2x = 2 tanx/(1 tan
2
x) suy ra
u
2
n
= 2u
n
/(1 u
2
n
). T y nu u
2
n
= 0 th u
n
= 0. Nu tn ti n sao cho
u
n
= 0 th s dng tnh cht ny, ta suy ra tn ti s sao cho u
2s
+ 1 = 0 hay
(2 +u
2s
)/(1 2u
2s
) = 0 hay u
2s
= 2, 2u
s
/(1 u
s2
) = 2. Suy ra u
s
v t. iu
ny v l. Phn b) l h qu ca cu a).
V d 1.23. Tm cng thc tng qut tnh s hng ca dy s x
0
= a, x
n+1
=
2 x
2
n
.
Gii. Nu [a[ 2 th t a = 2 cos , ta c x
n
= 2 cos(2n). Nu [a[ > 2,
t a = (a + 1/a) th ta c x
n
= (
2
n
+ 1/
2
n
).
V d 1.24 (Th Nh K 1997). Hai dy a
n
, b
n
c xc nh bi a
1
=
, b
1
= , a
n+1
= a
n
b
n
, b
n+1
= a
n
+b
n
. C bao nhiu cp (a, b) tha mn
a
1997
= b
1
, b
1997
= a
1
?
1.3. Mt s phng php gii bi ton v dy s 20
Gii. Ta c a
2
n+1
+ b
2
n+1
= (a
2
+ b
2
)(a
2
n
+ b
2
n
) nn yu cu bi ton xy ra ch
khi
2
+
2
= 1. t a = cos , = sin th a
n
= cos(n), b
n
= sin(n). T
suy ra li gii ca bi ton.
Php th lng gic thng c p dng trong cc bi ton c cng thc
"gi nh" n cc cng thc lng gic hoc c kt qu ging tnh cht hm
lng gic (chng hn tnh tun hon hoc tnh b chn). Tuy nhin, php th
lng gic c th xut hin nhng trng hp m tng chng khng dnh dng
g n vi lng gic.
V d 1.25. Vi mi s t nhin n > 1 v n s thc dng x
1
, x
2
, ..., x
n
t
f = maxx
1
, 1/x
1
+ x
2
, ..., 1/x
n1
+x
n
, 1/x
n
.
Hy tm minf.
Gii. Tng chng nh bi ton ny khng lin quan g n lng gic. V hn
th, cng chng lin quan g n dy s. Tuy nhin, iu kin t gi tr nh nht
ca f s to ra mt dy s! Ta chng minh rng nu x
1
, x
2
, ..., x
n
l n s thc
m ti f t min th ta phi c x
1
= 1/x
1
+ x
2
= ... = 1/x
n1
+ x
n
= 1/x
n
.
V bi ton dy s xut hin: Vi mi s nguyn dng n, xt dy s x
k

n
k=1
xc nh bi x
1
= a v x
k
= x
1
1/x
k1
, vi k = 2, ..., n. Hy tm a sao cho
1/x
n
= x
1
. V bi ton cui cng ny c th gii nh sau. t x
1
= 2 cos th
x
2
= 2 cos 1/2 cos = (4 cos
2
1)/2 cos = sin
3
/ sin
2
, x
3
= 2 cos
sin 2/ sin3 = sin4/ sin3... Tip tc nh vy suy ra x
n
= sin(n+1)/ sinn.
T ng thc 1/x
n
= x
1
sin n/ sin(n + 1) = 2 cos sin(n + 2) = 0. n
y, t iu kin x
k
dng ta suy ra = /(n + 2) v min f = 2 cos(/(n + 2)).
Cu hi:
1) Ti sao c th khng nh khi f t min th cc gi tr trn y phi bng
nhau?
2) Ti sao c th t x
1
= 2 cos ?
3) Lm sao c th d on ra cch t trn?
4) Php gii trn cn cha cht ch im no?
5) Mi s thc x u c th biu din di dng x = 2 cos hoc, x = a+1/a.
iu c ngha g?
Dy s ph
Khi kho st s hi t ca mt dy s ta thng nh l v dy n iu v b
chn. Nu dy khng n iu th c th th xt dy vi ch s chn v dy vi
ch s l. Tuy nhin, c nhng dy s c "hnh vi" phc tp hn nhiu. Chng
tng gim rt bt thng. Trong mt s trng hp nh th, ta c th xy dng
mt (hoc 2) dy s ph n iu, chng minh cc dy s ph c gii hn v
1.3. Mt s phng php gii bi ton v dy s 21
sau chng minh dy s ban u c cng gii hn. Tt nhin, dy s ph phi
c xy dng t dy s chnh.
V d 1.26. Dy s a
n
c xc nh bi a
1
> 0, a
2
> 0 v a
n+1
= 2/(a
n
+
a
n1
). Chng minh rng dy s a
n
hi t v tm gii hn ca dy s .
Gii. Xt hai dy
M
n
= maxa
n
, a
n+1
, a
n+2
, a
n+3

m
n
= mina
n
, a
n+1
, a
n+2
, a
n+3

Ta chng minh M
n
l dy s gim v m
n
l dy s tng. Tht vy, ta s chng
minh a
n+4
maxa
n+1
, a
n+3
. T y suy ra M
n+1
= a
n+1
hoc a
n+2
hoc
a
n+3
v r rng khi M
n
= maxa
n
, a
n+1
, a
n+2
, a
n+3
M
n+1
. Tht vt
nu a
n+4
a
n+3
th 2/(a
n+3
+ a
n+2
) a
n+3
suy ra 2 (a
n+3
+ a
n+2
)a
n+3
.
Khi a
n+1
= 2/a
n+3
a
n+2
= 2/a
n+3
2/(a
n+2
+ a
n+3
) a
n+2
+ a
n+4
=
2a
n+2
/(a
n+3
+ a
n+2
)a
n+3
a
n+2
+ a
n+4
a
n+4
suy ra pcm. Vy ta chng
minh c M
n
gim. Tng t m
n
tng. Hai dy s ny u b chn nn hi t.
Cui cng, ta ch cn cn chng minh hai gii hn bng nhau.
V d 1.27. Dy s a
n
c xc nh bi a
1
> 0, a
2
> 0 v a
n+1
=

a
n
+

a
n1
. Chng minh rng dy s a
n
hi t v tm gii hn ca dy s .
Gii. Xt dy s M
n
= maxa
n
, a
n+1
, 4.
Nu M
n
= 4 th a
n
, a
n+1
4, suy ra a
n+2
4, t M
n+1
= 4.
Nu M
n
= a
n+1
th a
n+1
a
n
, 4. Khi

a
n1
= a
n+1

a
n+1


a
n+1
,
suy ra a
n+2
=

a
n
+

a
n+1

a
n
+

a
n1
= a
n+1
suy ra M
n+1
= maxa
n+1
, a
n+2
, 4 =
a
n+1
.
Nu M
n
= a
n
th a
n
a
n+1
, 4. Khi a
n+2
=

a
n
+

a
n+1
2

a
n
. Suy ra
M
n+1
a
n
= M
n
.
Vy trong mi trng hp th M
n+1
M
n
, tc l dy M
n
l dy s gim.
Do M
n
b chn di bi 4 nn dy ny c gii hn. Ta chng minh gii hn ny
bng 4. Thc vy, gi s gii hn l M > 4. Khi vi mi > 0, tn ti N sao
cho vi mi n N th M < M
n
< M +. Chn n N sao cho M
n+2
= a
n+2
(theo cc lp lun trn v do M > 4 th tn ti ch s n nh vy). Ta c
M < M
n+2
= a
n+2
=

a
n
+

a
n1
< 2

M +
hay M(M 4) (2M + 4 ) < 0
Mu thun v M > 4 v c th chn nh tu .
1.3. Mt s phng php gii bi ton v dy s 22
Phng php sai phn
tnh tng n s hng u tin ca mt dy s, mt trong nhng phng
php hiu qu nht l phng php sai phn: tnh tng n s hng u tin
ca dy s a
n
, ta tm hm s f(n) sao cho a
n
= f(n + 1) f(n). Khi
a
0
+ +a
n1
= f(n) f(0).
Mt trong nhng v d kinh in chnh l phng php m Bernoulli v
cc nh ton hc th k 18 a ra tm cng thc tnh tng S(k, n) =
1
k
+ 2
k
+ +n
k
. Dng phng php h s bt nh, h tm a thc f
k
(n) sao
cho n
k
= f
k
(n+1)f
k
(n) v t tm c S(k, n) = f
k
(n+1)f
k
(n). Phng
php ny hiu qu hn phng php xy dng cng thc truy hi, v tnh S
k
ta khng cn phi dng n cc cng thc tnh S
k1
, S
k2
Khi d on cc hm f, ta c th s dng tch phn ri tng t ha qua.
V d tch phn ca a thc bc k l a thc bc k + 1. Vy th f
k
= n
k
suy
ra f
k
phi c bc k + 1.
Tuy nhin, khc vi tch phn, i khi cc hm ri rc khng c "nguyn
hm". Trong trng hp ta khng tnh c tng m ch c th nh gi tng
bng cc bt ng thc.
V d 1.28. Tm phn nguyn ca tng S = 1/1 + 1/

2 + + 1/

100.
Gii. Ta cn tm mt nh gi cho S. Nhn xt rng hm 1/

x c nguyn hm
l 2

x, ta xt hm s f(n) = 2

n. Khi f(n +1) f(n) = 2

n + 1 2

n =
2/(

n + 1 +

n).
Suy ra, 1/

n + 1 < f(n + 1) f(n) < 1/

n. T , 2(

101 1) < S <


2(

100 1) + 1, suy ra [S] = 18.


V d 1.29 ( ngh Ton quc t 2001). Cho x
1
, x
2
, ..., x
n
l cc s
thc bt k. Chng minh rng
x
1
/(1 +x
2
1
) + x
2
/(1 +x
2
1
+ x
2
2
) + +x
n
/(1 +x
2
1
+ +x
2
n
) <

n.
Gii. t v tri ca bt ng l A. p dng bt ng thc Bunhiacopsky ta
c
A
2
n[x
2
1
/(1 +x
2
1
)
2
+x
2
2
/(1 + x
2
1
+ x
2
2
)
2
+ +x
2
n
/(1 +x
2
1
+ +x
2
n
)
2
]
chng minh bt ng thc u bi, ta ch cn chng minh
x
2
1
/(1 + x
2
1
)
2
+x
2
2
/(1 + x
2
1
+ x
2
2
)
2
+ +x
2
n
/(1 +x
2
1
+ +x
2
n
)
2
< 1.
Nhng iu ny l hin nhin do bt ng thc
x
2
k
/(1 +x
2
1
+ +x
2
k
)
2
1/(1 + x
2
1
+ + x
2
k1
) 1/(1 + x
2
1
+ +x
2
k
).
1.4. Mt s phng php xy dng h thng bi tp 23
V d 1.30. Xt dy s x
n

n=1
cho bi: x
n+2
= [(n 1)x
n+1
+ x
n
]/n. Chng
minh rng vi mi gi tr ban u x
1
, x
2
, dy s cho hi t. Tm gii hn ca
dy nh mt hm s theo x
1
, x
2
.
Gii. Ta c t cng thc ca dy s x
n+2
x
n+1
= (x
n+1
x
n
)/n = (x
n

x
n1
)/n(n1) = = (1)
n
(x
2
x
1
)/n!. T suy ra x
n+2
= (x
n+2
x
n+1
) +
(x
n+1
x
n
) + +(x
2
x
1
) +x
1
= x
1
+(x
2
x
1
)K
n
, trong K
n
= 1 1/1! +
1/2! + (1)n/n!. T y suy ra dy s c gii hn v gii hn bng
x
1
+ (x
2
x
1
)/e.
Cu hi:
1) C th tng qut ha bi ton trn nh th no?
2) Hy tm sai phn ca cc hm s arctan(n). T t ra bi ton tnh
tng tng ng.
3) Tm sai phn ca hm s ln(n). T tm nh gi cho tng 1 + 1/2 +
+ 1/n.
4) T cng thc sin3x = 3 sinx 4 sin3x c th lp ra cng thc tnh tng
no?
1.4 Mt s phng php xy dng h thng bi tp
1.4.1 Xy dng dy hi t bng phng trnh
C th xy dng dy s hi t v mt s a xut pht t mt phng trnh c
nghim l a theo cch sau:
V d 1.31. Xt a =

2, l nghim ca phng trnh


2
= 2. Ta vit li di
dng
= 2/ 2 = + 2/ = ( + 2/)/2
v ta thit lp dy s x
n
tho mn x
0
= a, x
n+1
= (x
n
+ 2/x
n
)/2. Nu dy ny
hi t th gii hn s l

2. Tng t nh vy, ta c th xy dng c dy s
tin v cn bc k ca m nh sau:
x
0
= a, x
n+1
= (x
n
+m/x
k1
n
)/2
Cng vi gii hn cn n l

2, ta c th xy dng mt dy s khc theo
"phong cch" nh vy:
x
0
= a, x
n+1
= 1 + x
n
x
2
n
/2
Tt nhin, trong tt c cc v d trn, ta ch c c phng trnh vi nghim
theo mun khi chng minh c s hi t ca dy s. V vy, cn cn thn
vi cch thit lp bi ton kiu ny. V d, vi dy s x
n+1
= 1 + x
n
x
2
n
/2 th
khng phi vi x
0
no dy cng hi t, v khng phi lc no gii hn cng l.
1.4. Mt s phng php xy dng h thng bi tp 24
Mt cch tng qut, ta c th dng phng php tm nghim xp x Newton
xy dng cc dy s. tm nghim ca phng trnh F(x) = 0, phng php
Newton ngh chn x
0
tng i gn nghim v xy dng dy truy hi
x
n+1
= x
n
F(x
n
)/F

(x
n
)
khi dy x
n
s dn n nghim ca phng trnh F(x) = 0.
V d 1.32. Xt hm s F(x) = x
2
2, th F(x)/F

(x) = (x
2
2)/2x v ta
c dy s x
n+1
= (x
n
+ 2/x
n
)/2.
Xt hm s F(x) = x
3
x th F(x)/F

(x) = (x
3
x)/(3x
2
1) v ta c dy
s
x
n+1
= 2x
3
n
/(3x
2
n
1)
1.4.2 Xy dng dy truy hi t cp nghim ca phng trnh
bc 2
Chng ta thy, t hai nghim ca mt phng trnh bc 2 c th xy dng
ra cc dy truy hi tuyn tnh bc 2 (kiu dy s Fibonacci). Tng t nh th,
c th xy dng cc dy truy hi tuyn tnh bc cao t nghim ca cc phng
trnh bc cao. Trong phn ny, chng ta s i theo mt hng khc: xy dng
cc dy truy hi phi tuyn bc nht t cp nghim ca phng trnh bc 2.
Xt phng trnh bc 2: x
2
mx1 = 0 c hai nghim l v . Xt mt s
thc a bt k. Xt dy s x
n
= a(
2
n
+
2
n
). Khi x
2
n
= a
2
(
2
n+
+
2
n+1
+2) =
ax
n+1
+ 2a
2
, t suy ra dy s x
n
tho cng thc truy hi: x
n+1
= x
2
n
/a 2a.
V d chn a = 1/2, m = 4, ta c bi ton: Tm cng thc tng qut ca dy
s x
n
c xc nh bi x
0
= 2, x
n+1
= 2x
2
n
1.
Tng t nh vy, nu xt x
n
= a(
3
n
+
3
n
) th x
3
n
= a
3
(
3
n+1
+
3
n+1

3(
3
n
+
3
n
) = a
2
(x
n+1
3x
n
). T suy ra dy s x
n
tho cng thc truy hi
x
n+1
= x
3
n
/a
2
(3x
n
).
V d xt , l hai nghim ca phng trnh x
2
4x 1 = 0, a = 1/4,
ta c bi ton: Tm cng thc tng qut ca dy s x
n
c xc nh bi
x
0
= 1, x
n+1
= 16x
3
n
+ 3x
n
. Hon ton tng t, c th xy dng cc dy truy
hi phi tuyn dng a thc bc 4, 5. Bng php di trc, ta c th thay i dng
ca cc phng trnh ny.
V d 1.33. nu trong dy x
0
= 2, x
n+1
= 2x
2
n
1 ta t x
n
= y
n
1/2 th ta
c dy y
n
tho: y
0
= 5/2, y
n+1
= 2(y
2
n
y
n
).
Nu , l cc s thc th trong hai s c t nht mt s c tr tuyt i ln
hn 1, v vy dy s khng hi t (Tr trng hp hai nghim i nhau v dy
l dy hng). Tuy nhin, nu chn , l cp s phc lin hp c mun nh
hn hay bng 1, ta c th to ra cc dy tun hon hoc dy hi t. Ch rng
1.4. Mt s phng php xy dng h thng bi tp 25
chn , y chnh l chn m v cng chnh l chn x
0
. Do tnh cht ca
dy s s ph thuc rt nhiu vo x
0
.
V d vi dy s tho x
n+1
= 2x
2
n
1, nu x
0
= 2 th x
n
= [(2 +

3)
2
n
+(2

3)
2
n
]/2; nu x
0
= 1 th x
n
l dy hng; nu x
0
= cos th x
n
= cos(2
n
).
Cu hi:
1) Xt xem vi nhng a, b, c no th phng trnh sai phn x
n+1
= ax
2
n
+bx
n
+c
gii c bng phng php trn?
2) Hy tm dng ca cc dy truy hi to c bng cch xt x
n
= a(
k
n
+
k
n
)
vi k = 4, 5.
1.4.3 Xy dng cc dy s nguyn t li gii cc phng trnh
nghim nguyn
Mt dy truy hi tuyn tnh vi h s nguyn v cc s hng u u nguyn
s cha ton s nguyn. l iu hin nhin. Th nhng c nhng dy s m
trong cng thc truy hi c phn s, thm ch c c cn thc nhng tt c cc
s hng ca n vn nguyn. y mi l iu bt ng. Tuy nhin, nu xem xt
k, ta c th thy chng c mt mi quan h rt trc tip.
Chng ta hy bt u t bi ton quen thuc sau: Chng minh rng mi s
hng ca dy s a
n
xc nh bi a
0
= 1, a
n+1
= 2a
n
+
_
3a
2
n
2 u nguyn.
Chuyn v v bnh phng cng thc truy hi, ta c
a
2
n+1
4a
n+1
a
n
+ 4a
2
n
= 3a
2
n
2
a
2
n+1
4a
n+1
a
n
+a
2
n
+ 2 = 0
Thay n bng n 1, ta c
a
2
n
4a
n
a
n1
+a
2
n1
+ 2 = 0
T y suy ra a
n1
, a
n+1
l hai nghim ca phng trnh
x
2
4a
n
x + a
2
n
+ 2 = 0
Suy ra: a
n+1
+ a
n1
= 4a
n
hay a
n+1
= 4a
n
a
n1
. T y suy ra tt c cc s
hng trong dy u nguyn.
C cng thc ban u ln cng thc h qu a
n+1
= 4a
n
a
n1
u gi cho
chng ta n vi phng trnh Pell. Qu tht l c th xy dng hng lot dy
s tng t bng cch xt phng trnh Pell.
Xt phng trnh x
2
Dy
2
= k. Gi s phng trnh c nghim khng tm
thng (x
0
, y
0
) v (, ) l nghim c s ca phng trnh x
2
Dy
2
= 1. Khi ,
nu xt hai dy x
n
, y
n
xc nh bi x
n+1
= x
n
+Dy
n
, y
n+1
= x
n
+ y
n
th x
n
, y
n
l nghim ca x
2
Dy
2
= k.
1.4. Mt s phng php xy dng h thng bi tp 26
T h phng trnh trn, ta c th tm c
x
n+1
= x
n
+
_
D(x
2
n
k); y
n+1
= y
n
+
_
k + Dy
2
n
v nh vy xut hin hai dy s nguyn c cho bi mt cng thc khng
nguyn.
V d, vi D = 4a(a + 1), k = 1 th ta c x
0
= = 2a + 1, y
0
= = 1. Ta
c hai dy s nguyn sau y:
x
0
= 2a + 1, x
n+1
= 2a + 1 +
_
4a(a + 1)(x
2
n
1)
y
0
= 1, y
n+1
= 2a + 1 +
_
4a(a + 1)y
2
n
+ 1
Cui cng, ch rng ta c th to ra mt kiu dy s khc t kt qu a
n1
, a
n+1
l hai nghim ca phng trnh
x
2
4a
n
x + a
2
n
+ 2 = 0
trn y: Theo nh l Viet th a
n+1
a
n1
= a
2
n
+ 2, suy ra
a
n+1
= (a
2
n
+ 2)/a
n1
v ta c bi ton: Cho dy s a
n
xc nh bi a
0
= 1, a
1
= 3 v a
n+1
=
(a
2
n
+ 2)/a
n1
. Chng minh rng a
n
nguyn vi mi n.
1.4.4 Xy dng dy s l nghim ca mt h phng trnh ph
thuc bin n
Xt mt h phng trnh F(n, x) = 0. Nu vi mi n, phng trnh F(n, x) =
0 c nghim duy nht trn mt min D no th dy s x
n
c xc nh.
T mi lin h gia cc hm F(n, x), dy s ny c th c nhng tnh cht rt
th v.
V d 1.34. Vi mi s t nhin n 3, gi x
n
l nghim dng duy nht ca
phng trnh x
n
x
2
x1 = 0. Chng minh rng limx
n
= 1 v tm limn(x
n
1).
V d 1.35. Chng minh rng vi mi n nguyn dng, phng trnh
1/x + 2/(x 1) + 2/(x 4) + + 2/(x n
2
) = 0
c nghim duy nht x
n
thuc khong (0, 1). Tm lim
n
x
n
.
V d 1.36. Chng minh rng vi mi n nguyn dng, phng trnh
1/x + 2/(x 1) + 2/(x 4) + + 2/(x n
2
) = 0
c nghim duy nht x
n
thuc (0, 1). Tm lim
n
x
n
.
1.5. L thuyt dy s di con mt ton cao cp 27
to ra cc phng trnh c nghim duy nht trn mt khong no , c
th s dng tng ca cc hm n iu. Ring vi hm a thc ta c th s dng
quy tc -cc v s nghim dng ca phng trnh: Nu dy cc h s ca
phng trnh i du k ln th phng trnh c khng qu k nghim dng.
V d phng trnh x
4
x
2
nx 1 = 0 c nghim dng duy nht x
0
, cn
phng trnh x
4
x
2
+nx 1 = 0 c nhiu nht hai nghim dng.
Khi xy dng cc hm F(n, x), c th s dng cng thc truy hi. Nh trong
v d trn th F(n +1, x) = F(n, x) +1/(x n 1). Xy dng F(n, x) kiu ny,
dy nghim x
n
s d c nhng quy lut th v hn. V d, vi dy s trn, ta c
F(n + 1, x
n
) = F(n, x
n
) + 1/(x
n
n 1) < 0. T y, do F(n + 1, 0
+
) = ta
suy ra x
n+1
nm gia 0 v x
n
, tc dy x
n
gim.
Cu hi:
1) C th xy dng dy s no vi h hm s F(x) = x(x 1) . . . (x n)?
2) Cho 0 < a
1
< a
2
< < a
n
< l mt dy s dng tng nghim ngt.
Xt h phng trnh 1/x+1/(x
1
a
1
) + +1/(xa
n
) = 0 c nghim duy nht
x
n
thuc (0, a
1
). Khi no th x
n
dn v 0 khi n dn n v cng?
1.5 L thuyt dy s di con mt ton cao cp
1.5.1 Ri rc ha cc khi nim v nh l ca l thuyt hm
bin s thc
Dy s l hm s, do n c y cc tnh cht chung ca hm s. Tuy
nhin, do tnh cht c bit ca N, mt s khi nim nh o hm, tch phn
khng c nh ngha cho cc dy s. Nhng thc ra, dy s cng c cc khi
nim tng ng vi cc khi nim ny. Bng cch so snh v php tng t, ta
c th tm c nhng nh l th v ca l thuyt dy s. l qu trnh ri
rc ha.
Ri rc ha ca o hm f

(x) chnh l sai phn x


n
= x
n
x
n1
ca dy s.
Cng nh o hm ca hm bin s thc, sai phn dng xt tnh tng gim
ca dy s. Tng t nh vy, ta nh ngha sai phn cp 2 v dng o tnh
li lm ca dy. Ri rc ha ca khi nim tch phn chnh l khi nim tng:
S(x
n
) = x
0
+ +x
n
. Hai khi nim ny ngc nhau: (S(x
n
)) = x
n
, S(x
n
) =
x
n
.
V d 1.37 (nh l Stolz). Xt hai dy s x
n
v y
n
trong y
n
l dy
s dng tng v dn n v cng. Th th limx
n
/y
n
= lim(x
n
x
n1
)/(y
n
y
n1
)
vi gi thit l gii hn v phi tn ti. (So snh vi quy tc LHopitale)
Chng minh: t lim(x
n
x
n1
)/(y
n
y
n1
) = A. Vi mi > 0 tn ti
N
1
sao cho vi mi n N
1
ta c [(x
n
x
n1
)/(y
n
y
n1
) A[ < , suy ra
1.5. L thuyt dy s di con mt ton cao cp 28
A < (x
n
x
n1
)/(y
n
y
n1
) < A+ . T y, do y
n
l dy tng nn ta c
(A )(y
N
1
y
N
1
1
) < x
N
1
x
N
1
1
< (A+)(y
N
1
y
N
1
1
)
. . .
(A )(y
n
y
n1
) < x
n
x
n1
< (A+ )(y
n
y
n1
)
Cng cc bt ng thc trn li, ta c
(A )(y
n
y
N
1
1
) < x
n
x
N
1
1
< (A+)(y
n
y
N
1
1
)
Chia hai v cho y
n
, ta c
A + [x
N
1
(A)y
N
1
1
]/y
n
< x
n
/y
n
< A+ + [x
N
1
(A+ )y
N
1
1
]/y
n
V y
n
dn n v cng nn tn ti N
2
> N
1
sao cho
[x
N
1
(A)y
N
1
1
]/y
n
> v [x
N
1
(A+ )y
N
1
1
]/y
n
<
vi mi n N
2
. Khi vi mi n N
2
ta c A 2 < x
n
/y
n
< A + 2 v iu
ny c ngha l limx
n
/y
n
= A.
Cu hi: iu kin y
n
tng v dn n v cng c cn thit khng?
V d 1.38. Chng minh rng nu dy s x
n
tho mn iu kin x
n+1
2x
n
+
x
n1
0 v k
1
, k
2
, . . . , k
r
l cc s t nhin tho mn iu kin k
1
+k
2
+ +k
r
=
r.k th
x
k
1
+ + x
kr
r.x
k
(So snh vi bt ng thc Jensen)
V d 1.39. Cho dy s x
n
tho mn iu kin x
k+1
2x
k
+ x
k1
0 vi
mi k = 1, . . . , n. Ngoi ra x
0
= x
n+1
= 0. Chng minh rng x
k
0 vi mi
k = 1, . . . , n.
(o hm bc 2 khng m, suy ra o hm bc nht l hm tng v ch c
nhiu nht 1 nghim, suy ra chiu bin thin ca hm s ch c th l 0 gim
cc tiu ri tng 0)
V d 1.40. Cho dy s dng a
n
. Bit rng tn ti gii hn
lim
n
n

k=1
1
a
k
= A < .
t s
n
= a
1
+ a
2
+ +a
n
. Chng minh rng tng lim
n

n
k=1
k
2
a
k
/s
2
k
cng
c gii hn hu hn khi n .
1.5. L thuyt dy s di con mt ton cao cp 29
Gii. Dch sang ngn ng hm s, ta c bi ton sau "Nu f(x) l hm s tng
t R
+
vo R
+
v tn ti tch phn suy rng
_

0
dx
f(x)
th cng tn ti tch phn
_

0
x
2
f(x)dx
F
2
(x)
trong F(x) l nguyn hm ca f(x)". Bi ny c th gii bng
phng php tch phn tng phn nh sau:
_
A
0
xdx
F(x)
=
1
2
_
A
0
d(x
2
)
F(x)
=
1
2
_
x
2
F(x)

A
0
+
_
A
0
x
2
f(x)dx
F
2
(x)
_
nh vy ch cn chng minh tn ti
_

0
xdx
F(x)
v lim
x
x
2
F(x)
.
Cu hi:
1) nh l Rolle c dng ri rc nh th no?
2) Cng thc tnh tch phn tng phn c dng ri rc nh th no?
1.5.2 Phng php hm sinh v bi ton tm s hng tng qut
Cho dy s a
0
, a
1
, . . . , a
n
, . . . Hm sinh F(x) ca dy s ny l biu thc hnh
thc
F(x) = a
0
+ a
1
x + +a
n
x
n
+
Cc php ton trn hm sinh c thc hin mt cch t nhin v chng ta
khng quan tm n tnh cht gii tch ca chng (bn knh hi t ca chui
tng ng c th bng 0). Php ton c bit nht ca hm sinh l php nhn:
Nu F(x), G(x) l hm sinh ca cc dy a
n
, b
n
tng ng th F(x).G(x)
l hm sinh ca dy c
n
trong c
n
=

n
0
a
i
b
ni
.
S ng dng ca hm sinh vo bi ton tm s hng tng qut ca dy s
nh sau: Gi s ta cn tm s hng tng qut ca dy s a
n
cho bi mt cng
thc truy hi no . Ta thit lp hm sinh F(x) ca a
n
. Da vo h thc
truy hi, ta tm c mt phng trnh cho F(x), gii phng trnh, ta tm c
F(x). Khai trin F(x) theo lu tha x (Khai trin Taylor), ta tm c a
n
vi
mi n.
V d 1.41. Tm s hng tng qut ca dy s a
n
xc nh bi: a
0
= 3, a
1
=
2, a
n+2
= 5a
n+1
6a
n
.
Gii. Xt hm sinh F(x) = a
0
+a
1
x + a
2
x
2
+ +a
n+2
x
n+2
+ . Vi mi n
t nhin, ta thay a
n+2
bng 5a
n+1
6a
n
th c
F(x) = a
0
+a
1
x + (5a
1
6a
0
)x
2
+ + (5a
n+1
6a
n
)x
n+2
+
= a
0
+a
1
x + 5x(a
1
x + + a
n+1
x
n+1
+ ) 6x
2
(a
0
+ a
1
x + +a
n
x
n
+ )
= a
0
+a
1
x + 5x(F(x) a
0
) 6x
2
F(x)
1.5. L thuyt dy s di con mt ton cao cp 30
Suy ra F(x) = (3 13x)/(6x
2
5x + 1) = 7/(1 2x) 4/(1 3x) = 7(1 + 2x +
(2x)
2
+ + (2x)
n
+ ) 4(1 + 3x + (3x)
2
+ + (3x)
n
+ )
T a
n
= 7.2
n
4.3
n
.
Trn l thuyt, khi tm c F(x), ta phi dng cng thc Taylor tm khai
trin ca F(x). y l mt bi ton phc tp. Tuy nhin, trong nhiu trng
hp, cng thc nh thc Newton tng qut di y dng:
(1 +x)

= 1 + x + [( 1)/2]x
2
+ + [( 1) . . . ( n + 1)/n!]x
n
+
V d 1.42. Dy s a
n
xc nh bi a
0
= 1, a
0
a
n
+ a
1
a
n1
+ + a
n
a
0
= 1
vi mi n. Hy tm cng thc tng qut ca a
n
.
Gii. Xt hm sinh F(x) = a
0
+ a
1
x + a
2
x
2
+ + a
n
x
n
+ . T cng
thc truy hi ta suy ra F
2
(x) = 1 + x + x
2
+ + x
n
+ = (1 x)
1
. T
y F(x) = (1 x)
1/2
. Khai trin F(x) theo cng thc Newton, ta tm c
a
n
= C
n
2n
/2
2n
.
1.5.3 i s tuyn tnh v phng trnh sai phn
Trong phn trn, chng ta s dng phng php hm sinh gii bi ton
tm cng thc tnh s hng tng qut ca mt dy s. Trong phn ny, ta s xem
xt cu trc nghim ca phng trnh sai phn di gc i s tuyn tnh.
Xt phng trnh sai phn thun nht: x
n+k
= a
1
x
n+k1
+ + a
k
x
n
. D
thy rng nu dy s x
n
, y
n
tho mn phng trnh ny th ax
n
+by
n
cng
tho mn phng trnh vi mi a, b. Nh vy tp hp tt c cc dy s tho mn
phng trnh sai phn trn lp thnh mt khng gian vc-t. Hn th, ta c nh
l:
nh l 1.12. Tp hp tt c cc dy s tho mn phng trnh sai phn
x
n+k
= a
1
x
n+k1
+ + a
k
x
n
.
l mt khng gian vct k chiu.
Chng minh nh l ny kh n gin: Dy s s hon ton xc nh nu
bit k s hng u tin. Gi x
i
n
(i = 0, k 1) l dy s c x
i
j
= 0 nu i ,= j v
x
i
i
= 1. Khi c th chng minh d dng rng cc dy x
1
n
, . . . , x
k
n
c lp
tuyn tnh v vi mi dy x
n
ta c
x
n
= x
0
x
0
n
+ + x
k1
x
k1
n
Nh th, cu trc nghim ca phng trnh sai phn tuyn tnh thun nht
l r. Ta ch cn tm mt c s no ca khng gian nghim l c th m
t c tt c cc nghim ca phng trnh sai phn. C s m chng ta a ra
trn khng c tnh tng minh, do kh c th s dng trong vic thit lp
cng thc tng qut. xy dng mt c s khc tt hn, ta c nh l:
1.5. L thuyt dy s di con mt ton cao cp 31
nh l 1.13. Nu l nghim bi r ca phng trnh c trng
x
k
a
1
x
k1
a
k
= 0
th cc dy s
n
, . . . , n
r1

n
tho mn phng trnh sai phn x
n+k
=
a
1
x
n+k1
+ + a
k
x
n
.
Vi nh l ny, ta c th tm k dy s tng minh to thnh mt c s
ca khng gian nghim.
Cui cng, nu ta gp phng trnh sai phn tuyn tnh khng thun nht
x
n+k
= a
1
x
n+k1
+ +a
k
x
n
+ f(n).
th nghim tng qut ca phng trnh ny s c dng l tng ca nghim tng
qut ca phng trnh sai phn tuyn tnh thun nht tng ng vi mt nghim
ring ca phng trnh khng thun nht.
tm nghim ring, ta vn dng phng php hon ton tng t nh trong
phng trnh vi phn: Nu f(n) l a thc th ta x
n
tm di dng a thc, l
hm m th tm di dng hm m... y, trng hp c s l nghim kp ca
phng trnh c trng cng c x l tng t nh trong phng trnh vi phn.
1.5.4 S dng xp x trong d on kt qu
Trong nhiu trng hp, d on c kt qu l mt na, thm ch 2/3
li gii. Chng ta gp nhiu tnh hung l li gii u tin thu c mt cch
rt kh khn, nhng sau th hng lot li gii p hn, gn hn xut hin. V
sao chng ta khng ngh ngay c nhng li gii p? V chng ta cha bit p
s. Khi bit ri th c th nh hng d dng hn rt nhiu. Di y, chng ta
s xem xt mt s ng dng ca xp x trong vic d on kt qu.
Trong v d v dy s x
n+1
= sin(x
n
), chng ta p dng nh l trung
bnh Cesaro tm gii hn

nx
n
, mc d dy s khng c dng quen thuc
x
n+1
= x
n
(x
n
)

. Th nhng, nu rng x
n
0 khi n , m ti ln
cn 0 th sinx x x
3
/6 th ta s thy tnh quy lut ca kt qu tm c
trn.
Vi phng php tng t, ta c th thy dy dng x
n+1
= x
n
(x
n
)

hng lot cc dy s c b ngoi khc hn nh: x


n+1
= ln(1 + x
n
), x
n+1
=
x
n
cos x
n
, x
n+1
= arctg(x
n
) . . . (D nhin, phi kim tra iu kin x
n
0 khi
n ).
Ta cng c th gii thch c v sao trong bi ton a
n+1
= a
n
+1/

a
n
phn
trn, ta tmc s 3/2. Ta c a
n+1
= a
n
+1/

a
n
= a
n
(1+1/a
3/2
n
). V a
n

khi n nn vi mi ta c a

n+1
= a

n
(1 + 1/a
3/2
n
)

n
(1 + /a
3/2
n
) =
a

n
+ a
3/2
n
. Do hiu s ny xp x hng s, ta chn b = 3/2.
Ta xt mt v d khc
1.6. Bi tp 32
V d 1.43 (HSP, 2000). Cho dy s a
n
xc nh bi: a
1
= a
2
= 1, a
n+1
=
a
n
+a
n1
/n(n + 1). Chng minh rng dy a
n
c gii hn.
Gii. D thy a
n
l dy tng. V vy ta ch cn chng minh dy a
n
b chn
trn. Ta c
a
n+1
= a
n
+a
n1
/n(n + 1) < a
n
[1 + 1/n(n + 1)]
T y suy ra
a
n+1
< [1 + 1/n(n + 1)] . . . [1 + 1/2.3]a
2
= [1 + 1/n(n + 1)] . . . [1 + 1/2.3]
Nh vy ta ch cn chng minh tch [1 + 1/n(n + 1)] . . . [1 + 1/2.3] b chn. Kt
qu ny khng phc tp v c th chng minh hon ton s cp. Tuy nhin,
nhng kinh nghim v dy s 1/n(n + 1) gi cho chng ta ti mi quan h gia
tch trn v tng 1/2.3 + +1/n(n+1). Theo hng , chng ta c th a ra
mt kt qu tng qut hn v kt qu c d on t vic s dng xp x.
Gi s rng x
n
l dy s thc sao cho tng x
1
+ +x
n
c gii hn hu hn
khi n . Khi x
n
0 khi n . V vy, vi n ln th x
n
ln(1 +x
n
).
Do tng ln(1 +x
1
) + +ln(1 +x
n
) cng c gii hn hu hn khi n v
c ngha l tch (1 +x
1
) . . . (1 + x
n
) cng vy. Ta c nh l
nh l 1.14. Cho dy s thc x
n
. Khi nu tng x
1
+ +x
n
c gii hn
hu hn khi n th tch (1 + x
1
) . . . (1 + x
n
) cng c gii hn hu hn khi
n .
Cu hi:
1) Mnh o ca nh l trn c ng khng?
2) Cho n > 3 v x
n
l nghim dng duy nht ca phng trnh x
n
x
2

x 1 = 0. C th d on c lim
n
n(x
n
1)?
1.6 Bi tp
Bi 1.1 (Canada 1998). Cho m l s nguyn dng. Xc nh dy a
0
, a
1
, a
2
, . . .
nh sau: a
0
= 0, a
1
= m v a
m+1
= m
2
a
n
a
n1
vi n = 1, 2, . . . Chng minh
rng vi mi cp sp th t cc s t nhin (a, b) vi a b l nghim ca phng
trnh (a
2
+b
2
)/(ab+1) = m
2
khi v ch khi (a, b) = (a
n
, a
n+1
) vi n l mt s t
nhin no .
Bi 1.2 (Bulgari 1978). Cho dy s a
n
xc nh bi a
n+1
= (a
2
n
+c)/a
n1
.
Chng minh rng nu a
0
, a
1
v (a
2
0
+a
2
1
+c)/a
0
a
1
l s nguyn th a
n
nguyn vi
mi n.
Bi 1.3. Trong mt dy v hn cc s nguyn dng, mi mt s hng sau ln
hn s hng trc hoc l 54 hoc l 77. Chng minh rng trong dy ny tn
ti s hng c hai ch s tn cng ging nhau.
1.6. Bi tp 33
Bi 1.4 (Sc-Slovakia 1997). Chng minh rng tn ti dy s tng a
n

n=1
cc s nguyn dng sao cho vi mi s t nhin k, dy k + a
n
cha hu hn
s nguyn t.
Hng dn: Dng nh l Trung hoa v s d.
Bi 1.5 (Putnam 1995). t S() = [n][n = 1, 2, 3, . . .. Chng minh
rng tp hp cc s nguyn dng N

khng th phn hoch thnh 3 tp hp


S(), S(), S().
Bi 1.6 (Putnam 1999). Dy s a
n

n=1
c xc nh bi a
1
= 1, a
2
= 2, a
3
=
24 v vi n 4.
a
n
= (6a
2
n1
a
n3
8a
n1
a
2
n2
)/a
n2
a
n3
Chng minh rng vi mi n, a
n
l s nguyn chia ht cho n.
Bi 1.7. Trong dy s nguyn dng a
k

k=1
tng ca 10 s hng u tin bng
100, cn t a
11
, mi a
n
bng s cc ch s i < n sao cho a
i
+ i n. Bit rng
a
11
= 10. Chng minh rng k t mt ch s no , tt c cc s hng ca dy
bng nhau.
Bi 1.8 (Balkan). Cho x
0
x
1
x
2
x
n
l dy s khng gim
cc s t nhin sao cho vi mi s t nhin k, s cc s ca dy ny khng vt
qu k l hu hn (v k hiu l y
k
). Chng minh rng vi mi m, n
n

0
x
i
+
m

0
y
i
(n + 1)(m+ 1)
Bi 1.9 (Bulgari 87). Xt dy s x
n
xc nh bi x
1
= x
2
= 1, x
n+2
=
14x
n+1
x
n
4. Chng minh rng vi mi n, x
n
l bnh phng ca mt s
nguyn.
Hng dn: Xt dy u
1
= u
2
= 1, u
n+2
= 4u
n+1
u
n
. Chng minh rng
u
n+2
u
n
u
2
n+1
= 2 sau chng minh rng x
n
= u
2
n
. C th dng tng bi
ny xy dng cc bi ton khc nh th no?
Bi 1.10 (Canada 1988). Cho hai dy s x
n
, y
n
xc nh bi x
n+1
=
4x
n
x
n1
, x
0
= 0, x
1
= 1 v y
n+1
= 4y
n
y
n1
, y
0
= 1, y
1
= 2. Chng minh
rng vi mi n, y
2
n
= 3x
2
n
+ 1.
Bi 1.11 (Canada 1993). Cho y
1
, y
2
, y
3
, . . . l dy s xc nh bi y
1
= 1 v
vi mi s nguyn dng k
y
4k
= 2y
2k
, y
4k+1
= 2y
2k
+ 1, y
4k+2
= 2y
2k+1
+ 1, y
4k+3
= 2y
2k+1
Chng minh rng dy s y
1
, y
2
, y
3
. . . nhn tt c cc gi tr nguyn dng, mi
gi tr ng mt ln.
1.6. Bi tp 34
Bi 1.12. Gi s rng s
n
l dy s nguyn dng tho mn iu kin 0
s
n+m
s
n
s
m
K vi K l mt s nguyn dng cho trc. Vi s nguyn
dng N c tn ti cc s thc a
1
, a
2
, . . . , a
K
sao cho
s
n
= [a
1
n] + + [a
K
n] vi mi n = 1, 2, ...N?
Bi 1.13. Cho a
1
= 1, b
1
= 2, c
1
= 3. Gi S(n) l tp hp cc s nguyn dng
a
i
, b
i
, c
i
vi i n. Xy dng a
n
, b
n
, c
n
nh sau:
a
n+1
= s nguyn dng nh nht khng thuc S(n);
b
n+1
= s nguyn dng nh nht khng thuc S(n) v khc a
n+1
;
c
n+1
= a
n+1
+ b
n+1
;
Gi d
k
l dy tng cc ch s n sao cho b
n
= a
n
+ 2. Chng minh rng
a) d
k
/k 6 khi k dn n v cng
b) Nu B l s nguyn th (d
k
6k)/2 = B vi v s cc ch s k.
Bi 1.14 (AMM). Cc dy s a
n
, b
n
, c
n
c xc nh nh sau: a
1
=
1, b
1
= 2, c
1
= 4 v
a
n
= s nguyn dng nh nht khng thuc a
1
, . . . , a
n1
, b
1
, . . . , b
n1
, c
1
, . . . , c
n1

b
n
= s nguyn dng nh nht khng thuc a
1
, . . . , a
n1
, a
n
, b
1
, . . . , b
n1
, c
1
, . . . , c
n1

c
n
= 2b
n
+na
n
. Hy chng minh hoc ph nh rng 0 < n(1+

3)b
n
< 2
vi mi n.
Bi 1.15 (AMM). Cho a
1
= 1 v a
n+1
= a
n
+ [

a
n
] vi n = 1, 2, . . . Chng
minh rng a
n
l s chnh phng khi v ch khi n = 2
k
+k 2 vi k l s nguyn
dng no .
Bi 1.16 (Bulgari 1973). Cho dy s a
n

n=1
c xc nh bi a
1
= 2, a
n+1
=
a
2
n
a
n
+ 1.
a) Chng minh rng (a
n
, a
m
) = 1 vi mi m,= n.
b) Chng minh rng lim

n
1
1/a
k
= 1.
Hng dn:
a) a
m
1 = a
m1
. . . a
n
(a
n
1)
b) 1/a
k
= 1/(a
k
1) 1/(a
k+1
1)
Bi 1.17 (Ba Lan 2002). Cho trc s nguyn dng k. Dy s a
n
c xc
nh bi a
1
= k + 1, a
n+1
= a
2
n
ka
n
+ k vi mi n 1. Chng minh rng vi
mi m,= n ta c (a
m
, a
n
) = 1.
Bi 1.18 (KVANT). Cho 1 a
0
< a
1
< < a
n
l cc s nguyn dng.
Chng minh rng
1/[a
0
, a
1
] + 1/[a
1
, a
2
] + + 1/[a
n1
, a
n
] 1 1/2
n
1.6. Bi tp 35
Hng dn: Vi a < b, 1/[a, b] = (a, b)/ab (b a)/ab = 1/a 1/b.
Bi 1.19 (Ba Lan 1997). Dy s a
1
, a
2
, . . . xc nh bi
a
1
= 0, a
n
= a
[n/2]
+ (1)
n(n+1)/2
Vi mi s t nhin k, tm s cc ch s n sao cho 2
k
n < 2
k+1
v a
n
= 0.
Hng dn: Dng h m c s.
Bi 1.20 (Vit Nam, 1998). Cho dy s a
n
c xc nh bi a
0
= 20, a
1
=
100, a
n+2
= 4a
n+1
+5a
n
+20 vi n = 0, 1, 2, . . . Tm s nguyn dng h nh nht
tho mn iu kin a
n+h
a
n
chia ht cho 1998 vi mi n = 0, 1, 2, . . .
Bi 1.21 (Chn i tuyn VN, 1993). Gi (n) l hm Euler (ngha l (n)
l s cc c s nguyn dng khng ln hn b v nguyn t cng nhau vi n).
Tm tt c cc s nguyn dng k > 1 tho mn iu kin:
Vi a l s nguyn >1 bt k, t x
0
= a, x
n+1
= k(x
n
) vi n = 0, 1, . . . th
(x
n
) lun b chn.
Bi 1.22 (M 1997). Cho dy s t nhin a
1
, a
2
, . . . , a
1997
tho
a
i
+ a
j
a
i+j
a
i
+ a
j
+ 1
vi mi i, j nguyn dng tho i + j 1997. Chng minh rng tn ti s thc x
sao cho a
n
= [nx] vi mi n = 1, 2, ..., 1997.
Hng dn: Chng minh rng a
n
/n < (a
m
+ 1)/m vi mi m, n.
Bi 1.23. Cho dy s a
n

a) [Lin X 1977] Chng minh rng nu lim(a


n+1
a
n
/2) = 0 th lima
n
= 0.
b) Tm tt c cc gi tr a sao cho nu lim(a
n+1
a
n
) = 0 th lima
n
= 0.
Bi 1.24 (CRUX). Tm s hng tng qut ca dy s p
n
xc nh bi p
0
=
1, p
n+1
= 5p
n
(5p
4
n
5p
2
n
+ 1)
Bi 1.25. Dy s a
n
c xc nh bi a
1
> 0, a
2
> 0 v a
n+1
=

a
n
+

a
n1
.
Chng minh dy s a
n
hi t v tm gii hn.
Bi 1.26 (LMO 1989). Dy s thc a
k

k=1
tho mn iu kin a
k+1
= (ka
k
+
1)/(k a
k
). Chng minh rng dy s cha v hn s hng dng v v hn s
hng m.
Bi 1.27 (LMO 1989). Dy s thc a
k

k=1
tho mn iu kin [a
m
+ a
n

a
m+n
[ 1/(m+n) vi mi m, n. Chng minh rng a
k
l cp s cng.
1.6. Bi tp 36
Bi 1.28. Vi n 2, gi x
n
l nghim dng duy nht ca phng trnh x
n
=
x
n1
+x
n2
+ +x + 1.
a) Chng minh rng limx
n
= 2.
b) Hy tm lim(2 x
n
)
1/n
.
Bi 1.29 (Bulgari 82). Cho x
1
, . . . , x
n
l cc s thc thuc on [0, 2]. Chng
minh rng
n

i=1
n

j=1
[x
i
x
j
[ n
2
.
Du bng xy ra khi no?
Hng dn: Sp li th t!
Bi 1.30 (Bulgari 86). Cho dy s thc a
n

n=1
tho mn iu kin a
n+1

(1 + k/n)a
n
1, n = 1, 2, . . . trong 0 < k < 1. Chng minh rng tn ti s t
nhin t sao cho a
t
< 0.
Hng dn: a
n+1
/(n + 1) < a
n
/n 1/(n + 1).
Bi 1.31. Hai dy s a
n
, b
n
xc nh bi a
1
> 0, b
1
> 0, a
n+1
= a
n
+
1/b
n
, b
n+1
= b
n
+ 1/a
n
. Chng minh rng a
50
+b
50
> 20.
Hng dn: Xt c
n
= (a
n
+ b
n
)
2
.
Bi 1.32 (Canada 1985). Cho 1 < x
1
< 2. Vi n = 1, 2, . . . ta nh ngha
x
n+1
= 1 + x
n
x
2
n
/2. Chng minh rng vi mi n 3 ta c [x
n

2[ < 1/2
n
.
Bi 1.33 (PARABOLA). Cho a, b > 0. Hai dy s a
n
, b
n
xc nh bi
a
1
=

ab, b
1
= (a + b)/2, a
n+1
=

a
n
b
n
, b
n+1
= (a
n
+ b
n
)/2. Chng minh rng
vi mi n nguyn dng ta c [b
n
a
n
[ [b a[/2
n
.
Bi 1.34 (IMO 1978). Cho a
n
l dy cc s nguyn dng phn bit. Chng
minh rng vi mi n ta c
n

k=1
a
k
/k
2

k=1
1/k.
Bi 1.35 (Putnam 2001). Gi s a
n

n=1
l dy s tng cc s thc dng
sao cho lima
n
/n = 0. C th tn ti v s cc s nguyn dng n sao cho
a
ni
+ a
n+i
< 2a
n
vi mi i = 1, 2, . . . , n 1 hay khng?
Bi 1.36 (o - Ba Lan 2001). Cho a
1
, a
2
, . . . , a
2010
l dy s tho mn iu
kin
1.6. Bi tp 37
1. Tng 20 s hng lin tip ca dy s l khng m.
2. [a
i
a
i+1
[ 1 vi mi i = 1, 2, . . . , 2009.
Hy tm min

2001
i=1
a
i
.
Bi 1.37 (Ba Lan 2001). Cho dy s a
n
xc nh bi a
0
= 1, a
n
= a
[7n/9]
+
a
[n/9]
, n = 1, 2, . . . Chng minh rng tn ti k sao cho a
k
< k/2001!.
Bi 1.38 (Trung Quc 1997). Cho a
1
, a
2
, . . . l dy s thc tho mn iu
kin a
n+m
a
n
+a
m
vi mi m, n. Chng minh rng a
n
ma
1
+ (n/m1)a
m
vi mi n m.
Bi 1.39 (Singapore 1997). Cho dy s a
n
xc nh bi a
0
= 1/2, a
k+1
=
a
k
+a
2
k
/n, k = 1, 2, . . . , n 1. Chng minh rng 1 1/n < a
n
< 1.
Hng dn: Chng minh bng quy np rng (n + 1)/(2n k + 2) < a
k
<
n/(2n k).
Bi 1.40 (Baltic Way). Gi s a
1
, a
2
, . . . , a
9
l cc s khng m sao cho a
1
=
a
9
= 0 v t nht c mt s khc 0. Chng minh rng tn ti ch s i, 2 i 8
sao cho a
i1
+a
i+1
< 2a
i
. Khng nh c cn ng khng nu thay 2 bt ng
thc cui cng bng 1.9?
Bi 1.41. Dy s a
n
c xc nh bi cng thc truy hi
a
0
= 1, a
n+1
=
a
n
1 +na
n
, n = 0, 1, 2, . . .
Hy tm cng thc tng qut cho a
n
.
Bi 1.42 (Vit Nam, 1984). Dy s u
1
, u
2
, . . . c xc nh bi: u
1
= 1, u
2
=
2, u
n+1
= 3u
n
u
n1
vi n = 2, 3, . . . t v
n
=

1kn
arcotgu
k
.
Hy tm gii hn v
n
khi n dn n v cng.
Hng dn: Dng sai phn.
Bi 1.43 (PTNK, 1999). Cho a > 1 v dy s x
n
c xc nh nh sau
x
1
= a, x
n+1
= na
x
vi mi n 1.
Hy xc nh tt c cc gi tr ca a dy x
n
hi t.
1.6. Bi tp 38
Bi 1.44. Cho dy s dng a
n
. Bit rng tn ti gii hn
lim
n
n

k=1
1
a
k
= A <
t s
n
= a
1
+a
2
+ + a
n
. Chng minh rng tng
n

k=1
k
2
a
k
(s
k
)
2
cng c gii hn hu hn khi n .
Hng dn: Dng cng thc tnh tng tng phn
Bi 1.45. Cho f : N R tho iu kin f(a+b) f(a)+f(b) vi mi [ba[ k
(k l s nguyn dng c nh). Hi c tn ti gii hn f(n)/n khi n dn n v
cng khng?
Bi 1.46. Cc phn t ca dy s a
1
, a
2
, a
3
, . . ., l cc s nguyn dng khc
nhau. Chng minh rng vi mi k tn ti n sao cho tn ti a
n
n.
Bi 1.47. Chng minh rng nu a
1
> 2 v a
n
= a
2
n1
2 th
1
a
1
+
1
a
1
a
2
+
1
a1a2a3
+ =
1
2
[a
1

_
a
2
1
4].
Hng dn: Dng lng gic.
Bi 1.48. Dy s dng a
n
tho mn iu kin a
n
< a
n+1
+ a
2
n
. C th khng
nh tng

n
i=1
a
i
dn n v cng khi n dn n v cng hay khng?
Bi 1.49 (THTT). Cho s thc r > 2. Cho dy s thc dng a
n
tho mn
iu kin a
r
n
= a
1
+ + a
n1
vi mi n 2. Chng minh rng dy a
n
/n c
gii hn hu hn khi n v tm gii hn .
Bi 1.50 (Chn i tuyn Vit Nam, 1985). Dy s thc x
n
c xc
nh bi:
x
1
= 29/10, x
n+1
= (x
n
/
_
x
2
n
1) +

3, n = 1, 2, 3 . . .
Hy tm s thc nh hn x
2k1
v ln hn x
2k
vi mi k = 1, 2, . . .
Bi 1.51 (Chn i tuyn Vit Nam, 1996). Tm tt c cc gi tr ca a
dy s x
n
c xc nh bi
x
0
=

1996
x
n+1
= a/(1 +x
2
n
)
c gii hn hu hn khi n dn ti v cng.
1.6. Bi tp 39
Hng dn: Chuyn v dng x
n+1
= f(x
n
), x
0
= b.
Bi 1.52 (Vit Nam, 1997). Cho n l s nguyn >1, khng chia ht cho 1997.
t
a
i
= i + ni/1997 vi mi i = 1, 2, . . . , 1996,
b
j
= j + 1997j/n vi mi j = 1, 2, . . . , n 1.
Ta sp xp cc s a
i
v b
i
theo th t tng dn:
c
1
c
2
c
1995+n
Chng minh rng c
k+1
c
k
< 2 vi mi k = 1, 2, ..1994 + n.
Bi 1.53 (Vit Nam, 1998). Cho a l mt s thc khng nh hn 1. t
x
1
= a, x
n+1
= 1 + ln(x
2
n
/(1 + ln(x
n
)) vi n = 1, 2, . . .
Chng minh rng dy s x
n
c gii hn v tm gii hn .
Bi 1.54. Cho dy s x
n
xc nh bi, x
1
= a, x
n+1
= (2x
3
n
)/(3x
2
n
1) vi mi
n 1. Tm tt c cc gi tr ca a dy s xc nh v c gii hn hu hn.
Bi 1.55. Chng minh rng dy s xc nh bi iu kin x
n+1
= x
n
+ x
2
n
/n
2
vi n 1, trong 0 < x
1
< 1 l dy b chn.
Bi 1.56. Cho dy s
a
n
=

1 + 2
_
1 +
_
1 + (n 1)

1 + n
Chng minh rng lim
n
a
n
= 3.
Bi 1.57. Dy a
1
+ 2a
2
, a
2
+ 2a
3
, a
3
+ 2a
4
, . . . hi t. Chng minh rng dy
a
1
, a
2
, a
3
, . . . cng hi t.
Bi 1.58. Cho dy A(n), n = 1, 2, . . . tho mn: vi mi x thc th lim
n
A([x
n
]) =
0. Chng minh rng limA(n) = 0 khi n tin ti v cng.
Bi 1.59. Cho hm s
f(x) = x +Asinx + Bcos x vi A
2
+B
2
< 1.
Xt dy s
a
0
= a, a
1
= f(a
0
), . . . , a
n+1
= f(a
n
), . . .
Chng minh rng vi mi a, dy s a
n
c gii hn v hy tm gii hn .
1.6. Bi tp 40
Bi 1.60. Cho dy s a
n
, c xc nh nh sau: a
0
= a, a
1
= b, a
n+1
=
a
n
+ (a
n
a
n1
)/2n. Tm lim
n
a
n
.
Bi 1.61 (AMM). Cho H
n
l dy s Fibonacci tng qut, tc l H
1
, H
2
l
cc s nguyn bt k v vi n > 2 th H
n
= H
n1
+H
n2
.
a) Hy tm T, ph thuc vo H
1
v H
2
sao cho cc s H
2n
H
2n+2
+T, H
2n
H
2n+4
+
T, H
2n1
H
2n+1
T, H
2n1
H
2n+3
T u l cc s chnh phng.
b) Chng minh T l duy nht.
Bi 1.62. Cho r l s thc. Xc nh dy s x
n
bi x
0
= 0, x
1
= 1, x
n+2
=
rx
n+1
x
n
vi n 0. Chng minh rng x
1
+x
3
+ + x
2m1
= x
2
m
.
Bi 1.63 (IMO 1977). Trong mt dy s hu hn cc s thc, tng 7 s hng
lin tip ca dy lun m, cn tng 11 s hng lin tip lun dng. Hi dy s
c th c nhiu nht bao nhiu s hng.
Ti liu tham kho
1. Jean-Marie Monier, Gii tch 1, 2, 3, 4, NXBGD 1999-2000.
2. L Hi Chu: Tuyn tp cc thi ton quc t.
3. Titu Andreescu, Razvan Gelca: Mathematical Olympiad Challenges, Birkhauser
2000.
4. A. Gardiner, The Mathematical Olympiad Hanbook, Oxford, 1997.
5. Titu Andreescu, Zuming Feng: Mathematical Olympiads 1998-1999, 1999-
2000, 2000-2001, MAA, 2000-2002.
6. Arthur Engel: Problem-Solving Strategies, Springer 1997. 7
7. G.Polya, G.Szego: Cc bi tp v nh l ca gii tch, Nauka 1977 (Ting
Nga).
8. Cupsov, Nesterenko . . .: Thi v ch ton ton Lin X, Prosvesenie, 1999
(Ting Nga).
9. 400 bi ton t American Mathematical monthly, Mir, 1977 (Ting Nga).
10. thi ton ca Vit Nam, cc nc v khu vc.
11. Tp ch Ton hc v Tui tr (THTT), Parabola, Kvant, American Math-
ematical monthly (AMM).
Trn Nam Dng - HKHTN TP H Ch Minh
227 Nguyn Vn C, Qun 5, TP H Ch Minh
Email: namdung@fpt.com.vn, namdung@fsoft.com.vn
Chng 2
Phng trnh sai phn
2.1 Sai phn
2.1.1 nh ngha
Cho hm s y = f(x) xc nh trn R, t x
k
= x
0
+ kh (k N

) vi x
0

R, h R, bt k, cho trc. Gi y
k
= f(x
k
) l gi tr ca hm s f(x) ti x = x
k
.
Khi , Hiu s y
k
:= y
k+1
y
k
(k N

) c gi l sai phn cp 1 ca hm s
f(x). Hiu s
2
y
k
:= y
k+1
y
k
= (y
k
) (k N

) c gi l sai phn cp
2 ca hm s f(x). Tng qut,
i
y
k
:=
i1
y
k+1

i1
y
k
= (
i1
y
k
) (k
N

) c gi l sai phn cp i ca hm s f(x) (i = 1; 2; ; n; ).


2.1.2 Tnh cht
Mnh 2.1 (Biu din sai phn theo gi tr ca hm s). Sai phn mi cp
u c th biu din theo cc gi tr ca hm s:
y
0
; y
1
; y
2
; ; y
n
;
Chng minh. Tht vy, ta c
y
k
= y
k+1
y
k

2
y
k
= y
k+1
y
k
= y
k+2
y
k+1
(y
k+1
y
k
)
= y
k+2
2y
k+1
+ y
k
.
Tng t, bng quy np ta c th chng minh c.

i
y
k
=
i

s=1
(1)
s
C
s
i
y
k+is
, (pcm).
41
2.1. Sai phn 42
Mnh 2.2 (Sai phn ca hng s). Sai phn ca hng s bng 0.
Chng minh. Tht vy, vi y = f(x) = C = const ta c: f(x) = C C = 0.
Hn th na, sai phn mi cp ca hng s u bng 0.
Mnh 2.3 (Tnh cht tuyn tnh ca sai phn ). Sai phn mi cp l mt
ton t tuyn tnh trn tp cc hm s. Tc l.
i N

, ; R, f(x); g(x) : R R, ta lun c:

i
(f(x) +g(x)) =
i
f(x) +
i
g(x).
Chng minh. Tht vy, t f
k
= f(x
k
) ; g
k
= g(x
k
), ta thu c

i
(f
k
+ g
k
) =
i

s=0
(1)
s
C
s
i
[f
k+is
+ g
k+is
]
=
i

s=0
(1)
s
C
s
i
f
k+is
+
i

s=0
(1)
s
C
s
i
g
k+is
=
i
f
k
+
i
g
k
.
Vy nn

i
(f(x) +g(x)) =
i
f(x) +
i
g(x) vi mi i N

(pcm).
Mnh 2.4 (Sai phn ca a thc). Sai phn cp i ca mt a thc bc n.
+) L mt a thc bc n i khi i < n. +) L hng s khi i = n. +) Bng 0 khi
i > n.
Chng minh. Do sai phn mi cp l ton t tuyn tnh nn ta ch cn chng
minh tnh cht cho a thc y = P
n
(x) = x
n
.
+) Khi i < n ta c
1
o
) Vi i = 1 th: x
n
= (x + h)
n
x
n
= P
n1
(x) l a thc bc n 1 i
vi x. Vy khng nh ng vi i = 1.
2
o
) Gi s khng nh ng vi i = k < n, tc l
k
x
n
= P
nk
(x) l a thc
bc n k i vi x. Khi

k+1
x
n
= (
k
x
n
) =
k
((x + h)
n
)
k
(x
n
)
= P
nk
(x +h) P
nk
(x) = P
nk1
(x)
l a thc bc n k 1 = n (k + 1) i vi x.
Vy khng nh cng ng vi i = k + 1. T , theo nguyn l quy np ton
hc suy ra khng nh ng vi mi i N

(pcm).
2.2. Phng trnh sai phn tuyn tnh 43
+) Khi i = n th theo trn,
n
(x
n
) l a thc cp n n = 0 i vi x nn l
hng s.
+) Khi i > n th

i
(x
n
) =
in
(
n
(x
n
)) =
in
C (C = const) = 0.
Vy tnh cht c chng minh trn vn.
Mnh 2.5 (Cng thc sai phn tng phn).
(f
k
g
k
) = f
k
g
k
+ g
k+1
f
k
.
Chng minh. Ta c
(f
k
g
k
) = f
k+1
g
k+1
f
k
g
k
= f
k+1
g
k+1
f
k
g
k+1
+ f
k
g
k+1
f
k
g
k
= g
k+1
(f
k+1
f
k
) +f
k
(g
k+1
g
k
)
= f
k
g
k
+g
k+1
f
k
.
Mnh 2.6 (Tng cc sai phn).
n

k=1
y
k
= y
n+1
y
1
.
Chng minh.
n

k=1
y
k
= y
1
+ y
2
+ + y
n1
+ y
n
= y
2
y
1
+y
3
y
2
+ + y
n
y
n1
+y
n+1
y
n
= y
n+1
y
1
.
2.2 Phng trnh sai phn tuyn tnh
2.2.1 Mt s khi nim chung v phng trnh sai phn
nh ngha 2.1. Phng trnh sai phn (cp k) l mt h thc tuyn tnh cha
sai phn cc cp ti k.
f(y
n
; y
n
;
2
y
n
; ;
k
y
n
) = 0. (1)
2.3. Phng trnh sai phn tuyn tnh bc nht 44
V sai phn cc cp u c th biu din theo gi tr ca hm s nn (1) c
dng:
a
0
y
n+k
+ a
1
y
n+k1
+ +a
k
y
n
= f(n). (2)
trong a
0
; a
1
; ; a
k
, f(n) bit, cn y
n
, y
n+1
, , y
n+k
l cc gi tr cha
bit.
Phng trnh (2) c gi l phng trnh sai phn tuyn tnh cp k.
Nu f(n) = 0 th phng trnh (2) c dng
a
0
y
n+k
+a
1
y
n+k1
+ + a
k
y
n
= 0. (3)
v c gi l phng trnh sai phn tuyn tnh thun nht cp k.
Nu f(n) ,= 0 th (2) c gi l phng trnh sai phn tuyn tnh khng
thun nht.
b. Nghim.
Hm s y
n
bin n tho mn (2) c gi l nghim ca phng trnh sai
phn tuyn tnh (2).
Hm s y
n
ph thuc k tham s tho mn (3) c gi l nghim tng qut
ca (3).
Mt nghim y

n
tho mn (2) c gi l mt nghim ring ca (2).
2.3 Phng trnh sai phn tuyn tnh bc nht
2.3.1 nh ngha
Phng trnh sai phn tuyn tnh bc nht (cp mt) l phng trnh sai
phn dng:
u
1
= , au
n+1
+bu
n
= f(n) n N

(1)
trong ; a ,= 0 ; b ,= 0 l cc hng s v f(n) l biu thc ca n cho trc.
2.3.2 Phng php gii
A. Gii phng trnh sai phn thun nht tng ng.
1
+
) Gii phng trnh c trng: a + b = 0 tm .
2
+
) Tm nghim ca phng trnh sai phn tuyn tnh thun nht tng
ng: au
n+1
+ bu
n
= 0 di dng u
n
= c
n
(c l hng s ).
B. Tm mt nghim ring u

n
ca phng trnh khng thun nht.
C. Tm nghim tng qut ca phng trnh (1):
u
n
= u

n
+ u
n
.
2.3. Phng trnh sai phn tuyn tnh bc nht 45
V d 2.1. Phng trnh u
n+1
= 3u
n
+1 c u
n
= C.3
n
; u

n
=
1
2
nn c nghim
tng qut l u
n
= C.3
n

1
2
vi C l hng s bt k.
Sau y ta trnh by phng php tm nghim ring.
2.3.3 Phng php tm nghim ring ca phng trnh sai phn
tuyn tnh cp 1 khng thun nht khi v phi f(n) c dng
c bit
Trng hp 1. Nu f(n) = P
m
(n) l a thc bc m i vi n. Khi : +)
Nu ,= 1 th ta chn u

n
= Q
m
(n) cng l a thc bc m i vi n. +) Nu
= 1 th ta chn u

n
= nQ
m
(n) trong Q
m
(n) cng l a thc bc m i vi
n.
V d 2.2. Gii phng trnh sai phn:
_
x
0
= 7
x
n+1
= 15x
n
14n + 1
Gii. Ta c f(n) = 14n+1 l a thc bc nht, = 15 ,= 1 chn x

n
= an+b.
Thay vo phng trnh ta c
a(n + 1) + b = 15(an + b) 14n + 1.
Suy ra a = 1 ; b = 0. Vy x

n
= n cn x
n
= C.15
n
v nghim tng qut
l: x
n
= C.15
n
+ n. M x
0
= 7 nn C = 7. Vy phng trnh c nghim:
x
n
= 7.15
n
+n.
V d 2.3. Gii phng trnh sai phn:
_
x
0
= 99
x
n+1
= x
n
2n 1
Gii. f(n) = 2n 1 l a thc bc nht, = 1 chn x

n
= n(an +b). Thay
vo (1.2) c:
(n + 1)[a(n + 1) +b] = n(an + b) 2n 1 a = 1 ; b = 0 x

n
= n
2
.
Cn x
n
= C.1
n
= C x
n
= C n
2
, m x
0
= 99 C 0
2
= 99 C = 99. Vy
phng trnh (1.2) c nghim: x
n
= 99 n
2
.
Trng hp 2. f(n) = p.
n
(p; ,= 0). Khi :
+) Nu ,= th ta chn x

n
= d.
n
(d R).
+) Nu = th ta chn x

n
= d.n.
n
(d R).
2.3. Phng trnh sai phn tuyn tnh bc nht 46
V d 2.4. Gii phng trnh sai phn:
_
x
0
= 8
x
n+1
= 2x
n
+ 3
n
(1.3)
.
Gii. Do = 2 ,= 3 = nn ta chn x

n
= d.3
n
. Thay vo phng trnh (1.3)
c d = 1 x

n
= 3
n
. Cn x
n
= C.2
n
. Vy x
n
= C.2
n
+ 3
n
. Thay vo iu kin
bin c C = 7.
Tr li: phng trnh cho c nghim x
n
= 7.2
n
+ 3
n
.
V d 2.5. Gii phng trnh sai phn:
_
x
0
= 101
x
n+1
= 7.x
n
+ 7
n+1
(1.4)
.
Gii. Do = 7 = nn ta chn x

n
= d.n.3
n
. Thay vo phng trnh (1.4) c
d = 1 x

n
= n.7
n
. Cn x
n
= C.7
n
. Vy x
n
= C.7
n
+ n.7
n
. Thay vo iu kin
bin c C = 101.
Tr li: phng trnh cho c nghim x
n
= (101 + n).7
n
.
Trng hp 3. f(n) = . sinnx +. cos nx (+ ,= 0; x ,= k; k Z). Khi
, ta chn u

n
= A. sinnx + B. cos nx vi A; B R l cc hng s.
V d 2.6. Gii phng trnh sai phn:
_
x
0
= 1

2.x
n+1
= x
n
sin
n
4
(1.5)
.
Gii. C =
1

2
; f(n) = sin
n
4
nn ta chn x

n
= A. cos
n
4
+B. sin
n
4
.
Thay x

n
vo (1.5), bin i v so snh cc h s ta c A = 1 ; B = 0 x

n
=
cos
n
4
. Cn x
n
= C.(
1

2
)
n
x
n
= C.(
1

2
)
n
+ cos
n
4
. Thay vo iu kin bin
x
0
= 1 ta c C = 0. Vy phng trnh cho c nghim x
n
= cos
n
4
.
Trng hp 4.
f(n) =
m

k=1
f
k
(n).
Khi ta chn nghim ring x

n
ca (1) di dng: x

n
=

m
k=1
x

nk
trong x

nk
tng ng l nghim ring ca phng trnh sai phn (1) vi V P = f
k
(n).
V d 2.7. Gii phng trnh sai phn:
_
x
0
= 17
x
n+1
= 2x
n
n
2
+ 2n + 1 + 6.2
n
(1.6)
.
Gii. C = 2 ; f
1
(n) = n
2
+ 2n + 1 ; f
2
(n) = 6.2
n

x
n
= C.2
n
; x

n1
= an
2
+bn + c ; x

n2
= d.n.2
n
.
2.4. Phng trnh sai phn tuyn tnh cp 2 47
Vy ta chn x

n
= an
2
+bn+c+d.n.2
n
. Thay vo (1.6) v so snh cc h s c:
a = 1 ; b = c = 0 ; d = 3. Vy: x
n
= C.2
n
+n
2
+ 3n.2
n
. Thay vo iu kin bin
x
0
= 17 ta c C = 17 v do nghim ca phng trnh sai phn cho l:
x
n
= 17.2
n
+ n
2
+ 3n.2
n
.
2.3.4 Bi tp
Gii cc phng trnh sai phn tuyn tnh sau:
1. u
n+1
= 3u
n
6n + 1 ; u
1
= 1.
p s: u
n
= 3n + 1 3
n
.
2. u
n+1
= u
n
+ 2n
2
; u
1
= 1.
p s: u
n
=
1
3
(2n
3
3n
2
+ n + 3).
3. u
n+1
= 5u
n
3
n
; u
0
= 1.
p s: u
n
=
1
2
(5
n
+ 3
n
).
4. u
n+1
= 2u
n
+ 6.2
n
; u
0
= 1.
p s: u
n
= (3n + 1).2
n
.
5. u
n+1
= u
n
+ 2n.3
n
; u
0
= 0.
p s: u
n
=
1
2
[(2n 3).3
n
+ 3].
6. u
n+1
2u
n
= (n
2
+ 1).2
n
; u
0
= 1.
p s: u
n
=
_
n(2n
2
3n + 7)
6
+ 2
_
.2
n
.
7. u
n+1
2u
n
= n + 3
n
; u
0
= 1.
p s: u
n
= 2
n
+ 3
n
n 1.
2.4 Phng trnh sai phn tuyn tnh cp 2
2.4.1 nh ngha
Phng trnh sai phn tuyn tnh cp hai l phng trnh sai phn dng:
u
1
= , u
2
= , au
n+2
+ bu
n+1
+cu
n
= f(n), n N

(1)
trong a, b, c, , l cc hng s, a ,= 0, c ,= 0 v f(n) l biu thc cha n
cho trc.
2.4. Phng trnh sai phn tuyn tnh cp 2 48
2.4.2 Cch gii
+) Gii phng trnh thun nht tng ng. +) Tm nghim ring ca phng
trnh khng thun nht. +) Tm nghim tng qut ca phng trnh (1) di
dng:
u
n
= u
n
+u

n
.
A- Gii phng trnh thun nht tng ng
au
n+2
+bu
n+1
+ cu
n
= 0 (2)
1
+
) Gii phng trnh c trng:
a.
2
+ b. +c = 0. (3) tm .
2
+
) Tm nghim tng qut ca phng trnh thun nht tng ng.
Trng hp 1: Nu (3) c hai nghim phn bit: =
1
; =
2
th:
u
n
= A.
n
1
+ B.
n
2
,
trong A v B c xc nh khi bit u
1
v u
2
.
Trng hp 2: Nu (3) c nghim kp:
1
=
2
= th:
u
n
= (A+ Bn).
n
,
trong A v B c xc nh khi bit u
1
v u
2
.
Trng hp 3: Nu l nghim phc, = x +i.y th ta t
r = [[ =
_
x
2
+y
2
, tan =
y
x
,
_


2
;

2
_
.
lc = r(cos + i. sin) v
u
n
= r
n
(A. cos n +B. sinn),
trong A v B c xc nh khi bit u
1
v u
2
. V d
V d 2.8. Gii phng trnh sai phn:
_
x
0
= 2 ; x
1
= 8
x
n+2
= 8x
n+1
+ 9x
n
.
2.4. Phng trnh sai phn tuyn tnh cp 2 49
Gii. Phng trnh c trng:

2
+ 8 9 = 0 = 1 hoc = 9. (Hai nghim phn bit.)
Vy: x
n
= x
n
= A.1
n
+ B.(9)
n
= A +B.(9)
n
.
Gii iu kin bin:
_
x
0
= 2
x
1
= 8

_
A+B = 2
A9B = 8

_
A = 1
B = 1
.
Vy phng trnh cho c nghim: x
n
= 1 + (9)
n
.
V d 2.9. Gii phng trnh sai phn:
_
x
0
= 1 ; x
1
= 16
x
n+2
= 8x
n+1
16x
n
.
Gii. Phng trnh c trng:

2
8 + 16 = 0
1
=
2
= 4 (c nghim kp).
Vy: x
n
= x
n
= (A+Bn).4
n
.
Gii iu kin bin:
_
x
0
= 1
x
1
= 16

_
A = 1
(A+B).4 = 16

_
A = 1
B = 3
.
Vy phng trnh cho c nghim: x
n
= (1 + 3n).4
n
.
V d 2.10. Gii phng trnh sai phn:
_
_
_
x
0
= 1 ; x
1
=
1
2
x
n+2
= x
n+1
x n
.
Gii. Phng trnh c trng:

2
+ 1 = 0 =
1 + i

3
2
hoc =
1 i

3
2
. (Hai nghim phc.)
C:
=
1 +i

3
2
= cos

3
+ i. sin

3
r = 1 ; =

3
.
Nghim tng qut: x
n
= x
n
= A. cos
n
3
+B. sin
n
3
.
Gii iu kin bin:
_
_
_
x
0
= 1
x
1
=
1
2

_
_
_
A = 1
A. cos

3
+B. sin

3
=
1
2

_
A = 1
B = 0
.
2.4. Phng trnh sai phn tuyn tnh cp 2 50
Vy phng trnh cho c nghim: x
n
= cos
n
3
. Bi tp Gii cc phng trnh
sai phn sau:
1. y
n+2
= 4y
n+1
3y
n
vi y
0
= 1 ; y
1
= 1.
2. y
n+2
= 8y
n+1
16y
n
vi y
0
= 1 ; y
1
= 1.
3. y
n+2
= 2y
n+1
2y
n
vi y
0
=
1
2
; y
1
= 2.
B- Cc phng php tm nghim ring ca phng trnh sai phn tuyn tnh cp
hai khng thun nht
au
n+2
+bu
n+1
+ cu
n
= f(n)
vi v phi c dng c bit Trng hp 1.
f(n) = P
k
(n) l a thc bc k i vi n.
Khi : +) Nu phng trnh c trng (3) khng c nghim = 1 th ta chn
x

n
= Q
k
(n)
trong Q
k
(n) l a thc bc k no i vi n. +) Nu phng trnh c trng
(3) c nghim n = 1 th ta chn
x

n
= nQ
k
(n)
trong Q
k
(n) l a thc bc k no i vi n. +) Nu phng trnh c trng
(3) c nghim kp = 1 th ta chn
x

n
= n
2
Q
k
(n)
trong Q
k
(n) l a thc bc k no i vi n.
V d 2.11. Tm mt nghim ring x

n
ca phng trnh sai phn:
x
n+2
= 4x
n+1
+ 5x
n
+ 12n + 8.
Gii. Phng trnh c trng:

2
+ 4 5 = 0 = 1 hoc = 5 ; f(n) = 12n + 8.
Chn x

n
= n(an+b). Thay vo phng trnh cho v so snh cc h s ta c
a = 1 ; b = 0.
Vy phng trnh cho c mt nghim ring l x

n
= n
2
.
2.4. Phng trnh sai phn tuyn tnh cp 2 51
V d 2.12. Gii phng trnh sai phn:
2x
n+2
5x
n+1
+ 2x
n
= n
2
2n + 3 vi x
0
= 1 ; x
1
= 3.
Gii. Phng trnh c trng:
2
2
5 + 2 = 0 = 2 hoc =
1
2
; f(n) = n
2
2n + 3.
Vy phng trnh thun nht c nghim tng qut x
n
= A.2
n
+ B.(
1
2
)
n
.
Chn x

n
= an
2
+ bn + c. Thay vo phng trnh cho v so snh cc h s ta
c a = 1 ; b = 4 ; c = 10.
Vy phng trnh cho c mt nghim ring l x

n
= n
2
+ 4n 10.
Do phng trnh cho c nghim tng qut l:
x
n
= A.2
n
+ B.(
1
2
)
n
n
2
+ 4n 10.
Thay vo cc iu kin bin ta tm c A = 3 ; B = 8. Vy nghim ca phng
trnh cho l:
x
n
= 3.2
n
+ 8.(
1
2
)
n
n
2
+ 4n 10.
Trng hp 2.
f(n) = P
k
(n).
n
trong P
k
(n) l mt a thc bc k i vi n.
Khi : +) Nu khng phi l nghim ca phng trnh c trng (3) th ta
chn:
x

n
= Q
k
(n)
trong Q
k
(n) l mt a thc bc k no i vi n vi h s cn c xc
nh. +) Nu l mt nghim n ca phng trnh c trng (3) th ta chn:
x

n
= n.Q
k
(n)
trong Q
k
(n) l mt a thc bc k no i vi n . +) Nu l nghim kp
ca phng trnh c trng (3) th ta chn:
x

n
= n
2
.Q
k
(n),
trong Q
k
(n) l mt a thc bc k no i vi n .
V d 2.13. Tm mt nghim ring ca phng trnh sai phn sau:
2x
n+2
+ 5x
n+1
+ 2x
n
= (35n + 51).3
n
.
2.4. Phng trnh sai phn tuyn tnh cp 2 52
Gii. Ta c = 3 ; P
k
(n) = 35n + 51 l a thc bc nht.
Phng trnh c trng:
2
2
+ 5 + 2 = 0 = 2 :=
1
hoc =
1
2
:=
2
(
1
;
2
,= ).
Chn x

n
= (an + b).3
n
. Thay vo phng trnh cho v so snh cc h s ta
c: a = 1 ; b = 0.
Vy phng trnh cho c mt nghim ring l: x

n
= n.3
n
.
V d 2.14. Tm mt nghim ring ca phng trnh sai phn sau:
x
n+2
5x
n+1
+ 6x
n
= (8n + 11).2
n
.
Gii. Ta c = 2 ; P
k
(n) = 8n + 11 l a thc bc nht.
Phng trnh c trng:

2
5 + 6 = 0 = 2 :=
1
hoc = 3 :=
2
(
1
= ;
2
,= ).
Chn x

n
= n(an +b).2
n
. Thay vo phng trnh cho v so snh cc h s ta
c: a = 4 ; b = 23.
Vy phng trnh cho c mt nghim ring l: x

n
= (4n
2
+ 23n).2
n
.
V d 2.15. Tm mt nghim ring ca phng trnh sai phn sau:
x
n+2
10x
n+1
+ 25x
n
= (n + 2).5
n+1
Gii. Ta c = 5 ; P
k
(n) = 5n + 10 l a thc bc nht.
Phng trnh c trng:

2
10 + 25 = 0 = 5 :=
0
(nghim kp) (
1
=
2
= )
Chn x

n
= n
2
(an +b).5
n
. Thay vo phng trnh cho v so snh cc h s ta
c: a =
4
50
; b =
7
50
.
Vy phng trnh cho c mt nghim ring l: x

n
=
n
2
50
(4n + 7).5
n
.
Trng hp 3.
f(n) = P
m
(n). cos n +P
l
(n). sinn
trong P
m
(n) ; P
l
(n) ln lt l cc a thc bc m ; l i vi n.
K hiu k = Maxm; l v gi = cos +i sin (i
2
= 1). Khi :
+) Nu khng l nghim ca phng trnh c trng (3) th ta chn:
x

n
= T
k
(n). cos n +R
k
(n). sinn
2.4. Phng trnh sai phn tuyn tnh cp 2 53
trong , T
k
(n) ; R
k
(n) l cc a thc bc k i vi n no .
+) Nu l nghim ca phng trnh c trng (3) th ta chn:
x

n
= nT
k
(n). cos n + R
k
(n). sinn
trong , T
k
(n) ; R
k
(n) l cc a thc bc k i vi n no .
V d 2.16. Gii phng trnh sai phn:
_
x
0
= 1 ; x
1
= 0
x
n+2
= 2x
n+1
x
n
+ sin
n
2
Gii. Ta c P
m
(n) 0 ; P
l
(n) 1 ; =

2
= i.
Phng trnh c trng:
2
2 + 1 = 0 c nghim kp = 1 (,= i = ).
Nghim tng qut ca phng trnh thun nht : x
n
= an + b.
Nghim ring ca phng trnh cho c dng: x

n
= c. cos
n
2
+d. sin
n
2
.
Thay x

n
vo phng trnh cho, rt gn v so snh cc h s ta c:
c =
1
2
; d = 0 x

n
=
1
2
cos
n
2
Phng trnh cho c nghim tng qut l:
x
n
= an + b +
1
2
cos
n
2
Gii cc iu kin bin:
_
_
_
x
0
= b +
1
2
= 1
x
1
= a + b = 0

_
a =
1
2
b =
1
2
Vy phng trnh cho c nghim:
x
n
=
1
2
(1 n + cos
n
2
)
V d 2.17. Tm mt nghim ring ca phng trnh sai phn sau:
x
n+2
3x
n+1
+ 2x
n
= (n 2). cos
n
2
+ (3n + 1). sin
n
2
.
Gii. Ta c P
m
(n) = n 2 ; P
l
(n) = 3n + 1 ; =

2
= i.
Phng trnh c trng:

2
3 + 2 = 0 = 1 :=
1
; = 2 :=
2
(
1
;
2
,= ).
2.4. Phng trnh sai phn tuyn tnh cp 2 54
Nghim ring ca phng trnh cho c dng:
x

n
= (an + b). cos
n
2
+ (cn + d). sin
n
2
.
Thay x

n
vo phng trnh cho, rt gn v so snh cc h s ta c:
a = 1 ; b = c = d = 0 x

n
= n. cos
n
2
.
Vy phng trnh cho c mt nghim ring l: x

n
= n. cos
n
2
.
Trng hp 4:
f(n) =
m

k=1
f
k
(n).
Khi ta tm nghim ring ca (1) di dng:
x

n
=
m

k=1
x

nk
trong x

nk
l nghim ring ca phng trnh: (1) vi V P = f
k
(n) v c tm
theo mt trong cc trng hp trn.
V d 2.18. Tm mt nghim ring ca phng trnh sai phn sau:
x
n+2
= 5x
n+1
6x
n
+ 2
n
+n 1.
Gii. Phng trnh c trng:

2
5 + 6 = 0 = 2 :=
1
hoc = 3 := 2.
f(n) = 2
n
+n 1 := f
1
(n) +f
2
(n) vi f
1
(n) = 2
n
; f
2
(n) = n 1.
i vi phng trnh: x
n+2
= 5x
n+1
6x
n
+2
n
(1.8) do
1
= 2 = nn ta chn
nghim ring: x

n1
= an.2
n
. Thay vo (1.8) ri chia c hai v cho 2
n
ta c:
2a = 1 a =
1
2
x

n1
=
1
2
.n.2
n
= n.2
n1
.
i vi phng trnh: x
n+2
= 5x
n+1
6x
n
+n 1 (1.9) ta chn nghim ring:
x

n1
= cn + d. Thay vo (1.9) ri so snh cc h s hai v ta c:
c =
1
2
; d =
1
4
x

n2
=
1
2
.n +
1
4
.
Vy phng trnh cho c mt nghim ring l:
x

n
= x

n1
+x

n2
=
1
2
.n.2
n
+
1
2
.n +
1
4
=
1
4
(1 + 2n n.2
n+1
).
Bi tp Tm cc nghim ring ca cc phng trnh sai phn sau:
2.5. Phng trnh sai phn tuyn tnh cp 3 55
1. y
n+2
5y
n+1
+ 6y
n
= 0
2. 8y
n+2
6y
n+1
+ y
n
= 2
n
3. y
n+2
3y
n+1
+ 2y
n
= 5
n
+ 2n
3
+ 3n + 1
4. y
n+2
y
n+1
+ 2y
n
= n
2
5. y
n+2
+ y
n
= sin
n
2
6. 4y
n+2
+ 4y
n+1
+ y
n
= 2
7. y
n+2
2y
n+1
+y
n
= 5 + 3n
8. y
n+2
2y
n+1
+y
n
= 2
n
(n 1)
9. y
n+2
3y
n+1
+ 2y
n
= 3
n
10. 8y
n+2
6y
n+1
+ y
n
= 5 sin
n
2
2.5 Phng trnh sai phn tuyn tnh cp 3
2.5.1 nh ngha
Cho a, b, c, d, , , l cc hng s R ; a ,= 0 ; d ,= 0 cn f(n) l mt hm
s bin s n. Phng trnh:
_
u
1
= ; u
2
= ; u
3
=
au
n+3
+bu
n+2
+ cu
n+1
+ du
n
= f(n) quad(1)
c gi l phng trnh sai phn tuyn tnh cp (bc) ba.
2.5.2 Phng php gii
Phng trnh sai phn tuyn tnh cp ba lun gii c. Nghim tng qut
ca n c dng:
u
n
= u
n
+u

n
trong , u
n
l nghim tng qut ca phng trnh sai phn tuyn tnh thun
nht, cn u

n
l mt nghim ring no ca phng trnh cho. Cch tm u
n
Xt phng trnh c trng:
a
3
+b
2
+c +d = 0 (2)
2.5. Phng trnh sai phn tuyn tnh cp 3 56
+) Nu (3) c ba nghim thc phn bit:
1
,=
2
,=
3
,=
1
th:
u
n
= C
1

n
1
+ C
2

n
2
+C
3

n
3
+) Nu (3) c mt nghim thc bi 2 v mt nghim n:
1
=
2
,=
3
th:
u
n
= (C
1
+ C
2
n)
n
1
+ C
3

n
3
+) Nu (3) c mt nghim thc bi 3:
1
=
2
=
3
:=
0
th:
u
n
= (C
1
+C
2
n +C
3
n
2
)
n
0
+) Nu (3) c mt nghim thc
1
v hai nghim phc lin hp:

2;3
= r(cos i sin) th
u
n
= C
1

n
1
+ r
n
(C
2
cos n +C
3
sin n)
Trn y ta k hiu C
1
; C
2
; C
3
l cc hng s m s c xc nh bng cch
thay u
n
vo cc iu kin bin v gii h phng trnh thu c. Cch tm u

n
Trng hp 1. Nu f(n) = P
m
(n) l a thc bc m i vi n th: +) Khi
(3) khng c nghim = 1 th ta chn: u

n
= Q
m
(n) trong , Q
m
(n) l a thc
bc m i vi n. +) Khi (3) c nghim n = 1 th ta chn: u

n
= nQ
m
(n)
trong , Q
m
(n) l a thc bc m i vi n. +) Khi (3) c nghim bi hai = 1
th ta chn: u

n
= n
2
Q
m
(n) trong , Q
m
(n) l a thc bc m i vi n. +) Khi
(3) c nghim bi ba = 1 th ta chn: u

n
= n
3
Q
m
(n) trong , Q
m
(n) l a
thc bc m i vi n.
Trng hp 2. Nu f(n) = A.
n
( A ; l cc hng s cho trc) th: +)
Khi khng l nghim ca (3) th ta chn: u

n
= B.
n
vi B l hng s c xc
nh bng cch thay u

n
vo phng trnh cho. +) Khi l nghim n ca
(3) th ta chn: u

n
= B.n.
n
. +) Khi l nghim bi hai ca (3) th ta chn:
u

n
= B.n
2
.
n
. +) Khi l nghim bi ba (3) th ta chn: u

n
= B.n
3
.
n
.
2.5.3 V d
V d 2.19. Gii phng trnh sai phn:
_
x
1
= 0 ; x
2
= 1 ; x
3
= 3
x
n
= 7x
n1
11x
n2
+ 5x
n3
.
Gii. Phng trnh c trng:

3
7
2
+ 11 5 = 0 : c ba nghim:
1
=
2
= 1 ;
3
= 5.
Vy phng trnh c nghim tng qut
x
n
= (C
1
+ C
2
n).1
n
+C
3
.5
n
= C
1
+ C
2
n + C
3
.5
n
.
2.5. Phng trnh sai phn tuyn tnh cp 3 57
Thay vo iu kin bin ta c h phng trnh:
_

_
C
1
+ C
2
+ 5C
3
= 0
C
1
+ 2C
2
+ 25C
3
= 1
C
1
+ 3C
2
+ 125C
3
= 3

_
C
1
=
13
16
C
2
=
3
4
C
3
=
1
80
.
Vy phng trnh cho c nghim:
x
n
=
1
16
(5
n1
+ 12n 13).
V d 2.20. Gii phng trnh sai phn:
_
x
0
= 1 ; x
1
= 2 ; x
2
= 3
x
n+3
3x
n+2
+ 3x
n+1
x
n
= 1
Gii. Phng trnh c trng:

3
3
2
+ 3 1 = 0 : c nghim bi ba:
1
=
2
=
3
= 1.
Vy phng trnh thun nht c nghim tng qut
x
n
= (C
1
+C
2
n +C
3
n
2
).1
n
= C
1
+ C
2
n + C
3
n
2
.
Do f(n) = 1 = 1.1
n
nn ta chn nghim ring x

n
= B.n
3
.1
n
= B.n
3
. Thay vo
phng trnh cho ri so snh cc h s ta c
B =
1
6
x

n
=
1
6
n
3
.
Vy nghim tng qut ca phng trnh cho l:
x
n
= x

n
+ x
n
= C
1
+C
2
n +C
3
n
2
+
1
6
n
3
.
Thay vo cc iu kin bin v gii h phng trnh thu c ta c:
C
1
= 1 ; C
2
=
4
3
; C
3
=
1
2
. Vy phng trnh cho c nghim l
x
n
= 1 +
4
3
n
1
2
n
2
+
1
6
n
3
.
2.5. Phng trnh sai phn tuyn tnh cp 3 58
2.5.4 Phng trnh sai phn tuyn tnh cp k
nh ngha Phng trnh
a
0
y
n+k
+ a
1
y
n+k1
+ +a
k
y
n
= f(n) quad(1)
c gi l phng trnh sai phn tuyn tnh cp k. Cch gii A. Gii phng trnh sai phn tuyn
1
o
) Gii phng trnh c trng
a
0

k
+a
1

k1
+ +a
k1
+ a
k
= 0 (2) tm k.
2
o
) Tm nghim tng qut ca phng trnh thun nht tng ng.
Nu (2) c k nghim thc khc nhau l
1
,
2
, ,
k
th nghim tng
qut l
y
n
= c
1

n
1
+ c
2

n
2
+ +c
k

n
k
(3).
trong c
1
, c
1
, , c
k
l cc hng s tu .
Nu (2) c nghim thc
j
bi s th nghim tng qut l:
y
n
=
_
s1

i=1
c
j+i
n
i
_

n
j
+
k

i=1;i=j
c
i

n
i
.
Nu phng trnh c trng (2) c nghim phc n
j
= r(cos +i. sin)
th
j
= r(cos i. sin) cng l nghim ca (2). t
j+1
=
j
. thu
c cng thc nghim tng qut, trong cng thc (3) ta thay b phn
c
j

n
j
+ c
j+1

n
j+1
bi b phn tng ng:
c
j
r
n
cos n + c
j+1
r
n
sinn.
Nu phng trnh c trng (2) c nghim phc bi s

j
=
j+1
= =
j+s1
= r(cos + i. sin)
th (2) cng c nghim phc bi s lin hp vi
j
l
j
m ta t l

j+s
=
j+s+1
= =
j+2s1
= r(cos i. sin).
Trong trng hp ny, thu c cng thc nghim tng qut, trong cng
thc (3) ta thay b phn
c
j

n
j
+ c
j+1

n
j+1
+ + c
j+2s1

n
j+2s1
2.5. Phng trnh sai phn tuyn tnh cp 3 59
bi b phn tng ng
_
s1

i=0
c
j+i
n
i
_
r
n
cos n +
_
s1

i=0
c
j+s+i
n
i
_
r
n
sinn.
B. Tm nghim ring ca phng trnh sai phn tuyn tnh
khng thun nht. Vic tm nghim ring ca phng trnh sai phn
tuyn tnh khng thun nht cp k lm tng t nh tm nghim ring ca
phng trnh sai phn tuyn tnh khng thun nht cp hai v cp ba.
C. Tm nghim tng qut ca phng trnh sai phn tuyn tnh cp k.
Nghim tng qut c dng
y
n
= y
n
+ y

n
,
trong : +) y
n
l nghim ca phng trnh sai phn tuyn tnh cp k. +) y
n
l
nghim ca phng trnh thun nht tng ng. +) y

n
l mt nghim ring ca
phng trnh khng thun nht.
Chng 3
Xc nh s hng tng qut
ca mt dy s
Vic tnh gii hn ca mt dy s c cho bi cng thc truy hi thng
phi qua giai on chng minh s tn ti gii hn ca dy cho v sau s
dng h thc lim
n
x
n+1
= lim
n
x
n
i vi dy hi t bt k. iu thng
c thc hin bng cch s dng nguyn l Weierstrass ( iu kin dy
hi t ) hoc nguyn l hi t Bolzano - Cauchy. Qu trnh gp khng t kh
khn. Mt trong nhng phng thc khc phc kh khn l chuyn t cch
cho dy bng cng thc truy hi sang cho dy bng phng php gii tch, tc
l xc nh dy bng cng thc s hng tng qut ca n. Bi ton xc nh s
hng tng qut ca mt dy s c cho bi h thc truy hi l bi ton thng
gp trong chng trnh ph thng. Bi ton c pht biu nh sau. Xc nh
s hng tng qut ca dy s (x
n
) c cho bi h thc truy hi.
_
x
1
=
1
; x
2
=
2
; ; x
k
=
k
()
f(x
n+k
; x
n+k1
; ; x
n+1
; x
n
; n) = 0 (1)
quad(I)
trong
1
;
2
; ;
k
l cc s R, cho trc, cn f l mt biu thc cha
k + 2 bin, cho trc. Thc cht bi ton ang xt l bi ton xc nh hm s
x
n
= x(n) tho mn phng trnh sai phn (I) vi cc iu kin bin (). Do ,
i khi ta cng gi bi ton xc nh dy s c cho bi h thc truy hi (I) l
bi ton gii phng trnh sai phn (I).
60
3.1. Tm s hng tng qut ca dy (dng a thc) khi bit cc s hng u tin 61
3.1 Tm s hng tng qut ca dy (dng a thc)
khi bit cc s hng u tin
V d 3.1. Cho dy s:
1; 1; 1; 1; 5; 11; 19; 29; 41; 55; .
Hy tm quy lut biu din ca dy s v tm s tip theo.
Gii. Lp bng mt s sai phn ban u:
y = 1 -1 -1 1 5 11 19 29 41 55
y -2 0 2 4 6 8 10 12 14

2
y 2 2 2 2 2 2 2 2
Ta thy sai phn cp hai khng i nn dy s l dy cc gi tr ca a thc bc
hai:
y = an
2
+bn + c (a ,= 0)
trong n l s th t ca cc s trong dy s. Cho n = 0 ; 1 ; 2 (nh s cc
s bt u t 0) ta nhn c h phng trnh:
_

_
c = 1
a +b +c = 1
4a + 2b + c = 1

_
a = 1
b = 3
c = 1
.
Vy dy s tun theo quy lut sau:
y
n
= n
2
3n + 1
S hng u tin l y
0
= 1, s hng tip theo s hng 55 s ng vi n = 10 nn
s l:
y
10
= 10
2
3.10 + 1 = 71.
V d 3.2. Cho dy s:
5; 3; 11; 43; 99; 185; 307; 471; .
Hy tm quy lut biu din ca dy s v tm hai s hng tip theo.
Gii. Lp bng mt s sai phn ban u:
3.1. Tm s hng tng qut ca dy (dng a thc) khi bit cc s hng u tin 62
y = -5 -3 11 43 99 185 307 471
y 2 14 32 56 86 122 164

2
y 12 18 24 30 36 42

3
y 6 6 6 6 6
Ta thy sai phn cp ba khng i nn dy s l dy cc gi tr ca a thc bc
ba:
y = an
3
+bn
2
+cn + d (a ,= 0)
trong n l s th t ca cc s trong dy s. Cho n = 0 ; 1 ; 2 ; 3 (nh s
th t cc s hng bt u t 0) ta nhn c h phng trnh:
_

_
d = 5
a + b +c +d = 3
8a + 4b + 2c +d = 11
27a + 9b + 3c +d = 43

_
a = 1
b = 3
c = 2
d = 5
.
Vy dy s tun theo quy lut sau:
y
n
= n
3
+ 3n
2
2n 5.
S hng u tin l y
0
= 5, hai s hng tip theo s hng 471 s ng vi n = 8; 9
nn s l:
y
8
= 8
3
+ 3.8
2
2.8 5 = 683 ; y
9
= 9
3
+ 3.9
2
2.9 5 = 949.
Ch : 1) Quy lut tm c trn l khng duy nht v hin nhin, cc s hng
cho cng tho mn, chng hn quy lut:
y
n
= n
3
+3n
2
2n5+P(n).(n+5)(n+3)(n11)(n43)(n99)(n185)(n307)(n471)
trong P(x) l mt a thc bt k. Vy thc cht trn y ta mi ch tm c
mt quy lut m dy cc s cho tho mn m khng tm c tt c cc quy
lut m dy cc s cho tho mn. 2) Nh rng
2
(ax
2
+ bx + c) = Const,
nhng nu
2
y = Const th cha chc l (khng th suy ra c) y = ax
2
+bx+c.
Bi tp tng t
Bi ton 3.1. 1 Vi mi dy s sau y hy: a) Tm mt quy lut biu din ca
dy s. b) Vit hai s hng tip theo ca mi dy s theo quy lut va tm c
:
1. : 1; 2; 2; 1; 7; 16; 28; 43; 61; .
3.2. Cng thc truy hi l mt biu thc tuyn tnh 63
2. : 1; 6; 17; 34; 57; 86; 121; .
3. : 2; 3; 7; 14; 24; 37; .
4. : 3; 5; 10; 18; 29; .
5. : 5; 1; 5; 14; 28; 47; 71; 100; 134; 173; 217; .
Bi ton 3.2. 2 Tm quy lut ca cc dy s sau:
1. 2; 2; 8; 26; 62; 122; 212; 338 ; .
2. 1; 6; 17; 34; 57; 86; 121; 162; 209; 262 ; .
3. 5; 3; 11; 43; 99; 185; 307; 471; 683; 949 ; .
Bi ton 3.3. 3 Tm cng thc s hng tng qut ca dy s khi bit cc s
hng u tin.
1. 8; 14; 20; 26; 32 ; .
2. 0, 5 ; 1, 5 ; 4, 5 ; 13, 5 ; 40, 5 ; .
3. 2;
3
2
;
4
3
;
5
4
;
6
5
; .
4. 1; 3; 1; 3 ; .
5. 5; 7; 11; 19; 35 ; .
6. 1; 2; 6; 24; 120 ; .
7. 2 ;
1
2
;
4
3
;
3
4
;
6
5
; .
8. 0, 3 ; 0, 33 ; 0, 333 ; .
9.
1
2
;
1
2
;
3
8
;
1
4
;
5
32
; .
3.2 Cng thc truy hi l mt biu thc tuyn tnh
Trng hp h thc truy hi cho l h thc tuyn tnh.
a
0
x
n+k
+ a
1
x
n+k1
+ +a
k
x
n
= f(n).
vi a
0
; a
1
; ; a
k
(a
0
,= 0 ; a
k
,= 0) l cc hng s th bi ton c th c xem
nh mt phng trnh sai phn tuyn tnh v c gii nh trong chng trc.
Tuy nhin, cng c th gii bng cc phng php khc.
3.2. Cng thc truy hi l mt biu thc tuyn tnh 64
3.2.1 V d
V d 3.3. Tm s hng tng qut ca dy s (x
n
) c cho bi h thc truy
hi.
_
x
0
= 99
x
n+1
= x
n
2n 1 (1)
.
Gii. Coi (1) l phng trnh sai phn tuyn tnh cp 1. Do f(n) = 2n 1 l
a thc bc nht, = 1 nn ta chn x

n
= n(an +b). Thay vo (1) c.
(n + 1)[a(n + 1) +b] = n(an + b) 2n 1 a = 1 ; b = 0 x

n
= n
2
.
Cn x
n
= C.1
n
= C x
n
= C n
2
, m x
0
= 99 C 0
2
= 99 C = 99. Vy
phng trnh (1) c nghim. x
n
= 99 n
2
.
Gii (Cch 2). T h thc cho ta c.
x
0
= 99
x
1
= x
0
1
x
2
= x
1
3

x
n1
= x
n2
(2n 3)
x
n
= x
n1
(2n 1)
Cng tng v cc ng thc trn, ta c
x
n
= 99 [1 + 3 + 5 + + (2n 1)] = 99 n
2
.
Vy cng thc s hng tng qut ca dy s cn tm l x
n
= 99 n
2
.
V d 3.4. Tm s hng tng qut ca dy s (x
n
) c cho bi h thc truy
hi.
_
x
0
= 8
x
n+1
= 2x
n
+ 3
n
(2)
.
Gii. Do = 2 ,= 3 = nn ta chn x

n
= d.3
n
. Thay vo phng trnh (2)
c d = 1 x

n
= 3
n
. Cn x
n
= C.2
n
. Vy x
n
= C.2
n
+3
n
. Thay vo iu kin
bin c C = 7.
Tr li: phng trnh cho c nghim. x
n
= 7.2
n
+ 3
n
.
Gii (Cch khc). t y
n
= x
n
3
n
, ta c
_
y
0
= 8 1 = 7
y
n+1
+ 3.3
n
= 2(y
n
+ 3
n
) + 3
n

_
y
0
= 7
y
n+1
= 2y
n
3.2. Cng thc truy hi l mt biu thc tuyn tnh 65
T c (y
n
) l cp s nhn y
n
= 7.2
n
x
n
= 7.2
n
+3
n
l cng thc s hng
tng qut cn tm .
V d 3.5. Tm tt c cc dy s (a
n
) tho mn a
n+1
= 2
n
3a
n
v (a
n
) l mt
dy s tng.
Gii. Xt phng trnh sai phn. a
n+1
= 2
n
3a
n
(3).
t a
n
= u
n
.2
n
. Thay vo (3) c.
u
n+1
.2
n+1
= 3.u
n
.2
n
+ 2
n
u
n+1
=
3
2
u
n
+
1
2
. (3.1)
Phng trnh ny c nghim tng qut l.
u
n
= C.(
3
2
)
n
+
1
5
a
n
= C.(3)
n
+
1
5
.2
n
.
Ta c.
(a
n
) a
n+1
> a
n
3C.(3)
n
+
2
5
.2
n
> C.(3)
n
+
1
5
.2
n
vi mi n N
4C.(3)
n
<
1
5
.2
n
vi mi n N. ()
+) Vi C > 0 th ()
1
20C
> (
3
2
)
n
vi mi n N. Ta khng chn c C v
khi n chn th (
3
2
)
n
+.
+) Vi C < 0 th ()
1
20C
< (
3
2
)
n
vi mi n N. Ta cng khng chn c
C v khi n l th (
3
2
)
n
.
+) Vi C = 0 th a
n
=
1
5
.2
n
l dy s tng.
Vy dy s cn tm l. a
n
=
1
5
.2
n
.
V d 3.6. Cho a ; q ; d l cc s R, cho trc. Hy xc nh s hng tng
qut ca dy s (u
n
) c cho bi cng thc truy hi.
_
u
1
= a
u
n+1
= qu
n
+d (n 1)
. (4)
Hy xt tt c cc trng hp c th xy ra i vi cc tham s a ; q ; d.
(Dy s c cho bi cng thc trn cn c gi l cp s nhn - cng)
3.2. Cng thc truy hi l mt biu thc tuyn tnh 66
Gii.
+) Nu q = 0 th (4) xc nh dy s c cng thc s hng tng qut
u
1
= a ; u
n
= d, n N

, n 2.
+) Nu q = 1 th (4) xc nh mt cp s cng c cng thc s hng tng qut
u
n
= a + (n 1)d.
+) Nu d = 0 th (4) xc nh mt cp s nhn c cng thc s hng tng qut
u
n
= a q
n1
.
+) Ta xt trng hp d ,= 0 ; q ,= 0 ; q ,= 1. t u
n
:= v
n
+ (vi chn sau)
ta c
(4)
_
v
1
= a
v
n+1
+ = q(v
n
+ ) +d (n 1)
. (4.1)
Chn =
d
1 q
ta c (4.1) l h thc truy hi xc nh mt cp s nhn vi
cng bi q v do
v
n
= v
1
q
n1
u
n
=
_
a
d
1 q
_
q
n1
+
d
1 q
.
V d 3.7. Cho a ; b ; p ; q l cc s R, cho trc. Hy xc nh s hng
tng qut ca dy s (u
n
) c cho bi cng thc truy hi.
_
u
0
= a ; u
1
= b
u
n+1
= (p +q)u
n
pqu
n1
, (n 1) (5)
.
Hy xt tt c cc trng hp c th xy ra i vi cc tham s a ; b ; p ; q.
Gii. t v
n
= u
n
pu
n1
ta c c
v
1
= u
1
pu
0
= a pb ; v
n+1
= qv
n
v
n
= v
1
q
n1
().
Ap dng lin tip () ta c.
u
1
pu
0
= v
1
u
2
pu
1
= v
1
q
u
3
pu
2
= v
1
q
2

u
n
pu
n1
= v
1
q
n1
.
3.2. Cng thc truy hi l mt biu thc tuyn tnh 67
T cc dng thc trn d dng nhn c kt qu sau.
u
n
=
_
_
_
p
n
q
n
p q
b pq
p
n1
q
n1
p q
a nu p ,= q
np
n1
b (n 1)p
n
a nu p = q
.
V d 3.8. Cho a ; b ; p ; q ; r l cc s R, cho trc, pr ,= 0. Bit rng
phng trnh
pt
2
+qt +r = 0 ()
c hai nghim thc t = t
1
; t = t
2
. Hy xc nh s hng tng qut ca dy s
(u
n
) c cho bi cng thc truy hi.
_
u
1
= a ; u
2
= b
pu
n+2
+qu
n+1
+ru
n
= 0 (n 1)
. (6)
HDG. Chia hai v ca phng trnh cho p ri s dng nh l Vieete, a v v
d 5.
V d 3.9. Xc nh s hng tng qut ca dy s c cho bi h thc truy hi:
_
x
0
= 101
x
n+1
= 7.x
n
+ 7
n+1
(1.4)
.
Gii. Coi (1.4) l phng trnh sai phn tuyn tnh (cp 1), khng thun nht,
vi h s hng s. Do = 7 = nn ta chn x

n
= d.n.7
n
. Thay vo phng
trnh (1.4) c d = 1 x

n
= n.7
n
. Cn x
n
= C.7
n
. Vy x
n
= C.7
n
+ n.7
n
.
Thay vo iu kin bin c C = 101.
Tr li. phng trnh cho c nghim. x
n
= (101 +n).7
n
.
Gii (Cch khc). t x
n
= y
n
.7
n
. Ta thu c h thc truy hi i vi dy
s (y
n
).
_
x
0
= y
0
.7
0
y
n+1
.7
n+1
= 7.y
n
.7
n
+ 7
n+1

_
y
0
= 101
y
n+1
= y
n
+ 1
T ta thy (y
n
) l dy s cng vi s hng u y
0
= 101, cng sai d = 1. Theo
cng thc s hng tng qut ca dy s cng ta c
y
n
= y
0
+n.d y
n
= 101 + n.
Bi vy
x
n
= (101 +n).7
n
l cng thc s hng tng qut ca dy s cn tm.
3.2. Cng thc truy hi l mt biu thc tuyn tnh 68
V d 3.10. Xc nh cng thc s hng tng qut ca dy s (x
n
) c cho bi
h thc sau .
_
x
0
= 1 ; x
1
= 16
x
n+2
= 8x
n+1
16x
n
.
Gii. Coi h thc cho l phng trnh sai phn tuyn tnh (cp 2) thun
nht, vi h s hng s. Ta c phng trnh c trng.

2
8 + 16 = 0
1
=
2
= 4 (c nghim kp).
Vy. x
n
= x
n
= (A+ Bn).4
n
.
Gii iu kin bin.
_
x
0
= 1
x
1
= 16

_
A = 1
(A+ B).4 = 16

_
A = 1
B = 3
.
Vy phng trnh cho c nghim. x
n
= (1 + 3n).4
n
.
Gii. t x
n
= y
n
.4
n
. Ta thu c h thc truy hi i vi dy s (y
n
).
_

_
y
0
.4
0
= x
0
y
1
.4
1
= x
1
y
n+2
.4
n+2
= 8.y
n
.4
n+1
16.y
n
.4
n

_
y
0
= 1
y
1
= 4
y
n+2
= 2y
n+1
y
n
t tip z
n
= y
n+1
y
n
(n 0) ta c
_
z
0
= 3
z
n+1
= z
n
n 0
z
n
= 3 n 0
Nh vy, ta c h thc truy hi i vi dy s (y
n
) l.
_

_
y
0
= 1
y
1
= 4
y
n+1
= y
n
+ 3 (n 0)
T ta thy (y
n
) l dy s cng vi s hng u y
0
= 1, cng sai d = 3. Theo
cng thc s hng tng qut ca dy s cng ta c
y
n
= y
0
+ n.d y
n
= 1 + 3n.
Bi vy
x
n
= (1 + 3n).4
n
l cng thc s hng tng qut ca dy s cn tm.
3.2. Cng thc truy hi l mt biu thc tuyn tnh 69
Bi tp
1. Tm s hng tng qut ca dy s (u
n
) c cho bi.
u
n+1
= 3u
n
6n + 1 ; u
1
= 1.
p s: u
n
= 3n + 1 3
n
.
2. Tm s hng tng qut ca dy s (u
n
) c cho bi.
u
n+1
= u
n
+ 2n
2
; u
1
= 1.
p s: u
n
=
1
3
(2n
3
3n
2
+ n + 3).
3. Tm s hng tng qut ca dy s (u
n
) c cho bi.
u
n+1
= 5u
n
3
n
; u
0
= 1.
p s: u
n
=
1
2
(5
n
+ 3
n
).
4. Tm s hng tng qut ca dy s (u
n
) c cho bi.
u
n+1
= 2u
n
+ 6.2
n
; u
0
= 1.
p s: u
n
= (3n + 1).2
n
.
5. Tm s hng tng qut ca dy s (u
n
) c cho bi.
u
n+1
= u
n
+ 2n.3
n
; u
0
= 0.
p s: u
n
=
1
2
[(2n 3).3
n
+ 3].
6. Tm s hng tng qut ca dy s (u
n
) c cho bi.
u
n+1
2u
n
= (n
2
+ 1).2
n
; u
0
= 1.
p s: u
n
=
_
n(2n
2
3n + 7)
6
+ 2
_
.2
n
.
7. Tm s hng tng qut ca dy s (u
n
) c cho bi.
u
n+1
2u
n
= n + 3
n
; u
0
= 1.
p s. u
n
= 2
n
+ 3
n
n 1.
3.3. Cng thc truy hi l mt h biu thc tuyn tnh 70
3.3 Cng thc truy hi l mt h biu thc tuyn tnh
Xt bi ton sau: Xc nh s hng tng qut ca cc dy s (x
n
) ; (y
n
) tho
mn h thc truy hi dng.
_

_
x
1
= a ; y
1
= b ()
x
n+1
= px
n
+ qy
n
(1)
y
n+1
= rx
n
+ sy
n
(2)
(I) vi a; b; p; q; r; s l cc hng s R
Phng php gii. Trong (1) thay n bi n + 1 v bin i ta c.
x
n+2
= px
n+1
+qy
n+1
= px
n+1
+q(rx
n
+sy
n
)
= px
n+1
+qrx
n
+ s(x
n+1
px
n
)
x
n+2
(p + s)x
n+1
+ (ps qr)x
n
= 0
T (1) ta cng c x
2
= px
1
+ qy
1
= pa + qb. Vy ta thu c phng trnh sai
phn tuyn tnh cp hai thun nht:
_
x
1
= a ; x
2
= pa +qb
x
n+2
(p +s)x
n+1
+ (ps qr)x
n
= 0
M ta bit cch gii chng trc. Gii phng trnh ny ta tm c x
n
.
Thay vo (1) ta tm c y
n
.
3.3.1 V d
V d 3.11. Tm x
n
; y
n
tho mn.
_

_
x
1
= 1 ; y
1
= 1
x
n+1
= 4x
n
2y
n
(1)
y
n+1
= x
n
+ y
n
quad(2)
.
Gii. Trong (1) thay n bi n + 1 ta c
x
n+2
= 4x
n+1
2y
n+1
= 4x
n+1
2(x
n
+y
n
) = 4x
n+1
2x
n
2y
n
= 4x
n+1
2x
n
+ x
n+1
4x
n
= 5x
n+1
6x
n
x
n+2
5x
n+1
+ 6x
n
= 0.
3.3. Cng thc truy hi l mt h biu thc tuyn tnh 71
T (1) ta c: x
2
= 4x
1
2y
1
= 4.1 2.1 = 2. Vy ta c phng trnh sai phn
tuyn tnh thun nht:
_
x
1
= 1 ; x
2
= 2
x
n+2
5x
n+1
+ 6x
n
= 0
.
Gii phng trnh ny ta c x
n
= 2
n1
. Thay x
n
vo (1) c y
n
= 2
n1
. Vy
h cho c nghim.
_
x
n
= 2
n1
y
n
= 2
n1
.
V d 3.12. Tm x
n
; y
n
tho mn.
_

_
x
0
= 2 ; y
0
= 0
4x
n+1
= 2x
n
3y
n
(1)
2y
n+1
= 2x
n
+ y
n
quad(2)
.
Gii. Lp lun tng t v d trn ta c phng trnh sai phn tuyn tnh
thun nht:
_
x
0
= 2 ; x
1
= 1
x
n+2
x
n+1
+ x
n
= 0
.
Phng trnh c trng.

2
+ 1 = 0 = cos

3
i sin

3
.
Vy phng trnh trn c nghim tng qut.
x
n
= A. cos
n
3
+B. sin
n
3
.
Thay vo iu kin bin c A = 2 ; B = 0 x
n
= 2 cos
n
3
. Thay tip vo (1)
c y
n
=
4

3
sin
n
3
.
Vy h cho c nghim:
_

_
x
n
= 2 cos
n
3
y
n
=
4

3
sin
n
3
. Bi tp Tm x
n
, y
n
tho mn:
1.
_

_
x
0
= 2; y
0
= 2
x
n+1
=
1
2
x
n

3
4
y
n
y
n+1
= x
n
+
1
2
y
n
.
3.4. Cng thc truy hi l biu thc tuyn tnh vi h s bin thin 72
2.
_

_
x
0
= 0; y
0
= 6
x
n+1
= 3x
n
+y
n
y
n+1
= 5x
n
y
n
.
3.
_

_
x
0
= 2; y
0
= 1
x
n+1
= 2x
n
y
n
y
n+1
= x
n
+ 4y
n
.
4.
_

_
x
0
= 1; y
0
= 2
x
n+1
= 2x
n
8y
n
y
n+1
= 2x
n
6y
n
.
5.
_

_
x
0
= 1; y
0
= 1
x
n+1
= 4x
n
2y
n
+ 9n 3
y
n+1
= x
n
+ y
n
+ 3n
.
6.
_

_
x
0
= 1; y
0
= 1
1

2
x
n+1
= x
n
y
n
y
n+1
= x
n
+ y
n
.
3.4 Cng thc truy hi l biu thc tuyn tnh vi h
s bin thin
L thuyt v phng trnh sai phn tuyn tnh vi cc h s bin thin cho
n nay vn cha hon chnh. Vic gii cc phng trnh sai phn tuyn tnh vi
cc h s bin thin l rt phc tp. Trong phn ny ta s ch xt mt s dng
c bit, n gin ca cc phng trnh sai phn tuyn tnh vi cc h s bin
thin ch yu bng phng php t dy s ph, a v phng trnh sai phn
tuyn tnh.
V d 3.13. Tm u
n
bit rng.
u
1
= 0 ; u
n+1
=
n
n + 1
(u
n
+ 1) vi mi n 1.
Gii. T gi thit c: (n + 1)u
n+1
= nu
n
+ n. t x
n
= nu
n
, ta c
x
1
= 0 ; x
n+1
= x
n
+n.
3.4. Cng thc truy hi l biu thc tuyn tnh vi h s bin thin 73
Gii phng trnh ny ta c:
x
n
=
n(n 1)
2
. Vy ta c: u
n
=
n 1
2
.
V d 3.14. Tm u
n
bit rng.
u
1
= 0 ; u
n+1
=
n(n + 1)
(n + 2)(n + 3)
(u
n
+ 1) vi mi n 1.
Gii. T gi thit c:
(n + 1)(n + 2)
2
(n + 3)u
n+1
= n(n + 1)
2
(n + 2)u
n
+ n(n + 1)
2
(n + 2).
t x
n
= n(n + 1)
2
(n + 2)u
n
, ta c
x
1
= 0 ; x
n+1
= x
n
+n(n + 1)
2
(n + 2).
Gii phng trnh ny ta c:
x
n
=
(n 1)n(n + 1)(n + 2)(2n + 1)
10
.
Vy ta c p s:
u
n
=
(n 1)(2n + 1)
10(n + 1)
.
V d 3.15. Tm:
J
n
=
_

2
0
sin
n
x dx.
Gii. S dng cng thc tch phn tng phn ta c.
J
n
= cos x. sin
n1
x

2
0
+
_

2
0
(n 1) sin
n2
x cos
2
x dx
=
_

2
0
(n 1) sin
n2
x(1 sin
2
x) dx
= (n 1)
_

2
0
_
sin
n2
x sin
n
x
_
dx
= (n 1)(J
n2
J
n
).
J
n+2
=
n + 1
n + 2
J
n
(1).
3.4. Cng thc truy hi l biu thc tuyn tnh vi h s bin thin 74
D thy: J
0
=

2
; J
1
= 1. T v t (1) ta c
+) Khi n chn (n = 2k) th
J
2
=
1
2
J
0
J
4
=
3
4
J
2
J
6
=
5
6
J
4

J
2k
=
2k 1
2k
J
2k2
.
Nhn tng v cc ng thc trn v rt gn ta c.
J
2k
=
1.3. .(2k 1)
2.4. .(2k)
.

2
=
(2k 1)!!
(2k)!!
.

2
.
+) Khi n l (n = 2k + 1) th.
J
3
=
2
3
J
1
J
5
=
4
5
J
3
J
7
=
6
7
J
5

J
2k+1
=
2k
2k + 1
J
2k1
.
Nhn tng v cc ng thc trn v rt gn ta c.
J
2k+1
=
2.4. .(2k)
1.3. .(2k + 1)
.1 =
(2k)!!
(2k + 1)!!
.
p s.
J
2k
=
(2k 1)!!
(2k)!!
.

2
; J
2k+1
=
(2k)!!
(2k + 1)!!
.
V d 3.16. Tm x
n
bit rng.
x
1
= a > 0 ; x
n+1
= g(n).x
k
n
(1) vi mi n 1,
trong g(n) > 0 vi mi n N

; k R
+
.
3.4. Cng thc truy hi l biu thc tuyn tnh vi h s bin thin 75
Gii. T gi thit suy ra x
n
> 0 vi mi n N

. Ly logarit Neper hai v ca


(1) ta c
lnx
n+1
= ln g(n) +k. lnx
n
. (2)
t y
n
= lnx
n
, khi (2) c dng.
y
n+1
ky
n
= lng(n). (3)
t tip y
n
= k
n1
u
n
khi (3) c dng.
u
n+1
u
n
=
lng(n)
k
n
u
n
= u
1
+
n1

i=1
ln g(i)
k
i
.
T gi thit x
1
= a > 0 u
1
= ln a. Vy.
u
n
= lna +
n1

i=1
lng(i)
k
i
y
n
= k
n1
_
lna +
n1

i=1
lng(i)
k
i
_
.
Cui cng, ta c:
x
n
= e
k
n1
(lna+
n1
i=1
lng(i)
k
i
)
= exp
_
k
n1
(lna +
n1

i=1
lng(i)
k
i
)
_
.
V d 3.17. Tm x
n
bit rng.
x
1
= a > 0 ; x
n+1
=
f(n + 1)
f
k
(n)
.x
k
n
(1) vi mi n 1.
Trong f(n) > 0 vi mi n N

cn k N

, cho trc.
Gii. T (1) ta c:
x
n+1
f(n + 1)
=
x
k
n
f
k
(n)
. (2)
t dy ph: v
n
=
x
n
f(n)
, khi (2) c dng: v
n+1
= v
k
n
. (3). t tip dy s
ph: u
n
= lnv
n
, khi (3) c dng.
u
n+1
= ku
n
u
n
= C.k
n
(Vi C l hng s).
M x
1
= a v
1
=
a
f(1)
u
1
= ln
a
f(1)
= C.k u
n
= ln
a
f(1)
k
n1
. Vy ta c.
v
n
= e
k
n1
ln
a
f(1)
=
_
a
f(1)
_
k
n1
. Hay l. x
n
= f(n)(
a
f(1)
)
k
n1
.
3.4. Cng thc truy hi l biu thc tuyn tnh vi h s bin thin 76
V d 3.18. Tm s hng tng qut ca dy s (x
n
) bit rng x
1
= a v
x
n+1
= a(n)x
n
+b(n) quad(6)
trong a(n) ; b(n) l cc hm s i vi n N, a(n) ,= 0, n N.
Gii. t dy s ph
x
n
= y
n
.
n1

k=0
a(k)
Khi ta c y
1
=
a
a(0)
v
(6) y
n+1
y
n
=
b(n)

n
k=0
a(k)
:= g(n). quad(6.1)
T ng thc (6.1) ta d dng nhn c
y
n
= y
1
+
n1

k=1
g(k) =
a
a(0)
+
n1

k=1
g(k).
Vy nn ta c
x
n
=
_
a
a(0)
+
n1

k=1
b(k)

k
j=0
a(j)
_
n1

k=0
a(k).
l cng thc s hng tng qut cn tm.
V d 3.19. Tm s hng tng qut ca dy s (x
n
) bit rng x
1
= a ; x
2
= b
v
x
n+2
= a(n)x
n+1
+ b(n)x
n
+ f(n) quad(7)
trong a(n) ; b(n) ; f(n) l cc hm s i vi n N, b(n) ,= 0, n N v tn
ti s p ,= 0, tn ti hm s q(n) ,= 0 n N sao cho:
p +q(n) = a(n) ; p.q(n) = b(n). ()
Gii. S dng iu kin () ta c th vit li (7) di dng
(x
n+2
px
n+1
) q(n)(x
n+1
px
n
) = f(n) quad(7.1)
t y
n
= x
n+1
px
n
(
1
). Khi , (7.1) c dng.
y
n+1
= q(n)y
n
+ f(n) ; y
1
= b pa := .
3.4. Cng thc truy hi l biu thc tuyn tnh vi h s bin thin 77
Theo v d 6, ta tm c
y
n
=
_

q(0)
+
n1

k=1
f(k)

k
j=0
q(j)
_
n1

k=0
q(k) := h(n), n > 1.
Thay y
n
vo (
1
), gii phng trnh sai phn tuyn tnh cp 1 thu c ta tm
c cng thc s hng tng qut cn tm l.
x
n
= a.p
n1
+p
n
.
n1

k=1
h(k)
p
k+1
, n > 1.
V d 3.20. Tm s hng tng qut ca dy s (x
n
) nu bit
x
1
= 1 ; x
2
= 2 ; x
n+2
= (n + 1)(x
n+1
+ x
n
), n 1. (7)
Gii. t x
n
= n!y
n
. Khi , (7) c dng
n!y
n
= (n 1)[(n 1)!x
n1
+ (n 2)!x
n2
]
hay l
ny
n
(n 1)y
n1
y
n2
= 0 (y
n
y
n1
=
1
n
(y
n1
y
n2
), n 2.
T c
y
n
y
n1
=
(1)
n1
n!
, y
3
y
2
=
1
6
, y
2
y
1
=
1
2
.
Vit cc ng thc trn ri cng li ta c
y
n
= y
1
+
n1

k=2
(1)
k1
k!
= 1 +
n1

k=2
(1)
k1
k!
, n 3.
Vy cng thc s hng tng qut cn tm l
x
n
= n!
_
1 +
n1

k=2
(1)
k1
k!
_
, n 3.
Bi tp
Tm x
n
bit rng:
3.5. Cng thc truy hi dng phn tuyn tnh vi h s hng 78
1.
_
_
_
x
1
= 0
x
n+1
=
n
n + 1
(x
n
+ 1)
.
2.
_
_
_
x
1
= 0
x
n+1
=
n(n + 1)
(n + 2)(n + 3)
(x
n
+ 1)
.
3.
_

_
x
1
=
1
2
x
n+1
=
(n + 1)
2
n(n + 2)
x
n
+
n(n + 1)
n + 2
n!
.
4.
_
x
1
= a
x
n+1
= (n + 1)(x
n
+ 1)
.
5.
_
_
_
x
1
= 2
x
n+1
=
n
n + 2
x
n
+
4.3
n
(n + 1)(n + 2)
.
6.
_

_
x
1
= 8 ; x
2
=
33
2
x
n+2
2
(n + 1)(n + 3)
(n + 2)
2
x
n+1
3
n(n + 2)
(n + 1)(n + 3)
x
n
= 4
n(n + 3)
n + 2
.
3.5 Cng thc truy hi dng phn tuyn tnh vi h
s hng
Trong phn ny ta s tm s hng tng qut ca dy s c cho di dng
cng thc truy hi dng phn tuyn tnh vi h s hng thng qua cc v d c th.
V d 3.21. Tm dy s (x
n
) tho mn cc iu kin sau.
x
1
= a > 0 ; x
n+1
=
x
n
x
n
+ 2
vi mi n 1.
Gii. T gi thit suy ra x
n
> 0 vi mi n N

. M.
x
n+1
=
x
n
x
n
+ 2

1
x
n+1
= 1 +
2
x
n
.
t y
n
=
1
x
n
, khi ta c.
y
n+1
= 2y
n
+ 1 y
n+1
2y
n
1 = 0 ; y
1
=
1
a
.
3.5. Cng thc truy hi dng phn tuyn tnh vi h s hng 79
Gii phng trnh sai phn ny ta c:
y
n
=
(a + 1)2
n1
a
a
. Hay l. x
n
=
a
(a + 1)2
n1
a
V d 3.22. Tm dy s (x
n
) tho mn cc iu kin sau.
x
0
= a ; x
n+1
=
px
n
+q
rx
n
+s
vi mi n N. (1)
trong a, p, q, r, s R, cho trc.
Gii. Li gii ca v d ny thu c trc tip t B sau.
B 3.1. Nu y
n
v z
n
l nghim ca h phng trnh sai phn.
_
y
n+1
= py
n
+qz
n
; y
0
= a
z
n+1
= ry
n
+sz
n
; z
0
= 1
. (2)
th x
n
=
y
n
z
n
l nghim ca phng trnh.
x
0
= a ; x
n+1
=
px
n
+ q
rx
n
+ s
.
Chng minh. Tht vy, ta c: x
0
=
y
0
z
0
=
a
1
= a. Ngoi ra.
x
n+1
=
y
n+1
z
n+1
=
py
n
+ qz
n
ry
n
+ sz
n
=
p
y
n
z
n
+q
r
y
n
z
n
+ s
=
px
n
+ q
rx
n
+s
.
T suy ra pcm. T B trn ta c c cch gii ca phng trnh sai phn
dng phn tuyn tnh (1) bng cch lp v gii h phng trnh (2). T thu
c nghim ca (1) theo B . V d c th xem trong li gii ca v d 3 sau
y.
V d 3.23. Tm dy s (x
n
) tho mn cc iu kin sau.
x
0
= 0 ; x
n+1
=
x
n
+ 1
x
n
+ 1
vi mi n 1.
3.5. Cng thc truy hi dng phn tuyn tnh vi h s hng 80
Gii. Xt h phng trnh
_

_
y
0
= 0 ; z
0
= 1
y
n+1
= y
n
+z
n
z
n+1
= y
n
+z
n
.
Gii h ny ta c.
y
n
= (

2)
n
sin
n
4
; z
n
= (

2)
n
cos
n
4
.
T , theo B chng minh trn ta c nghim ca phng trnh cho
l.
x
n
=
y
n
z
n
=
(

2)
n
sin
n
4
(

2)
n
cos
n
4
= tg
n
4
.
V d 3.24. Tm cng thc s hng tng qut ca dy s (x
n
) c cho bi h
thc truy hi sau
x
1
= a ; x
n+1
=
bx
n
cx
n
+ d
quad(4).
Trong , a, b, c R

, d R.
Gii. t y
n
:=
1
x
n
;
c
b
:= p ;
d
b
:= q ta c
(4) y
n+1
= px
n
+q ; y
1
=
1
a
.
l h thc xc nh dy s nhn - cng. Cng thc s hng tng qut ca dy
s ny c xc nh trong phn trc (xem v d 4 mc 2.1 ca chng ny).
V d 3.25. Xc nh s hng tng qut ca dy s cho bi h thc truy hi
sau.
x
1
=
1
2
; x
n+1
=
1
2 x
n
.
Gii. Ta xc nh mt s s hng u tin.
x
1
=
1
2
; x
2
=
2
3
; x
3
=
3
4
.
Ta s chng minh dy s cho c s hng tng qut
x
n
=
n
n + 1
quad()
3.6. H thc truy hi phi tuyn 81
bng phng php quy np. Tht vy, theo trn, () ng ti n = 3. Gi s
() ng ti n, khi
x
n+1
=
1
2 x
n
=
1
2
n
n + 1
=
n + 1
n + 2
.
Vy () cng ng ti n + 1 nn theo nguyn l quy np ton hc, () ng vi
mi n N

. chnh l iu phi chng minh.


Bi tp
Tm dy s tho mn iu kin sau:
1. x
n+1
=
1 4x
n
1 6x
n
; x
0
= 1.
2. x
n+1
=
2x
n
2
3x
n
4
; x
0
= 1.
3. x
n+1
=
x
n
+ 1
x
n
+ 1
; x
0
= 0.
4. x
n+1
=
x
n
2
x
n
+ 4
; x
0
= 0.
5. x
n+1
=
x
n
1
x
n
+ 3
; x
0
= 1.
6. x
n+1
=
x
n
3
x
n
+ 1
; x
0
= 0.
7. x
n+1
=
x
n
2x
n
+ 1
; x
0
= 0.
3.6 H thc truy hi phi tuyn
Trong phn ny ta xt cc v d gii cc phng trnh sai phn phi tuyn. L
thuyt tng qut gii cc phng trnh dng ny cho n nay cn cha xy dng
c. Trong phn ny ch yu ta s xt cc phng trnh c th c tuyn tnh
ho bng php t hm ph hoc bng phng php quy np ton hc.
3.6. H thc truy hi phi tuyn 82
3.6.1 Quy trnh tuyn tnh ho mt phng trnh sai phn
Tuyn tnh ho mt phng trnh sai phn ngha l a mt phng trnh sai
phn dng phi tuyn v dng tuyn tnh. Gi s dy s (u
n
) tho mn iu
kin.
_
u
1
=
1
; u
2
=
2
u
k
=
k
u
n
= f(u
n1
, u
n2
, , u
nk
) vi n; k N

; n > k
.
Trong f l mt a thc i s bc m hoc l phn thc, hoc l biu thc siu
vit. Gi s hm s f(u
n1
, u
n2
, , u
nk
) c th tuyn tnh ho c, khi
tn ti cc gi tr x
1
; x
2
; ; x
k
sao cho.
u
n
= x
1
u
n1
+x
2
u
n2
+ +x
k
u
nk
(1)
tm x
1
; x
2
; ; x
k
trc ht ta xc nh u
k+1
; u
k+2
; ; u
2k
.
T cng thc lp cho ta c.
_

_
u
k+1
= f(
k
;
k1
; ;
2
;
1
) :=
k+1
u
k+2
= f(
k+1
;
k
; ;
3
;
2
) :=
k+2
quad quad quad
u
2k
= f(
2k1
;
2k2
; ;
k+1
;
k
) :=
2k
.
Thay cc gi tr u
1
; u
2
; ; u
k
cho v cc gi tr u
k+1
; u
k+2
; u
2k
va tm
c trn vo (1) ta c h phng trnh tuyn tnh gm k phng trnh vi
k n x
1
; x
2
; ; x
k
.
_

_
u
k+1
= x
1

k
+ x
2

k1
+ +x
k

1
u
k+2
= x
1

k+1
+ x
2

k
+ +x
k

2
quad quad quad
u
2k
= x
1

2k1
+x
2

2k2
+ +x
k

k
. ()
Gii h phng trnh ny ta thu c nghim: x
1
; x
2
; ; x
k
. Thay vo (1) ta s
c biu din tuyn tnh cn tm.
u
n
= f(u
n1
, u
n2
, , u
nk
) = x
1
u
n1
+x
2
u
n2
+ + x
k
u
nk
Sau ta chng minh cng thc biu din trn bng phng php quy np ton
hc.
Ch . Nu h (*) v nghim th hm f khng th tuyn tnh ho c.
3.6. H thc truy hi phi tuyn 83
3.6.2 V d
V d 3.26. Cho dy s (a
n
) tho mn.
a
1
= a
2
= 1 ; a
n
=
a
2
n1
+ 2
a
n2
() vi mi n 3
Hy tuyn tnh ho, tm s hng tng qut. Chng minh rng a
n
nguyn vi mi
n N

.
Gii. Gi s a
n
c biu din tuyn tnh l:
a
n
= a
n1
+a
n2
+ . (1)
Ta c.
a
3
=
a
2
2
+ 2
a
1
=
1 + 2
1
= 3.
a
4
=
a
2
3
+ 2
a
2
=
9 + 2
1
= 11.
a
5
=
a
2
4
+ 2
a
3
=
121 + 2
3
= 41.
Thay a
3
= 3 ; a
4
= 11 ; a
5
= 41 vo (1) ta thu c h phng trnh.
_

_
a
2
+a
1
+ = a
3
a
3
+a
2
+ = a
4
a
4
+a
3
+ = a
5

_
+ + = 3
3 + + = 11
11 + 3 + = 41

_
= 4
= 1
= 0
.
Vy ta c:
a
n
= 4a
n1
a
n2
(2)
Ta s chng minh dy s (a
n
) tho mn (*) c biu din tuyn tnh l.
a
1
= a
2
= 1 ; a
n
= 4a
n1
a
n2
vi mi n 3. (3)
Tht vy, vi n = 3 ta c: a
3
= 4.a
2
a
1
= 4.1 1 (2) ng vi n = 3.
3.6. H thc truy hi phi tuyn 84
Gi s (2) ng ti n = k tc l: a
k
= 4a
k1
a
k2
(k 3). Ta c.
a
k+1
=
a
2
k
+ 2
a
k1
=
(4a
k1
a
k2
)
2
+ 2
a
k1
=
16a
2
k1
8a
k1
a
k2
+a
2
k2
+ 2
a
k1
=
15a
2
k1
4a
k1
a
k2
+a
2
k1
4a
k1
a
k2
+ a
k1
a
k3
a
k1
( Nh rng: a
2
k2
+ 2 = a
k1
a
k3
)
=
15a
2
k1
4a
k1
a
k2
+a
k1
(a
k1
4a
k2
+a
k3
)
a
k1
=
15a
2
k1
4a
k1
a
k2
a
k1
(Do a
k1
4a
k2
+ a
k3
= 0)
= 15a
k1
4a
k2
= 4(a
k1
a
k2
) a
k1
= 4a
k
a
k1
.
Vy (2) cng ng ti n = k + 1. Theo nguyn l quy np ta c (2) ng vi
mi n N ; n 3.
T (3) ta thy ngay n N

: a
n
Z. Ngoi ra, ta chng minh c.
_
_
_
a
1
= a
2
= 1
a
n
=
a
2
n1
+ 2
a
n2
(n 3)
()
_
a
1
= a
2
= 1
a
n
= 4a
n1
a
n2
(n 3)
. ()
tm s hng tng qut ta gii phng trnh (**). C phng trnh c trng:

2
4 + 1 = 0 = 2 +

3 hoc = 2

3.
Do :
a
n
= a.(2 +

3)
n
+ b.(2

3)
n
. (4)
Thay vo iu kin bin ta tm c
a =
1
2
(3
5

3
) ; b =
1
2
(3 +
5

3
).
Vy ta c s hng tng qut cn tm l.
a
n
=
1
2
_
(3
5

3
)(2 +

3)
n
+ (3 +
5

3
)(2

3)
n
_
.
3.6. H thc truy hi phi tuyn 85
V d 3.27. Cho dy s (u
n
) tho mn.
u
1
= ; u
n+1
= au
n
+
_
bu
2
n
+ c vi a
2
b = 1 ; > 0 ; a > 1 ()
Hy tuyn tnh ho dy s trn.
Gii.
u
n+1
= au
n
+
_
bu
2
n
+ c u
n+1
au
n
=
_
bu
2
n
+c
(u
n+1
au
n
)
2
= bu
2
n
+ c
u
2
n+1
+ (a
2
b)u
2
n
= 2au
n+1
u
n
+c
u
2
n+1
+ u
2
n
= 2au
n+1
u
n
+c (1)
u
2
n
u
2
n1
= 2au
n
u
n1
+c (2)
Tr tng v (1) v (2) ta c .
u
2
n+1
u
2
n1
= 2au
n
(u
n+1
u
n1
)
M u
n+1
u
n1
> 0 nn suy ra.
u
n+1
2au
n
+ u
n1
= 0
Ni cch khc:
()
_
u
1
= ; u
2
= a +

b
2
+ c
u
n+1
2au
n
+ u
n1
= 0
Nh vy vic tuyn tnh ho thc hin xong.
V d 3.28. Cho dy s (x
n
) tho mn.
x
1
= ; x
n+1
=
x
n
a +
_
x
2
n
+ b
vi a
2
b = 1 ; > 0 ; a > 1 ()
Hy tuyn tnh ho dy s trn.
Gii.
x
n+1
=
x
n
a +
_
x
2
n
+ b

1
x
n+1
=
a
x
n
+
_
1 +
b
x
n
(3)
t u
n
=
1
x
n
. Khi ta c th vit (2) di dng.
u
1
=
1

; u
n+1
= au
n
+
_
bu
2
n
+ 1 vi a
2
b = 1 ; > 0 ; a > 1
chnh l phng trnh sai phn m ta tuyn tnh ho trong v d 2 trn.
3.6. H thc truy hi phi tuyn 86
V d 3.29. Cho dy s (u
n
) tho mn.
u
1
= ; u
2
= ; u
n+1
=
a +u
2
n
u
n1
vi a ; ; R ()
Hy tuyn tnh ho dy s trn.
Gii.
u
n+1
=
a +u
2
n
u
n1
u
n+1
u
n1
= u
2
n
+a (1)
u
n
u
n2
= u
2
n1
+a (2)
Tr tng v ca (1) v (2) ta c.
quadu
n+1
u
n1
u
n
u
n2
= u
2
n
u
2
n1
u
n+1
u
n1
+u
2
n1
= u
2
n
+u
n
u
n2

u
n
u
n+1
+u
n1
=
u
n1
u
n
+ u
n2

u
n
u
n+1
+u
n1
=
u
n1
u
n
+ u
n2
= =
u
2
u
3
+ u
1
=

2
+ +
2
:= k
Do u
n
= k(u
n+1
+ u
n1
) hay l.
()
_
u
1
= ; u
2
=
ku
n+1
u
n
+ku
n1
= 0
Nh vy vic tuyn tnh ho thc hin xong.
V d 3.30. Cho dy s (x
n
) tho mn.
x
1
= ; x
2
= ; x
n+1
=
x
2
n
+ 2bx
n
bx
n1
+c
b +x
n1
vi ; R ; n 2 (1)
Hy tuyn tnh ho dy s trn.
Gii.
(1) x
n+1
+b =
x
2
n
+ 2bx
n
bx
n1
+c
b +x
n1
+ b =
(x
n
+b)
2
+ c
x
n1
+b
(2)
t y
n
= x
n
+ b ta c phng trnh sai phn.
y
1
= +b ; y
2
= + b ; y
n+1
=
c +y
2
n
y
n1
vi c ; ; R ()
chnh l phng trnh sai phn m ta tuyn tnh ho trong v d 4 trn.
3.6. H thc truy hi phi tuyn 87
3.6.3 Mt s v d khc
V d 3.31. Xc nh s hng tng qut ca dy s f(n) (n N) c cho
bi.
f(0) = 2 ; f(n + 1) = 3f(n) +
_
8f
2
(n) + 1 (1) vi mi n 0
Gii. T gi thit ta c: f(n + 1) 3f(n) =
_
8f
2
(n) + 1 0 nn.
(f(n + 1) 3f(n))
2
= 8f
2
(n) + 1 f
2
(n + 1) + f
2
(n) = 6f(n)f(n + 1) + 1 ()
Thay n bi n 1 ta c:
f
2
(n) + f
2
(n 1) = 6f(n 1)f(n) + 1 ()
Tr tng v (*) v (**) ta c
f
2
(n + 1) f
2
(n 1) = 6f(n)(f(n + 1) f(n 1)) ( )
T gi thit ta cn c f(n) > 0 vi mi n (chng minh bng quy np). Ngoi ra.
f(n+1) > 3f(n) = 9f(n1)+3
_
8f
2
(n 1) + 1 > f(n1) f(n+1)f(n1) > 0
nn: ( ) f(n + 1) + f(n 1) = 6f(n). Vy ta c phng trnh sai phn
tuyn tnh.
_
f(0) = 2 ; f(1) = 6 +

33
f(n + 2) 6f(n + 1) + f(n) = 0
Gii phng trnh ny ta c.
f(n) =
(8 +

66)(3 +

8)
n
8
+
(8

66)(3

8)
n
8
D thy f(n) xc nh nh trn tho mn (1).
V d 3.32. Xc nh s hng tng qut ca dy s f(n) (n N

) c cho
bi.
f(1) = ; f(2) = ; f(n + 1) =
f
2
(n) +a
f(n 1)
() vi mi n 2
Gii. Khi , (*) c th c vit li di dng.
f(n + 1)f(n 1) = f
2
(n) +a vi mi n 2
Trong ng thc trn thay n bi n 1 ta c .
f(n)f(n 2) = f
2
(n 1) +a vi mi n 3
3.6. H thc truy hi phi tuyn 88
Tr tng v hai ng thc sau ta c.
f(n + 1)f(n 1) f(n)f(n 2) = f
2
(n) f
2
(n 1) vi mi n 3
Hay l.
f(n + 1)f(n 1) +f
2
(n 1) = f(n)f(n 2) +f
2
(n) vi mi n 3
T c.
f(n)
f(n + 1) +f(n 1)
=
f(n 1)
f(n) +f(n 2)
vi mi n 3
t g(n) :=
f(n)
f(n + 1) +f(n 1)
(n N 0; 1) ta c.
g(n) = g(n 1) vi mi n N 0; 1
Do .
g(n) = g(n 1) = = g(2) =
f(2)
f(3) +f(1)
=

2
+
2
+a
:= k
Ta c phng trnh sai phn tuyn tnh
f(1) = ; f(2) = ; kf(n + 2) f(n + 1) +kf(n) = 0 (n 1)
Gii phng trnh ny ta c biu thc ca f(n) cn tm.
Ch : Cc phng trnh dng.
1) f(n + 2) =
f
2
(n + 1) + 2bf(n + 1) bf(n) +c
f(n) + b
, (n N

) (1)
(Trong : f(1) = , f(2) = )
2) f(n + 1) =
f
2
(n)
(1 + af
2
(n))f(n 1)
, (n N, n 2) (2)
(Trong : a > 0, f(1) = ,= 0, f(2) = ,= 0)
3.6. H thc truy hi phi tuyn 89
c th a c v dng ca v d trn. Tht vy, ta c.
(1) f(n + 1) =
f
2
(n) + 2bf(n) + b
2
bf(n 1) b
2
+c
f(n 1) + b
=
[f(n) +b]
2
b[f(n 1) +b] + c
f(n 1) +b
=
[f(n) +b]
2
+c
f(n 1) + b
b
f(n + 1) + b =
[f(n) +b]
2
+c
f(n 1) + b
vi mi n 1
t g(n) = f(n + 1) + b Ta c.
g(n + 1) =
g
2
(n) +c
g(n 1)
, n N

.
l phng trnh c dng xt v d 2 trn. T cc iu kin ca phng
trnh (2) ta c f(n) ,= 0 vi mi n N

, do .
(2)
1
f(n + 1)
=
1 + af
2
(n)
f
2
(n)
.f(n 1)
=
_
1
f
2
(n)
+ a
_
1
1
f(n 1)
g(n + 1) =
g
2
(n) +a
g(n 1)
, n N

.
Trong , g(n) =
1
f(n)
, n N

, g(1) =
1

, g(1) =
1

l phng trnh c dng xt v d 2.


V d 3.33. Xc nh s hng tng qut ca dy s f(n) (n N

) c cho
bi.
f(1) =
9
8
; f(n + 1) = nf(n) + n.n! vi mi n N

Gii. Ta c nghim tng qut ca phng trnh f(n + 1) nf(n) = 0 l.

f(n) = C.1.2. .(n 1) = C.(n 1)!


Ta s tm nghim ring ca phng trnh cho di dng f

(n) = C(n).(n1)!.
Thay vo phng trnh cho c:
C(n+1).n! = nC(n).(n1)! +n.n! C = C(n+1)C(n) = n vi mi n N
3.6. H thc truy hi phi tuyn 90
T y d dng c.
C(n) =
1
2
(n
2
n) f

=
1
2
(n
2
n)(n 1)!
Do nghim tng qut ca phng trnh cho l.
f(n) = C.(n 1)! +
1
2
(n 1)
2
(n 1)!
Thay vo iu kin bin c C = 1. Vy f(n) = (n 1)! +
1
2
(n 1)
2
(n 1)!.
D thy f(n) xc nh nh trn tho mn bi ra.
V d 3.34. Xc nh s hng tng qut ca dy s f(n) (n N) c cho
bi.
f(0) = ; f(n + 1) = 2f
2
(n) 1 vi mi n N
Gii. Ta c. Nu = 1 th.
f(0) = 1
f(1) = 2f
2
(0) 1 = 1
f(2) = 2f
2
(1) 1 = 1
f(3) = 2f
2
(2) 1 = 1
Bng phng php quy np ta chng minh c: n N, f(n) = 1. Nu = 1
th tng t ta cng chng minh c: n N, f(n) = 1. Nu [[ < 1 th tn ti
sao cho cos = = arccos . Khi , ta c:
f(0) = cos = cos 2
0

f(1) = 2f
2
(0) 1 = 2 cos
2
1 = cos 2
1

f(2) = 2f
2
(1) 1 = 2 cos
2
2 1 = cos 2
2

f(3) = 2f
2
(2) 1 = 2 cos
2
2
2
1 = cos 2
3

Bng phng php quy np ta chng minh c: n N, f(n) = cos 2


n
. Nu
[[ > 1 th tn ti sao cho cosh = . Khi , ta c:
f(0) = cosh = cosh 2
0

f(1) = 2f
2
(0) 1 = 2 cosh
2
1 = cosh 2
1

f(2) = 2f
2
(1) 1 = 2 cosh
2
2 1 = cosh2
2

f(3) = 2f
2
(2) 1 = 2 cosh
2
2
2
1 = cosh 2
3

3.6. H thc truy hi phi tuyn 91


Bng phng php quy np ta chng minh c: n N, f(n) = cosh2
n
. V
cosh = nn:
e

+e

2
= . Gii phng trnh ny ta c.
e

=
_

2
1 hoc e

= +
_

2
1
Suy ra.
f(n) =
1
2
_
(e

)
2
n
+
1
(e

)
2
n
_
=
1
2
_
(
_

2
1)
2
n
+ ( +
_

2
1)
2
n
_
D thy cc dy s xc nh nh trn tho mn phng trnh cho. Vy ta c.
f(n) =
_

_
1 khi = 1
_
1 (n = 0)
1 (n 1)
khi = 1
cos 2
n
arccos khi [[ < 1
1
2
_
(
_

2
1)
2
n
+ ( +
_

2
1)
2
n
_
khi [[ > 1
V d 3.35. Xc nh s hng tng qut ca dy s f(n) (n N) c cho
bi.
f(0) = ; f(n + 1) = af
2
(n) +b (n N) vi ab = 2
HDG: t f(n) = bg(n), n N

, ta c.
g(0) =

b
:= ; g(n + 1) = 2g
2
(n) 1 (n N).
chnh l v d 4.
V d 3.36. Xc nh s hng tng qut ca dy s f(n) (n N) c cho
bi.
f(1) = ; f(n + 1) = f
2
(n) 2a
2
n
(n N

) vi a > 0.
HDG: t f(n) = 2a
2
n1
g(n), n N

, ta c.
g(1) =

2a
:= ; g(n + 1) = 2g
2
(n) 1 (n N

).
chnh l v d 4.
V d 3.37. Xc nh s hng tng qut ca dy s f(n) (n N) c cho
bi.
f(n + 1) = af
2
(n) +bf(n) +c (n N) (1) vi a ,= 0 ; c =
b
2
2b
4a
.
3.6. H thc truy hi phi tuyn 92
Gii. Khi ta c.
(1) f(n + 1) +
b
2a
= a
_
f(n) +
b
2a
_
2
+
b
2
4ac
4a
+
b
2a
= a
_
f(n) +
b
2a
_
2
+
b
2
(b
2
2b) + 2b
4a
= a
_
f(n) +
b
2a
_
2
g(n + 1) = ag
2
(n) (vi g(n) = f(n) +
b
2a
)
g(n + 1) = a[g(n)]
2
= a[ag
2
(n 1)]
2
= a
3
[g(n 1)]
2
2
=
=
= a
2
n
1
[g(1)]
2
n
= a
2
n
1
[ +
b
2a
]
2
n
g(n) = a
2
n1
1
[ +
b
2a
]
2
n1
f(n) = a
2
n1
1
[ +
b
2a
]
2
n1
+
b
2a
Bng php quy np ta d dng chng t c f(n) xc nh nh trn tho mn
phng trnh cho. Vy ta c p s.
f(n) = a
2
n1
1
[ +
b
2a
]
2
n1
+
b
2a
(n N

).
V d 3.38. Tm dy s (x
n
) tho mn cc iu kin sau.
x
1
= a ; x
n+1
=
x
2
n
+d
2x
n
vi mi n 1. (1).
Gii.
+) Nu d = 0 th ta c ngay.
x
n+1
=
1
2
x
n
x
n
= a.(
1
2
)
n1
.
+) Xt trng hp d > 0. Gi s u
n
; v
n
l mt nghim ca h phng trnh
sai phn.
_

_
u
1
= a ; v
1
= 1
u
n+1
= u
2
n
+dv
2
n
v
n+1
= 2u
n
v
n
, (2)
3.6. H thc truy hi phi tuyn 93
khi , x
n
=
u
n
v
n
l nghim ca phng trnh (1). Tht vy, ta c:
x
1
=
u
1
v
1
=
a
1
= a Khng nh ng vi n = 1.
Gi s khng nh ng ti n, tc l: x
n
=
u
n
v
n
l nghim ca (1). Khi .
x
n+1
=
u
n+1
v
n+1
=
u
2
n
+dv
2
n
2u
n
v
n
=
u
2
n
v
2
n
+ d
2
u
n
v
n
=
x
2
n
+d
2x
n
.
Vy x
n+1
cng l nghim ca (1). Tc l khng nh cng ng ti n + 1. Theo
nguyn l quy np ton hc, khng nh trn ng vi mi n N

. Vy, gii
(1) ta i gii (2). Vit li (2) di dng.
_

_
u
1
= a ; v
1
= 1
u
n+1
= u
2
n
+ dv
2
n
()

dv
n+1
= 2

du
n
v
n
()
. (3)
Cng tng v (*) v (**) ta c.
u
n+1
+

dv
n+1
= (u
n
+

dv
n
)
2
vi mi n 1.
T c.
u
n+1
+

dv
n+1
= (u
1
+

dv
1
)
2
n
= (a +

d)
2
n
. (4)
Tr tng v (*) v (**) ta c.
u
n+1

dv
n+1
= (u
n

dv
n
)
2
vi mi n 1.
T c.
u
n+1

dv
n+1
= (u
1

dv
1
)
2
n
= (a

d)
2
n
. (5)
T (4) v (5) ta c.
_

_
u
n+1
=
1
2
_
(a +

d)
2
n
+ (a

d)
2
n
_
v
n+1
=
1
2

d
_
(a +

d)
2
n
(a

d)
2
n
_
. (6)
Do x
n
=
u
n
v
n
nn t (6) ta c.
x
n
=

d
(a +

d)
2
n1
+ (a

d)
2
n1
(a +

d)
2
n1
(a

d)
2
n1
.
3.6. H thc truy hi phi tuyn 94
C th kim tra nghim ny tho mn bng cch th vo (1).
+)Xt trng hp d < 0. t d = q (q > 0). Tng t trn, ta s chng
minh.
Gi s u
n
; v
n
l mt nghim ca h phng trnh sai phn.
_

_
u
1
= a ; v
1
= 1
u
n+1
= u
2
n
qv
2
n
v
n+1
= 2u
n
v
n
, (7)
khi , x
n
=
u
n
v
n
l nghim ca phng trnh (1). Tht vy, ta c:
x
1
=
u
1
v
1
=
a
1
= a Khng nh ng vi n = 1.
Gi s khng nh ng ti n, tc l: x
n
=
u
n
v
n
l nghim ca (1). Khi .
x
n+1
=
u
n+1
v
n+1
=
u
2
n
qv
2
n
2u
n
v
n
=
u
2
n
v
2
n
q
2
u
n
v
n
=
x
2
n
+d
2x
n
.
Vy x
n+1
cng l nghim ca (1). Tc l khng nh cng ng ti n + 1. Theo
nguyn l quy np ton hc, khng nh trn ng vi mi n N

. Vy, gii
phng trnh (1) ta i gii h (7). Vit li (7) di dng.
_

_
u
1
= a ; v
1
= 1
u
n+1
= u
2
n
qv
2
n
( )
i

qv
n+1
= 2i

qu
n
v
n
( )
. (8)
Trong i l n v o (i
2
= 1). Cng tng v (***) v (****) ta c.
u
n+1
+i

qv
n+1
= (u
n
+ i

qv
n
)
2
vi mi n 1.
T c.
u
n+1
+i

qv
n+1
= (u
1
+ i

qv
1
)
2
n
= (a +i

q)
2
n
. (9)
Tr tng v (***) v (****) ta c.
u
n+1
i

qv
n+1
= (u
n
i

qv
n
)
2
vi mi n 1.
T c.
u
n+1
i

qv
n+1
= (u
1
i

qv
1
)
2
n
= (a i

q)
2
n
. (10)
3.6. H thc truy hi phi tuyn 95
T (9) v (10) ta c.
_

_
u
n+1
=
1
2
_
(a +i

q)
2
n
+ (a i

q)
2
n

v
n+1
=
1
2i

q
_
(a + i

q)
2
n
(a i

q)
2
n
. (11)
Do x
n
=
u
n
v
n
nn t (6) ta c.
x
n
= i

q
(a +i

q)
2
n1
+ (a i

q)
2
n1
(a +i

q)
2
n1
(a i

q)
2
n1
.
Cng c th kim tra nghim ny tho mn bng cch th vo (1).
V d 3.39. Bit rng dy s (x
n
) c dng x
n
= f(n), trong f(x) l a thc
bc khng qu 2. Hy xc nh cng thc tng qut ca dy s bit ba s hng
u: x
1
, x
2
, x
3
.
Gii. Gi s f(x) = ax
2
+bx +c. Theo gi thit ta c
_

_
a +b +c = x
1
4a + 2b +c = x
2
8a + 4b +c = x
3
Gii h ny ta c
_

_
a =
x
1
2x
2
+ x
3
2
b =
5x
1
8x
2
+ 3x
3
2
c = 3x
1
3x
2
+ x
3
Vy
f(n) =
x
1
2x
2
+ x
3
2
n
2

5x
1
8x
2
+ 3x
3
2
n + 3x
1
3x
2
+ x
3
.
l hm s cn tm.
Gii. Vit f(n) di dng.
f(n) = a(n 2)(n 3) +b(n 1)(n 3) + c(n 1)(n 2).
Ln lt cho n = 1, 2, 3 ta c
_

_
x
1
= f(1) = a.(1 2)(1 3) = 2a a =
x
1
2
x
2
= f(2) = b.(2 1)(2 3) = b b = x
2
x
3
= f(3) = c.(3 1)(3 2) = 2c c =
x
3
2
3.6. H thc truy hi phi tuyn 96
Vy cng thc cn tm c dng
x
n
= f(n) =
x
1
2
(n 2)(n 3) x
2
(n 1)(n 3) +
x
3
2
(n 1)(n 2).
V d 3.40. Xc nh s hng tng qut ca dy s cho bi h thc truy hi
sau.
x
1
= 0 ; x
n+1
=
x
n
+ 1
n + 1
.
Gii. Vit li iu kin cho di dng.
x
1
= 0 quad (1)
2x
2
= x
1
+ 1 quad(2)
3x
3
= x
2
+ 1 quad(3)

(n 1)x
n
= x
n2
+ 1 quad(n 1)
nx
n
= x
n1
+ 1 quad(n)
Nhn hai v ca ng thc th k trn vi (k 1)!, cng tng v cc ng thc
thu c v rt gn cc s hng ng dng hai v ta c.
n!.x
n
= 1! + 2! + + (n 1)! x
n
=
1
n!
.
n1

k=1
k!
l cng thc s hng tng qut cn tm.
3.6.4 Bi tp.
1. Xc nh s hng tng qut ca dy s f(n) (n N

) c cho bi h
thc truy hi.
f(1) = ; f(n + 1) = 2a
2
n
f
2
(n) a
(n+1)2
n
(n N

) vi a > 0.
p s.
f(n) =
_

_
a
n2
n1
(n N

) nu = a
_
a quadkhi n = 1
a
n2
n1
khi n 2
nu = a
a
n2
n1
cos 2
n1
(n N

) nu [[ < a
1
2
a
(n1)2
n1
_
_
+
_

2
a
2
_
2
n1
+
_

_

2
a
2
_
2
n1
_
(n N

) nu [[ > a
3.6. H thc truy hi phi tuyn 97
2. Xc nh s hng tng qut ca dy s f(n) (n N) c cho bi h thc
truy hi.
f(0) = ; f(n + 1) = af
3
(n) 3f(n) (n N) vi a > 0.
p s.
f(n) =
_

_
2

a
cos 3
n
arccos

a
2
(n N) nu [[
2

a
1
2
3
n
_
_

a
_

2
a 4
_
3
n
+
_

a +
_

2
a 4
_
3
n
_
(n N) nu [[ >
2

a
3. Xc nh s hng tng qut ca dy s f(n) (n N) c cho bi h thc
truy hi.
f(0) = ; f(n + 1) = f
3
(n) 3a
3
n
f(n) (n N) vi a > 0.
p s.
f(n) =
_

_
2

a
3
n
cos 3
n
arccos

2

a
(n N) nu [[ 2

a
1
2
3
n
_
_

_

2
4a
_
3
n
+
_
+
_

2
4a
_
3
n
_
(n N) nu [[ >
2

a
4. Xc nh s hng tng qut ca dy s f(n) (n N) c cho bi h thc
truy hi:
f(0) = ; f(n + 1) = af
3
(n) + 3f(n) (n N) vi a > 0 ; [[ >
2

a
.
p s:
f(n) =
1

a
_
_
_

a
2

_

2
a
4
+ 1
_
3
n
+
_

a
2
+
_

2
a
4
+ 1
_
3
n
_
_
(n N

)
5. Xc nh s hng tng qut ca dy s f(n) (n N) c cho bi h thc
truy hi:
f(0) = ; f(n + 1) = f
3
(n) + 3a
3
n
f(n) (n N) vi [[ > 2

a.
p s:
f(n) =
1
2
3
n
_
_
+
_

2
+ 4a
_
3
n
+
_

_

2
+ 4a
_
3
n
_
(n N).
3.6. H thc truy hi phi tuyn 98
6. Xc nh s hng tng qut ca dy s f(n) (n N

) c cho bi h
thc truy hi:
f(1) = ; f(n + 1) = af
3
(n) +bf
2
(n) + cf(n) + d (n N

)
vi a > 0 ; c =
b
2
3a
; d =
b(c 3)
9a
; >
b
3a
.
p s:
f(n) =
_
+
b
3a
_
3
n1
(

a)
3
n1
1

b
3a
(n N

).
7. Xc nh s hng tng qut ca dy s f(n) (n N) c cho bi h thc
truy hi:
f(0) = ; f(n + 1) = af
3
(n) + bf
2
(n) +cf(n) +d (n N).
vi a > 0 ; c =
b
2
+ 9a
3a
; d =
b
3
+ 18ab)
27a
2
; >
2

a

b
3a
.
p s:
f(n) =
1
2
3
n

a
_
_

a
_

2
a + 4
_
3
n
+
_

a +
_

2
a + 4
_
3
n
_

b
3a
(n N

).
( Vi = +
b
3a
).
8. Xc nh s hng tng qut ca dy s (x
n
) (n N

) c cho bi h thc
truy hi:
x
1
= a ; x
n+1
=
n + 1
x
n
+ 1
(n 1).
p s:
x
n
= n!
_
a +
n1

k=1
1
k!
_
.
9. Xc nh s hng tng qut ca dy s (x
n
) (n N

) c cho bi h thc
truy hi:
x
1
=
1
2
; x
n+1
=
2
3 x
n
(n 1).
p s:
x
n
=
3.2
n1
2
3.2
n1
1
.
Chng 4
Phng trnh hm sai phn
bc hai
Trong chng ny, ta gii hai bi ton v phng trnh hm tuyn tnh thun
nht bc hai i vi hm tun hon v phn tun hon cng tnh; phng trnh
hm tuyn tnh thun nht bc hai i vi hm tun hon v phn tun hon
nhn tnh. Ta s gii quyt hai bi ton trn da vo kt qu ca cc bi ton v
phng trnh hm tuyn tnh bc nht c trong ti liu tham kho [3].
4.1 Hm tun hon v phn tun hon cng tnh
nh ngha 4.1. Cho hm s f(x) v tp M (M D(f)) Hm f(x) c gi
l hm tun hon trn M nu tn ti s dng a sao cho
_
x M ta u c x a M
f(x +a) = f(x), x M
a c gi l chu k ca hm tun hon f(x).
Chu k nh nht (nu c) trong cc chu k ca f(x) c gi l chu k c s
ca hm tun hon f(x).
nh ngha 4.2. Cho hm s f(x) v tp M (M D(f)) Hm f(x) c gi
l hm tun hon trn M nu tn ti s dng a sao cho
_
x M ta u c x a M
f(x +a) = f(x), x M
a c gi l chu k ca hm tun hon f(x).
99
4.2. Phng trnh hm sai phn bc hai vi hm tun hon v phn tun hon 100
Chu k nh nht (nu c) trong cc chu k ca f(x) c gi l chu k c s
ca hm tun hon f(x).
nh ngha 4.3. f(x) c gi l hm tun hon nhn tnh chu k a (a /
0, 1, 1) trn M nu M D(f) v
_
x M a
1
x M
f(ax) = f(x), x M
nh ngha 4.4. f(x) c gi l hm phn tun hon nhn tnh chu k a
(a / 0, 1, 1) trn M nu M D(f) v
_
x M a
1
x M
f(ax) = f(x), x M
4.2 Phng trnh hm sai phn bc hai vi hm tun
hon v phn tun hon
Bi ton 4.1. Cho a R0; , R, ,= 0. Tm tt c cc hm f : R R
tho mn iu kin:
f(x + 2a) +f(x +a) +f(x) = 0 (4.1)
Phng trnh c dng (1.2.1) c gi l phng trnh tuyn tnh thun nht bc
hai
Gii. Xt phng trnh

2
+ + = 0 (4.2)
(gi l phng trnh c trng ca phng trnh (1.2.1))
C
=
2
4
a) Trng hp > 0
Khi phng trnh (4.2) c hai nghim thc
1
,=
2
. p dng nh l Viete:
_

1
+
2
=

2
=
thay vo (4.1)
(4.1) f(x + 2a) (
1
+
2
)f(x + a) +
1

2
f(x) = 0
f(x + 2a)
1
f(x + a) =
2
[f(x +a)
1
f(x)] (4.3)
4.2. Phng trnh hm sai phn bc hai vi hm tun hon v phn tun hon 101
t g
1
(x) = f(x +a)
1
f(x), (1.2.3) tr thnh
g
1
(x +a) =
2
g
1
(x) (1.2.3*)
t g
1
(x) = [
2
[
x
a
.h(x). Khi ta c h(x +a) =
_
h(x) nu
2
> 0
h(x) nu
2
< 0
Khi ta
c
f(x +a)
1
f(x) = [
2
[
x
a
h
1
(x) (4.4)
i vai tr
2
cho
1
v bin i tng t ta c
f(x +a)
2
f(x) = [
1
[
x
a
h
2
(x) (4.5)
Tr (4.4) cho (1.2.5) ta c
(
2

1
)f(x) = [
2
[
x
a
h
1
(x) [
1
[
x
a
h
2
(x)
Vy
f(x) =
1

1
_
[
2
[
x
a
h
1
(x) [
1
[
x
a
h
2
(x)
_
trong h
1
(x) v h
2
(x) l hai hm tu tho mn:
h
1
(x +a) =
_
h
1
(x) nu
2
> 0
h
1
(x) nu
2
< 0
; h
2
(x +a) =
_
h
2
(x) nu
1
> 0
h
2
(x) nu
1
< 0
b) Trng hp = 0
Tc l

2
4 = 0 hay =

2
4
Khi phng trnh (1.2.2) c nghim kp
1
=
2
=

2
. Do
(4.1) f(x + 2a) +f(x + a) +

2
4
f(x) = 0
f(x + 2a) +
_

2
_
f(x +a) =

2
_
f(x + a) + (

2
)f(x)
_
(4.6)
t f(x + a) +

2
f(x) = g(x)
b1) Trng hp

2
= 1 hay = 2.
Khi (1.2.6) tr thnh
f(x + 2a) f(x + a) = f(x +a) f(x)
f(x + a) f(x) = g(x), (1.2.6*)
4.2. Phng trnh hm sai phn bc hai vi hm tun hon v phn tun hon 102
vi g(x +a) = g(x)
Ta c g(x) =
(x+a)x
a
g(x) =
x+a
a
g(x+a)
x
a
g(x), phng trnh (1.2.6*) tr thnh
f(x + a)
(x+a)
a
g(x +a) = f(x)
x
a
g(x)
t f(x)
x
a
g(x) = h(x) ta c
f(x) = h(x) +
xg(x)
a
trong h(x) l hm tu sao cho h(x +a) = h(x).
b2) Trng hp 0 <

2
,= 1 hay 2 ,= < 0 Bi ton quy v vic gii phng
trnh dng
f(x + a) + (

2
)f(x) = g(x)
vi g(x +a) =

2
g(x)
Tng t vic gii (1.2.3*) ta c
g(x) = (

2
)
x
a
h
1
(x), vi h
1
(x + a) = h
1
(x)
Suy ra
f(x +a) (

2
)f(x) = (

2
)
x
a
h
1
(x)

f(x +a)
(

2
)
x
a
(

2
)
f(x)
(

2
)
x
a
= h
1
(x)

f(x + a)
(

2
)
x+a
a

f(x)
(

2
)
x
a
=
h
1
(x)

2
(4.7)
t
f(x)
(

2
)
x
a
= I
1
(x);
h
1
(x)

2
= h
2
(x)
(4.7) I
1
(x + a) I
1
(x) = h
2
(x), vi h
2
(x +a) = h
2
(x)
4.2. Phng trnh hm sai phn bc hai vi hm tun hon v phn tun hon 103
Tng t cch gii (1.2.6*) ta c
I
1
(x) = k
1
(x) +
xh
2
(x)
a
= k
1
(x)
2xh
1
(x)
a
f(x) = (

2
)
x
a
_
k
1
(x)
2xh
1
(x)
a
_
trong k
1
(x) l hm tu tho mn k
1
(x + a) = k
1
(x).
b3) Trng hp

2
< 0 hay > 0.
Bi ton quy v vic gii phng trnh
f(x + a) +

2
f(x) = g(x) (4.8)
vi g(x +a) = (

2
)g(x).
Tng t vic gii (1.2.3*) ta c
g(x) =

x
a
h
3
(x) =
_

2
_x
a
h
3
(x) (4.9)
vi h
3
(x + a) = h
3
(x) (v

2
< 0)
T (4.8) v (4.9) ta c
f(x +a) +

2
f(x) =
_

2
_x
a
h
3
(x)

f(x +a)
_

2
_x
a
+

2
f(x)
_

2
_x
a
= h
3
(x)

f(x +a)
_

2
_x+a
a
+
f(x)
_

2
_x
a
=
h
3
(x)

2
t
f(x)
_

2
_x
a
= I
2
(x);
2h
3
(x)

= h
4
(x)
vi h
4
(x) l hm tu tho mn: h
4
(x+a) = h
4
(x), (v: h
4
(x+a) =
2h
3
(x +a)

=
2h
3
(x)

= h
4
(x))
Khi ta c
I
2
(x + a) + I
2
(x) = h
4
(x). (1.2.9*)
4.2. Phng trnh hm sai phn bc hai vi hm tun hon v phn tun hon 104
Ta c h
4
(x) =
(ax)+x
a
h
4
(x) =
x
a
h
4
(x + a)
xa
a
h
4
(x).
V vy (1.2.9*) tr thnh k
2
(x) = k
2
(x)
vi k
2
(x) = I
2
(x) +
xa
a
h
4
(x).
Suy ra
I
2
(x) = k
2
(x)
2(x a)h
3
(x)
a
vi k
2
(x) l hm tu tho mn k
2
(x +a) = k
2
(x)
Vy
f(x) = (

2
)
x
a
_
k
2
(x)
2(x a)h
3
(x)
a
_
c) Trng hp < 0
Phng trnh (1.2.2) c hai nghim phc lin hp
1
,
2
C;
1
=
2
, do
t
1
= p iq,
2
= p + iq suy ra [
0
[ = [
1
[ = [
2
[
_
p
2
+q
2
, arg
2
= =
arg
1
, tan =
q
p
. Bin i tng t nh trng hp > 0 ta c
g
1
(x +a) =
2
g
1
(x) (*)
vi g
1
(x) = f(x +a)
1
f(x).
Nh vy hm: g
1
: R C
Ta c
() g
1
(x +a) = e
ln
2
g
1
(x)
g
1
(x +a)
e
x
a
ln
2
= e
ln
2
.
g
1
(x)
e
x
a
ln
2

g
1
(x +a)
e
x+a
a
ln
2
=
g
1
(x)
e
x
a
ln
2

g
1
(x +a)
e
x+a
a
ln
2
=
g
1
(x)
e
x
a
ln
2
t
g
1
(x)
e
x
a
ln
2
= h
1
(x) (4.10)
Ta c
h
1
(x +a) = h
1
(x)
i vai tr ca
1
,
2
cho nhau v bin i tng t nh trn ta c
h
2
(x +a) = h
2
(x)
Vi
g
2
(x)
e
x
a
ln
1
= h
2
(x) (4.11)
4.2. Phng trnh hm sai phn bc hai vi hm tun hon v phn tun hon 105
T (4.10) v (4.11) ta c
_
g
1
(x) = e
x
a
ln
2
h
1
(x)
g
2
(x) = e
x
a
ln
1
h
2
(x)
(i)
Ta chng minh h
1
(x) = h
2
(x)
Tht vy, trc ht ta chng minh g
1
(x) = g
2
(x).
Ta c g
1
(x) = f(x + a)
1
f(x),
Ly x
0
bt k, x
0
R.
Ta c g
1
(x
0
) = f(x
0
+a)
1
f(x
0
) = f(x
0
+a)
1
f(x
0
) = f(x
0
+a)
2
f(x
0
) =
g
2
(x
0
).
V x
0
bt k nn x R ta c
g
1
(x) = g
2
(x) (4.12)
Tip theo ta chng minh
e
x
a
ln
2
= e
x
a
ln
1
Tht vy
e
x
a
ln
2
= e
x
a
(ln|
2
|+iarg
2
+2ki)
= e
x
a
ln|
2
|
.e
iarrg
2
x
a
e
i2k
x
a
= e
x
a
ln|
2
|
_
cos x
a
+ i sin
x
a
__
cos
2kx
a
+ i sin
2kx
a
_
= e
x
a
ln|
2
|
_
cos
x
a
+ i sin
x
a
__
cos
2kx
a
+ i sin
2kx
a
_
= e
x
a
ln|
2
|
_
cos
x
a
i sin
x
a
__
cos
2kx
a
i sin
2kx
a
_
= e
x
a
ln|
2
|
_
cos
_

x
a
_
+i sin
_

x
a
___
cos
_

2kx
a
_
+i sin
_

2kx
a
__
= e
x
a
ln|
2
|
e
i(
x
a
)
e
i(
2kx
x
)
= e
x
a
ln|
1
|
e
i
arg
1
x
a
e
i
2k

x
a
= e
x
a
(ln|
1
|+iarg
1
+2k

i)
= e
x
a
ln
1
(4.13)
y: (arg
1
= ; arg
2
= ; k = k

)
T (4.12) v (4.13):
_
g
1
(x)
e
x
a
ln
1
_
=
g
2
(x)
e
x
a
ln
1
h
1
(x) = h
2
(x)
Theo trn ta c cc hm h
1
: R C; h
2
: R C
4.2. Phng trnh hm sai phn bc hai vi hm tun hon v phn tun hon 106
Nh vy ta s t: h
1
(x) = m(x) + in(x); trong cc hm:
m : R R; n : R R
Theo chng minh trn: h
1
(x) = h
2
(x) h
2
(x) = m(x) in(x)
Quay tr li bi ton ban u ta c
_
f(x + a)
1
f(x) = e
x
a
ln
2
h
1
(x)
f(x + a)
2
f(x) = e
x
a
ln
1
h
2
(x)
(4.14)
Tr (??) cho (??)
f(x) =
1

1
_
e
x
a
ln
2
h
1
(x) e
x
a
ln
1
h
2
(x)
_
=
1

1
_
e
x
a
ln
2
h
1
(x) e
x
a
ln
2
h
2
(x)
_
=
1

2

1
_
e
x
a
(ln|
2
|+iarg
2
+2ki)
h
1
(x) e
x
a
(ln|
2
|+iarg
2
+2ki)
h
2
(x)
_
V hm e
x
a
ln
2
l hm a tr, ta s chn mt nhnh lin tc bng cch chn
k = 0, nn ta c:
f(x) =
1

2

1
_
e
x
a
(ln|
2
|+iarg
2
)
h
1
(x) e
x
a
(ln|
2
|+iarg
2
)
h
2
(x)
_
=
1

2

1
_
e
x
a
ln|
2
|
_
cos
x
a
+ i sin
x
a
_
h
1
(x) e
x
a
ln|
2
|
_
cos
x
a
i sin
x
a
_
h
2
(x)
_
=
e
x
a
ln |
0
|
2iq
_
cos
x
a
(h
1
(x) h
2
(x)) + i sin
x
a
(h
1
(x) + h
2
(x))
_
=
e
x
a
ln |
0
|
2iq
_
2i cos
x
a
.n(x) + 2i sin
x
a
.m(x)
_
=
e
x
a
ln |
0
|
q
_
cos
x
a
.n(x) + sin
x
a
.m(x)
_
=
[
0
[
x
a
q
_
cos
x
a
.n(x) + sin
x
a
.m(x)
_
Trong n(x) v m(x) l hai hm s bt k tho mn
n(x +a) = n(x); m(x + a) = m(x)
n : R R; m : R R

1
= p iq

2
= p +iq
_
[
1
[ = [
2
[ =
_
p
2
+q
2
; arg
2
= = arg
1
4.2. Phng trnh hm sai phn bc hai vi hm tun hon v phn tun hon 107
tan =
q
p
; arg
2
=
Kt lun:
+) > 0
f(x) =
1

1
_
[
2
[
x
a
.h
1
(x) [
1
[
x
a
.h
2
(x)
_
h
1
(x) v (h
2
(x) l hai hm tu tho mn:
h
1
(x +a) =
_
h
1
(x) nu
2
> 0
h
1
(x) nu
2
< 0
h
2
(x +a) =
_
h
2
(x) nu
1
> 0
h
2
(x) nu
1
< 0
+) = 0
Trng hp 1: = 2 f(x) = h(x) +
xg(x)
a
h(x) v g(x) l hai hm tu tho mn:
_
h(x + a) = h(x)
g(x + a) = g(x)
Trng hp 2: 2 ,= < 0
f(x) =
_


2
_x
a
_
k
1
(x)
2xh
1
(x)
a
_
k
1
(x) v h
1
(x) l hai hm tu tho mn:
_
h
1
(x +a) = h
1
(x)
k
1
(x + a) = k
1
(x)
Trng hp 3: > 0
f(x) =
_

2
_x
a
_
k
2
(x)
2(x a)h
2
(x)
a
_
k
2
(x) v h
2
(x) l hai hm tu tho mn:
_
h
2
(x +a) = h
2
(x)
k
2
(x + a) = k
2
(x)
+) < 0
f(x) =
[
0
[
x
a
q
_
cos
x
a
.n(x) + sin
x
a
.m(x)
_
m(x) v n(x) l hai hm tu tho mn
_
m(x +a) = m(x); m : R R
n(x + a) = n(x); n : R R
4.3. Phng trnh vi hm s tun hon, phn tun hon nhn tnh 108

1
= p iq;
2
= p + iq
[
1
[ = [
2
[ = [
0
[ =
_
q
2
+q
2
; arg
2
= arg
1
=
Cc v d p dng
V d 4.1. Tm tt c cc hm f : R R tho mn iu kin:
f(x + 2) 8f(x + 1) + 15f(x) = 16 (1)
Li gii. t f(x) = g(x) +C; (C: xc nh sau)
(1) g(x + 2) 8g(x + 1) + 15g(x) + 8C = 16.
Chn C = 2
g(x + 2) 8g(x + 1) + 15g(x) = 0.
Xt phng trnh c trng:

2
8 + 15 = 0; = 1 > 0

1
= 3;
2
= 5
p dng cng thc nghim vi > 0 ta c
g(x) =
1
2
[5
x
h
1
(x) 3
x
h
2
(x)] f(x) =
1
2
[5
x
h
1
(x) 3
x
h
2
(x)] + 2
h
1
(x) v h
2
(x) l hai hm tu tho mn
_
h
1
(x + 1) = h
1
(x); h
1
: R R
h
2
(x + 1) = h
2
(x); h
2
: R R
V d 4.2. Tm tt c cc hm f : R R tho mn iu kin
4.3 Phng trnh vi hm s tun hon, phn tun
hon nhn tnh
Phng trnh hm l mt chuyn c bn ca chng trnh ton cho
cc trng THPT Chuyn. Cc bi ton v phng trnh hm cng l nhng bi
tp thng gp trong cc k thi hc sinh gii ton cp Quc gia, thi Olympic khu
vc hay Olympic Quc t. Phng trnh hm tuyn tnh bc hai l mt vn
quan trng trong lp phng trnh hm ni chung. Trong chng trnh ton cho
cc trng THPT chuyn,phng trnh hm tuyn tnh bc hai c cp trong
trng hp > 0 ca phng trnh c trng:
2
+ + = 0() i vi hm
4.3. Phng trnh vi hm s tun hon, phn tun hon nhn tnh 109
tun hon cng tnh; cc trng hp = 0 v < 0 ca phng trnh () cha
c cp n. Ngoi ra, phng trnh hm tuyn tnh bc hai i vi hm tun
hon nhn tnh cha c cp n c ba trng hp: > 0; = 0v < 0
ca phng trnh (). Hn th na, phng trnh hm tuyn tnh bc hai i vi
hm tun hon cng tnh v nhn tnh cng cha c cp n. Bo co ny
a ra ba bi ca phng trnh hm tuyn tnh bc hai vi v phi l hm s i
vi hm tun hon v phn tun hon nhn tnh.
4.3.1 nh ngha
Cho a R0; 1; 1; , R. Tm tt c cc hm: f : R0 R tha
mn iu kin:
f(a
2
x) + f(ax) + f(x) = g(x).
trong g(x) l hm cho trc.
4.3.2 Mt s bi ton
Bi ton 4.2. Cho h(x) l hm tun hon nhn tnh chu k a trn R(h(ax) =
h(x)); a R0; 1; 1; , R. Tm tt c cc hm: f : R0 R tha mn
iu kin:
f(a
2
x) +f(ax) +f(x) = h(x). (4.15)
Li gii:
Xt phng trnh c trng:

2
+ + = 0; =
2
4. (4.16)
a) Trng hp > 0 :
Phng trnh (2.2.40) c hai nghim thc:
1
,=
2
. p dng nh l Viete ta
c:
_

1
+
2
=

2
=
Thay vo (2.2.39):
(2.2.39) f(a
2
x) (
1
+
2
)f(ax) +
1

2
f(x) = h(x)
f(a
2
x)
1
f(ax)
2
[f(ax)
1
f(x)] = h(x) (4.17)
4.3. Phng trnh vi hm s tun hon, phn tun hon nhn tnh 110
t g
1
(x) = f(ax)
1
f(x) ta c:
(2.2.41) g
1
(ax)
2
g
1
(x) = h(x) (4.18)
a1) Trng hp 1:
1
= 1 hoc
2
= 1.
Khng mt tnh tng qut ta gi s:
2
= 1. Khi :
(2.2.42) g
1
(ax) g
1
(x) = h(x) (theo Bi ton 1.1

- chng 1)
g
1
(x) = g(x) +
ln[x[h(x)
ln[a[
trong g(x) l hm ty sao cho:
g(ax) = g(x). Hay:
f(ax)
1
f(x) = g(x) +
ln[x[h(x)
ln[a[
. (4.19)
i vai tr
2
cho
1
v bin i tng t ta c:
g
2
(ax)
1
g
2
(x) = h(x)
trong :g
2
(x) = f(ax) f(x).
V
2
= 1 v
1
,=
2

1
,= 1. Theo Bi ton 1.3

chng 1 ta c:
g
2
(x) =
h(x)
1
1
+[x[
log
|a|
|
1
|
.q(x)
trong q(x) l hm ty sao cho:
q(ax) =
_
q(x) nu
1
> 0
q(x) nu
1
< 0
Hay:
f(ax) f(x) =
h(x)
1
1
+[x[
log
|a|
|
1
|
.q(x) (4.20)
Tr (2.2.43) cho (2.2.44) ta c:
f(x) =
ln[x[(1
1
) ln [a[
ln [a[(1
1
)
2
h(x) +
1
1
1
_
g(x) +[x[
log
|a|
|
1
|
q(x)

.
a2) Trng hp 2:
1
,= 1 v
2
,= 1 1 + + ,= 0.
(v nu 1 + + = 0
1
= 1 hoc
2
= 1 iu ny mu thun vi gi thit.)
Ta c:
h(x) =
1 + +
1 + +
h(x) =
h(a
2
x) + h(ax) +h(x)
1 + +
4.3. Phng trnh vi hm s tun hon, phn tun hon nhn tnh 111
(2.2.41) f(a
2
x)
h(a
2
x)
1 + +
+
_
f(ax)
h(ax)
1 + +
_
+
+
_
f(x)
h(x)
1 + +
_
= 0
(4.21)
t g(x) = f(x)
h(x)
1 + +
ta c:
(2.2.45) g(a
2
x) + g(ax) +g(x) = 0.
Theo bi ton 1.2- chng 1- trng hp > 0 ta c:
g(x) =
1

1
_
[x[
log
|a|
|
2
|
h
1
(x) [x[
log
|a|
|
1
|
h
2
(x)

trong :
1
,
2
l nghim ca phng trnh:
2
+ + = 0.
h
1
(x), h
2
(x) l hai hm ty sao cho:
h
1
(ax) =
_
h
1
(x) nu
2
> 0
h
1
(x) nu
2
< 0
; h
2
(ax) =
_
h
2
(x) nu
1
> 0
h
2
(x) nu
1
< 0
T ta c:
f(x) =
h(x)
1 + +
+
1

1
_
[x[
log
|a|
|
2
|
h
1
(x) [x[
log
|a|
|
1
|
h
2
(x)

b) Trng hp = 0.
Phng trnh c trng (2.2.40) c nghim:
1
=
2
=

2
=

2
4
(2.2.41) f(a
2
x) +f(ax) +

2
4
f(x) = h(x))
f(a
2
x) +

2
f(ax) +

2
_
f(ax) +

2
f(x)
_
= h(x) (4.22)
t g(x) = f(ax) +

2
f(x) th:
(2.2.46) g(ax) +

2
g(x) = h(x) (4.23)
b1) Trng hp 1: = 2 : (2.2.47) g(ax) g(x) = h(x). Theo Bi ton
1.1

- chng 1 ta c: g(x) = k(x) +


ln[x[h(x)
ln [a[
trong k(x) l hm ty sao
4.3. Phng trnh vi hm s tun hon, phn tun hon nhn tnh 112
cho:
k(ax) = k(x). Khi :
f(ax) f(x) = k(x) +
ln[x[h(x)
ln[a[
f(ax) f(x)
ln[x[h(x)
ln [a[
= k(x)
f(ax) f(x)
1
ln[a[
_
(ln[ax[)
2
h(ax) (ln[x[)
2
h(x) (ln[a[)
2
h(x)
2 ln[a[

= k(x)
f(ax) f(x)
_
(ln[ax[)
2
h(ax)
2(ln[a[)
2

(ln [x[)
2
h(x)
2(ln[a[)
2

_
ln [ax[h(ax)
2 ln[a[

ln[x[h(x)
2 ln[a[
_
= k(x)
f(ax) f(x)
_
(ln[ax[)
2
ln [a[ ln[ax[
2(ln[a[)
2
h(ax)

(ln[x[)
2
ln [a[ ln[x[
2(ln[a[)
2

= k(x)
f(ax)
(ln[ax[)
2
ln[a[ ln[ax[
2(ln[a[)
2
h(ax)

_
f(x)
(ln[x[)
2
ln [a[ ln[x[
2(ln[a[)
2
h(x)

= k(x) (4.24)
t p(x) = f(x)
(ln[x[)
2
ln [a[ ln[x[
2(ln[a[)
2
h(x), khi :
(2.2.48) p(ax) p(x) = k(x).
Theo Bi ton 1.1

chng 1 ta c: p(x) = I(x) +


ln [x[k(x)
ln [a[
, trong I(x) l
hm ty sao cho: I(ax) = I(x).
f(x) = I(x) +
ln[x[k(x)
ln[a[
+
(ln [x[)
2
ln[a[ ln[x[
2(ln[a[)
2
h(x).
b2) Trng hp 2: ,= 2; < 0 :
(2.2.47)[ g(ax) +

2
g(x) = h(x).
Theo Bi ton 1.3

chng 1 ta c:
g(x) =
h(x)

2
+ 1
+[x[
log
|a|
|

2
|
q
1
(x) =
2h(x)
+ 2
+ [x[
log
|a|
(

2
)
q
1
(x)
4.3. Phng trnh vi hm s tun hon, phn tun hon nhn tnh 113
trong q
1
(x) l hm ty sao cho: q
1
(ax) = q(x) (v < 0). Ta c:
f(ax) +

2
f(x) =
2h(x)
+ 2
+ [x[
log
|a|
(

2
)
q
1
(x)
f(ax) +

2
f(x)
2h(x)
+ 2
= [x[
log
|a|
(

2
)
q
1
(x)
(4.25)
f(ax) +

2
f(x)
2
+ 2
_
h(ax)

2
+ 1
+ dfrac2
h(x)

2
+ 1
_
= [x[
log
|a|
(

2
)
q
1
(x)
(4.26)
f(ax) +

2
f(x)
_
4h(ax)
( + 2)
2
+

2
4h(x)
( + 2)
2
_
= [x[
log
|a|
(

2
)
q
1
(x)
f(ax)
4h(ax)
( + 2)
2
+

2
_
f(x)
4h(x)
( + 2)
2

= [x[
log
|a|
(

2
)
q
1
(x) (4.27)
t f
1
(x) = f(x)
4h(x)
( + 2)
2
ta c:
(2.2.49) f
1
(ax) +

2
f
1
(x) = [x[
log
|a|
(

2
)
q
1
(x).
p dng kt qu Bi ton 1.2-chng 1 (trng hp = 0, < 0, ,= 2) ta
c:
f
1
(x) = [x[
log
|a|
(

2
)
_
k
1
(x)
2 ln[x[q
1
(x)
ln[a[

trong : k
1
(x) l hm ty sao cho: k
1
(ax) = k
1
(x). Do :
f(x) =
4h(x)
( + 2)
2
+[x[
log
|a|
(

2
)
_
k
1
(x)
2 ln[x[q
1
(x)
ln [a[

b2) Trng hp 3: > 0, khi (2.2.47) g(ax) +



2
g(x) = h(x)
Theo Bi ton 1.3

- chng 1 ta c:
g(x) =
h(x)

2
+ 1
+ [x[
log
|a|
|

2
|
q
2
(x) =
2h(x)
+ 2
+[x[
log
|a|

2
q
2
(x)
trong q
2
(x) l hm ty sao cho: q
2
(ax) = q
2
(x) (v > 0). T c:
f(ax) +

2
f(x) =
2h(x)
+ 2
+[x[
log
|a|

2
q
2
(x) ()
4.3. Phng trnh vi hm s tun hon, phn tun hon nhn tnh 114
Bin i tng t nh trng hp 2 trn ta c:
() f(ax)
4h(ax)
( + 2)
2
+

2
_
f(x)
4h(x)
( + 2)
2

= [x[
log
|a|

2
q
2
(x) (4.28)
t f
2
(x) = f(x)
4h(x)
( + 2)
2
ta c:
(2.2.50) f
2
(ax) +

2
f
2
(x) = [x[
log
|a|

2
q
2
(x)
p dng kt qu Bi ton 1.2- chng 1 (trng hp = 0, > 0) ta c:
f
2
(x) = [x[
log
|a|

2
_
k
2
(x)
2 ln[
x
a
[q
2
(x)
ln[a[
_
trong k
2
(x) l hm ty sao cho: k
2
(ax) = k
2
(x). Vy:
f(x) =
4h(x)
( + 2)
2
+[x[
log
|a|

2
_
k
2
(x)
2 ln[
x
a
[q
2
(x)
ln[a[
_
c) Trng hp < 0 1 + + ,= 0. (Chng minh hon ton tng t nh
Bi ton 2.1.) ta c:
h(x) =
1 + +
1 + +
h(x) =
h(a
2
x) + h(ax) +h(x)
1 + +
(V: h(a
2
x) = h(ax) = h(x))
(2.2.39) f(a
2
x)
h(a
2
)
1 + +
+
_
f(ax)
h(ax)
1 + +

+
_
f(x)
h(x)
1 + +

= 0
(4.29)
t g(x) = f(x)
h(x)
1 + +
ta c:
(2.2.51) g(a
2
x) + g(ax) +g(x) = 0.
Phng trnh c trng:
2
+ + = 0, < 0 nn c hai nghim phc lin
hp
1
,
2
. Theo Bi ton (1.2)- chng 1 (trng hp < 0) ta c:
g(x) =
[
2
[
ln|x|
ln|a|
q
_
cos
ln[x[
ln[a[
n(x) + sin
ln[x[
ln [a[
m(x)
_
trong :

1
= p iq;
2
= p +iq [
1
[ = [
2
[ =
_
p
2
+q
2
4.3. Phng trnh vi hm s tun hon, phn tun hon nhn tnh 115
tan =
q
p
; arg
2
= arg
1
=
m(x), n(x) l hai hm ty tha mn: m(ax) = m(x); n(ax) = n(x). T ta
c:
f(x) =
h(x)
1 + +
+
[
2
[
ln|x|
ln|a|
q
_
cos
ln[x[
ln [a[
n(x) + sin
ln[x[
ln[a[
m(x)
_
Bi ton 4.3. Cho h(x) l hm phn tun hon nhn tnh chu k a trn R
(h(ax) = h(x)); a R0; 1; 1; , R. Xc nh tt c cc hm f :
R0 R tha mn iu kin:
f(a
2
x) +f(ax) +f(x) = h(x). (4.30)
Li gii:
Xt phng trnh c trng:

2
+ + = 0 (4.31)
c =
2
4.
a) Trng hp > 0 : Phng trnh (2.2.53) c hai nghim thc
1
,=
2
. p
dng nh l Viete ta c:
_

1
+
2
=

2
=
thay vo (2.2.52) ta c:
(2.2.52) f(a
2
x) (
1
+
2
)f(ax) +
1

2
f(x) = h(x)
f(a
2
x)
1
f(ax)
2
[f(ax)
1
f(x)] = h(x)
(4.32)
t g
1
(x) = f(ax)
1
f(x), khi :
(2.2.54) g
1
(ax)
2
g
1
(x) = h(x). (4.33)
a1) Trng hp 1:
1
,= 1 v
2
= 1.
(2.2.52) f(a
2
x) (
1
+ 1)f(ax) +
1
f(x) = h(x).
(2.2.55) g
1
(ax) g
1
(x) = h(x). Theo Bi ton 1.2

- chng 1 th: g
1
(x) =
k(x)
h(x)
2
. trong k(x) l hm ty sao cho: k(ax) = k(x).
Hay
f(ax)
1
f(x) = k(x)
h(x)
2
. (4.34)
4.3. Phng trnh vi hm s tun hon, phn tun hon nhn tnh 116
i vai tr
2
cho
1
v bin i tng t ta c:
g
2
(ax)
1
g
2
(x) = h(x) trong : g
2
(x) = f(ax) f(x).
V
1
,= 1 nn theo Bi ton 1.4

(ii)- chng 1, ta c:
g
2
(x) = [x[
log
|a|
|
1
|
q(x) +
h(x)

1
1
= [x[
log
|a|
|
1
|
q(x)
h(x)

1
+ 1
trong q(x) l hm ty sao cho:
q(ax) =
_
q(x) nu
1
> 0
q(x) nu
1
< 0
Khi :
f(ax) f(x) = [x[
log
|a|
|
1
|
q(x)
h(x)

1
+ 1
(4.35)
Tr (2.2.56) cho (2.2.57) ta c:
f(x) =
h(x)
2(
1
+ 1)
+
1
1
1
_
k(x) [x[
log
|a|
|
1
|
q(x)

a2) Trng hp 2:
1
= 1 v
2
= 1
(2.2.52) f(a
2
x) f(x) = h(x).
(2.2.55) g
1
(ax) g
1
(x) = h(x). Theo Bi ton 1.2

- chng 1 ta c:
g
1
(x) = k(x)
h(x)
2
, trong k(x) l hm ty tha mn: k(ax) = k(x).
Vy:
f(ax) +f(x) = k(x)
h(x)
2
(4.36)
i vai tr ca
1
v
2
v bin i tng t ta c:
g
2
(ax) + g
2
(x) = h(x) trong g
2
(x) = f(ax) f(x).
Theo Bi ton 1.4

(i)- chng 1 ta c: g
2
(x) = q(x)
ln [
x
a
[h(x)
ln[a[
vi q(x) l hm
ty sao cho: q(ax) = q(x).
Hay:
f(ax) f(x) = q(x)
ln[
x
a
[h(x)
ln [a[
(4.37)
Tr (2.2.58) cho (2.2.59) c: f(x) =
k(x) q(x)
2
+
ln[
x
a
3
[
4 ln[a[
h(x).
a3) Trng hp 3:
1
,= 1;
2
= 1.
(2.2.52) f(a
2
x) (
1
1)f(ax)
1
f(x) = h(x).
4.3. Phng trnh vi hm s tun hon, phn tun hon nhn tnh 117
(2.2.55) g
1
(ax) + g
1
(x) = h(x), theo Bi ton 1.4

(i)- chng 1 ta c:
g
1
(x) = q
1
(x)
ln[
x
a
[h(x)
ln[a[
, trong q
1
(x) l hm ty sao cho:q
1
(ax) = q
1
(x).
Vy:
f(ax) f(x) = q
1
(x)
ln[
x
a
[h(x)
ln[a[
(4.38)
i vai tr
1
cho
2
v bin i tng t ta c:
g
2
(ax)
1
g
2
(x) = h(x), trong g
2
(x) = f(ax) + f(x).
V
1
,= 1 nn theo Bi ton 1.4

(ii)- chng 1 c:
g
2
(x) = [x[
log
|a|
|
1
|
q
2
(x)
h(x)

1
+ 1
trong q
2
(x) l hm ty sao cho:
q
2
(ax) =
_
q
2
(x) nu
1
> 0
q
2
(x) nu
1
< 0
hay:
f(ax) +f(x) = [x[
log
|a|
|
1
|
q
2
(x)
h(x)

1
+ 1
. (4.39)
Tr (2.2.61) cho (2.2.60) ta c:
f(x) =
ln [
x

1
+1
a

1
+2
[
ln[a[(
1
+ 1)
2
h(x) +
1

1
+ 1
_
[x[
log
|a|
|
1
|
q
2
(x) q
1
(x)

.
a4) Trng hp 4:
1
,= 1,
2
,= 1 1 + + ,= 0.
Ta c: h(x) =
1 + +
1 + +
h(x) =
h(a
2
x) +h(ax) +h(x)
1 + +
(V: h(a
2
x) = h(ax) = h(x).)
(2.2.52) f(a
2
x)
h(a
2
x)
1 + +
+
_
f(ax)
h(ax)
1 + +

+
+
_
f(x)
h(x)
1 + +

= 0
(4.40)
t g(x) = f(x)
h(x)
1 + +
ta c: (2.2.62) g(a
2
x) + g(ax) +g(x) = 0.
Phng trnh c trng:
2
+ + = 0 ta c: > 0 nn c hai nghim thc
4.3. Phng trnh vi hm s tun hon, phn tun hon nhn tnh 118
phn bit
1
,
2
.
p dng kt qu Bi ton 1.2- chng 1 (trng hp > 0) ta c:
g(x) =
1

2

1
_
[x[
log
|a|
|
2
|
h
1
(x) [x[
log
|a|
|
1
|
h
2
(x)
_
trong h
1
(x), h
2
(x) l hai hm ty tha mn:
h
1
(ax) =
_
h
1
(x) nu
2
> 0
h
1
(x) nu
2
< 0
, h
2
(ax) =
_
h
2
(x) nu
1
> 0
h
2
(x) nu
1
< 0
Vy:
f(x) =
h(x)
1 + +
+
1

2

1
_
[x[
log
|a|
|
2
|
h
1
(x) [x[
log
|a|
|
1
|
h
2
(x)
_
b) Trng hp = 0
Phng trnh (2.2.53) c nghim:
1
=
2
=

2
=

2
4
.
(2.2.52) f(a
2
x) + f(ax) +

2
4
f(x) = h(x)
f(a
2
x) +

2
f(ax) +

2
[f(ax) +

2
f(x)] = h(x) (4.41)
t g(x) = f(ax) +

2
f(x) th:
(2.2.63) g(ax) +

2
g(x) = h(x). (4.42)
b1) Trng hp 1: = 2, khi : (2.2.64) g(ax) g(x) = h(x).
Theo Bi ton 1.2

- chng 1 ta c: g(x) = k(x)


h(x)
2
trong k(x) l hm
ty tha mn: k(ax) = k(x). Ta c:
f(ax) f(x) = k(x)
h(x)
2
f(ax) f(x) +
h(x)
2
= k(x)
f(ax) f(x) +
1
2
[
h(x)
2

h(ax)
2
] = k(x)
f(ax)
h(ax)
4
[f(x)
h(x)
4
] = k(x) (4.43)
t p(x) = f(x)
h(x)
4
, khi : (2.2.65) p(ax) p(x) = k(x). Theo Bi
ton 1.1

- chng 1 ta c:
4.3. Phng trnh vi hm s tun hon, phn tun hon nhn tnh 119
p(x) = q(x) +
ln [x[k(x)
ln[a[
trong q(x) l hm ty tha mn: q(ax) = q(x). Vy:
f(x) = q(x) +
h(x)
4
+
ln [x[k(x)
ln [a[
.
b2) Trng hp 2: = 2.
Khi : (2.2.64) g(ax) + g(x) = h(x). Theo Bi ton 1.4

(i)- chng 1 ta c:
g(x) = g
1
(x)
ln[
x
a
[h(x)
ln[a[
vi g
1
(x) l hm ty : g
1
(ax) = g
1
(x). Ta c:
f(ax) + f(x) = g
1
(x)
ln [
x
a
[h(x)
ln[a[
f(ax) + f(x) +
ln[
x
a
[h(x)
ln [a[
= g
1
(x)
f(ax) + f(x) +
ln[
x
a
[h(x)
ln [a[
h(x) = g
1
(x)
f(ax) + f(x) +
1
ln[a[
(ln[ax[)
2
h(ax) + (ln[x[)
2
h(x) + (ln[a[)
2
h(x)
2 ln[a[

ln[x[h(ax)
ln[a[

ln[
x
a
[h(x)
ln [a[

= g
1
(x)
f(ax) + f(x)
(ln[ax[)
2
h(ax)
2(ln[a[)
2

(ln [x[)
2
h(x)
2(ln[a[)
2

h(x)
2
+
ln [x[h(ax)
ln [a[
+
ln [
x
a
[h(x)
ln[a[
= g
1
(x).
f(ax) + f(x)
(ln[ax[)
2
h(ax)
2(ln[a[)
2

(ln [x[)
2
h(x)
2(ln[a[)
2
+
ln [x[h(ax)
ln[a[
+
ln[
x
a
[h(x)
ln[a[

1
2
_

ln[x[h(ax)
ln [a[

ln [
x
a
[h(x)
ln[a[

= g
1
(x)
f(ax) +
3 ln[a[ ln[x[ (ln [ax[)
2
2(ln[a[)
2
h(ax)+
+f(x) +
3 ln[a[ ln[
x
a
[ (ln[x[)
2
2(ln[a[)
2
h(x) = g
1
(x). (4.44)
t f
1
(x) = f(x) +
3 ln[a[ ln[
x
a
[ (ln[x[)
2
2(ln[a[)
2
h(x) ta c:
(2.2.66) f
1
(ax) +f
1
(x) = g
1
(x). Theo Bi ton 1.4

(ii)- chng 1 ta c:
4.3. Phng trnh vi hm s tun hon, phn tun hon nhn tnh 120
f
1
(x) = g
2
(x)
ln[
x
a
[g
1
(x)
ln[a[
, g
1
(x) tha mn: g
2
(ax) = g
2
(x). Vy:
f(x) = g
2
(x)
ln[
x
a
[g
1
(x)
ln[a[

3 ln[a[ ln[
x
a
[ (ln[x[)
2
2(ln[a[)
2
h(x).
b3) Trng hp 3: < 0, ,= 2

2
,= 1.
g(ax) +

2
g(x) = h(x). Theo Bi ton 1.4

(ii)- chng 1 ta c:
g(x) =
h(x)

2
1
+[x[
log
|a|
|

2
|
q
1
(x)
=
2h(x)
2
+ [x[
log
|a|
(

2
)
q
1
(x)
trong q
1
(x) l hm ty tha mn: q
1
(ax) = q
1
(x).(V: < 0). Nh vy:
f(ax) +

2
f(x) =
2h(x)
2
+[x[
log
|a|
(

2
)
q
1
(x)
f(ax) +

2
f(x)
2h(x)
2
= [x[
log
|a|
(

2
)
q
1
(x)
f(ax) +

2
f(x)
2
2
_
h(ax)

2
1
+

2
h(x)

2
1

= [x[
log
|a|
(

2
)
q
1
(x)
f(ax)
4h(x)
( 2)
2
+

2
_
f(x)
4h(x)
( 2)
2

= [x[
log
|a|
(

2
)
q
1
(x) (4.45)
t g
1
(x) = f(x)
4h(x)
( 2)
2
, (2.2.67) g
1
(ax) +

2
g
1
(x) = [x[
log
|a|
(

2
)
q
1
(x).
Theo Bi ton 1.2- chng 1 (trng hp = 0; < 0; ,= 2) ta c:
g
1
(x) = [x[
log
|a|
(

2
)
q
1
(x)
_
p
1
(x)
2 ln[x[q
1
(x)
ln[a[

trong p
1
(x) l hm ty : p
1
(ax) = p
1
(x). Vy:
f(x) =
4h(x)
( 2)
2
+[x[
log
|a|
(

2
)
_
p
1
(x)
2 ln[x[q
1
(x)
ln [a[

.
b4) Trng hp 4: > 0; ,= 2

2
,= 1 Khi :
g(ax) +

2
g(x) = h(x). Theo Bi ton 1.4

(ii)- chng 1:
g(x) =
h(x)

2
1
+ [x[
log
|a|
|

2
|
q
2
(x) =
2h(x)
2
+[x[
log
|a|

2
q
2
(x)
4.3. Phng trnh vi hm s tun hon, phn tun hon nhn tnh 121
vi q
2
(x) l hm ty tha mn: q
2
(ax) = q
2
(x) (V: > 0). Do :
f(ax) +

2
f(x) =
2h(x)
2
+ [x[
log
|a|
|

2
|
q
2
(x) ()
Bin i tng t nh trng hp 3 ta c:
() f(ax)
4h(ax)
( 2)
2
+

2
_
f(x)
4h(x)
( 2)
2

= [x[
log
|a|

2
q
2
(x) (4.46)
t g
2
(x) = f(x)
4h(x)
( 2)
2
ta c:
(2.2.68) g
2
(x) +

2
g
2
(x) = [x[
log
|a|

2
q
2
(x).
Theo bi ton 1.2- chng 1(trng hp = 0; > 0; ,= 2):
g
2
(x) = [x[
log
|a|

2
_
p
2
(x)
2 ln[
x
a
[q
2
(x)
ln[a[

,
trong p
2
(x) l hm ty tha mn: p
2
(ax) = p
2
(x). T ta c:
f(x) =
4h(x)
( 2)
2
+ [x[
log
|a|

2
_
p
2
(x)
2 ln[
x
a
[q
2
(x)
ln[a[

.
c) Trng hp: < 0 D dng chng minh c 1 + ,= 0. Ta c:
h(x) =
1 +
1 +
h(x) =
h(a
2
x) + h(ax) +h(x)
1 +
(V: h(a
2
x) = h(ax) = h(x))
(2.2.52) f(a
2
x)
h(a
2
)
1 +
+
_
f(ax)
h(ax)
1 +

+
+
_
f(x)
h(x)
1 +

= 0
(4.47)
t g(x) = f(x)
h(x)
1 +
ta c:
(2.2.69) g(a
2
x) + g(ax) +g(x) = 0.
Phng trnh c trng:
2
+ + = 0, < 0 nn c hai nghim phc lin
hp
1
,
2
. Theo Bi ton (1.2)- chng 1 (trng hp < 0) ta c:
g(x) =
[
o
[
ln|x|
ln|a|
q
_
cos
ln[x[
ln[a[
n(x) + sin
ln[x[
ln [a[
m(x)
_
4.3. Phng trnh vi hm s tun hon, phn tun hon nhn tnh 122
trong :

1
= p iq;
2
= p +iq [
o
[ = [
1
[ = [
2
[ =
_
p
2
+ q
2
tan =
q
p
; arg
2
= arg
1
=
m(x), n(x) l hai hm ty tha mn: m(ax) = m(x); n(ax) = n(x). T ta
c:
f(x) =
h(x)
1 +
+
[
o
[
ln|x|
ln|a|
q
_
cos
ln[x[
ln[a[
n(x) + sin
ln[x[
ln[a[
m(x)
_
.
Nhn xt 2.2 Sau khi th li hai bi ton trn ta nhn thy: Trong biu thc
nghim ca tt c cc trng hp, phn biu thc c cha h(x) l nghim ring
ca phng trnh: f(a
2
x) + f(ax) +f(x) = h(x).
Bi ton 4.4. Cho g(x) l hm tun hon nhn tnh chu k a, (g(ax) = g(x)); h(x)
l hm phn tun hon nhn tnh chu k a, (h(ax) = h(x)); a R0, 1, 1; ,
R. Xc nh tt c cc hm: f : R0 R tha mn iu kin:
f(a
2
x) +f(ax) +f(x) = g(x) + h(x). (4.48)
Li gii:
Xt phng trnh c trng:

2
+ + = 0, =
2
4. (4.49)
a) Trng hp > 0: Phng trnh c trng c hai nghim thc:
1
,=
2
.
a1) Trng hp 1:
1
,= 1;
2
= 1.
p dng nh l Viete:
_
1 +
1
=

1
=
(2.2.70) f(a
2
x) (
1
+ 1)f(ax) +
1
f(x) = g(x) + h(x) (4.50)
Xt phng trnh:
f
1
(a
2
x) (
1
+ 1)f
1
(ax) +
1
f
1
(x) = g(x). (4.51)
p dng kt qu ca Bi ton 2.4 (trng hp a1) v nhn xt 2.2 ta c biu
thc:
ln [x[(1
1
) ln [a[
ln [a[(1
1
)
2
g(x) l nghim ring ca (2.2.73). Thay vo ta c:
ln[a
2
x[(1
1
) ln[a[
ln [a[(1
1
)
2
g(a
2
x) (
1
+ 1)
ln[ax[(1
1
) ln [a[
ln[a[(1
1
)
2
g(ax)+
+
1
ln[x[(1
1
) ln[a[
ln[a[(1
1
)
2
g(x) = g(x)
(4.52)
4.3. Phng trnh vi hm s tun hon, phn tun hon nhn tnh 123
Thay (2.2.74) vo (2.2.72):
(2.2.72) f(a
2
x)
ln [a
2
x[(1
1
) ln [a[
ln [a[(1
1
)
2
g(a
2
x)
(
1
+ 1)
_
f(ax)
ln [ax[(1
1
) ln [a[
ln [a[(1
1
)
2
g(ax)

+
+
1
_
f(x)
ln [x[(1
1
) ln[a[
ln[a[(1
1
)
2
g(x)

= g(x)
(4.53)
t f
2
(x) = f(x)
ln [x[(1
1
) ln [a[
ln [a[(1
1
)
2
g(x) (i) ta c:
(2.2.75) f(a
2
x) (
1
+ 1)f
2
(ax) +
1
f
2
(x) = h(x).
p dng kt qu Bi ton 2.5 (trng hp a1) ta c:
f
2
(x) =
h(x)
2(
1
+ 1)
+
1
1
1
_
k(x) [x[
log
|a|
|
1
|
q(x)

(ii)
trong k(x); q(x) l hai hm ty tha mn:
k(ax) = k(x); q(ax) =
_
q(x) nu
1
> 0
q(x) nu
1
< 0
T (i) v (ii) ta c:
f(x) =
h(x)
2(
1
+ 1)
+
ln[x[(1
1
) ln[a[
ln[a[(1
1
)
2
g(x) +
1
1
1
_
k(x) [x[
log
|a|
|
1
|
q(x)

.
a2) Trng hp 2:
1
= 1;
2
= 1.
Bin i tng t nh trng hp a1) ta c:
f(x) =
ln[
x
a
3
[
4 ln[a[
h(x) +
ln[
x
2
a
[
lna
4
g(x) +
k(x) q(x)
2
trong k(x), q(x) l hai hm ty tha mn: k(ax) = k(x); q(ax) = q(x).
a3) Trng hp 3:
1
,= 1;
2
= 1.
Bin i tng t nh trng hp a1) ta c:
f(x) =
ln[
x

1
+1
a

1
+2
[h(x)
ln[a[(
1
+ 1)
2
+
g(x)
2(1
1
)
+
_
[x[
log
|a|
|
1
|
q
2
(x) q
1
(x)
_
4.3. Phng trnh vi hm s tun hon, phn tun hon nhn tnh 124
vi q
1
(x), q
2
(x) l hai hm ty tha mn:
q
1
(ax) = q
1
(x); q
2
(ax) =
_
q
2
(x) nu
1
> 0
q
2
(x) nu
1
< 0
a4) Trng hp 4:
1
,= 1;
2
,= 1.
Bin i tng t nh trng hp a1) ta c:
f(x) =
h(x)
1 +
+
g(x)
1 + +
+
1

2

1
_
[x[
log
|a|
|
2
|
h
1
(x) [x[
log
|a|
|
1
|
h
2
(x)

trong h
1
(x), h
2
(x) l hai hm ty tha mn:
h
1
(x) =
_
h
1
(x) nu
2
> 0
h
1
(x) nu
2
< 0
h
2
(x) =
_
h
2
(x) nu
1
> 0
h
2
(x) nu
1
< 0
b) Trng hp = 0
b1) Trng hp 1: = 2
Bin i tng t nh trng hp a1) ta c:
f(x) =
h(x)
4
+
(ln[x[)
2
ln [a[ ln[x[
2(ln[a[)
2
h(x) +q(x) +
ln [x[k(x)
ln [a[
Vi k(x), q(x) l hai hm ty tha mn: k(ax) = k(x), q(ax) = q(x).
b2) Trng hp 2: = 2
Bin i tng t nh trng hp a1) ta c:
f(x) =
3 ln[a[ ln[
x
a
[ (ln[x[)
2
2(ln[a[)
2
h(x) +
g(x)
4
+ g
2
(x)
ln [
x
a
[g
1
(x)
ln [a[
vi g
1
(x), g
2
(x) l hai hm ty tha mn: g
1
(ax) = g
1
(x); g
2
(ax) = g
2
(x).
b3) Trng hp 3: < 0; ,= 2
Bin i tng t nh trng hp a1) ta c:
4h(x)
( 2)
2
+
4g(x)
( + 2)
2
+ [x[
log
|a|
(

2
)
_
p
1
(x)
2 ln[x[q
1
(x)
ln[a[

,
trong : p
1
(x), q
1
(x) l hai hm ty : p
1
(ax) = p
1
(x); q
1
(ax) = q
1
(x)
b4) Trng hp 4: > 0; ,= 2
Bin i tng t nh trng hp a1) ta c:
4h(x)
( 2)
2
+
4g(x)
( + 2)
2
+ [x[
log
|a|

2
_
p
2
(x)
2 ln[
x
a
[q
2
(x)
ln[a[

4.3. Phng trnh vi hm s tun hon, phn tun hon nhn tnh 125
trong : p
2
(x), q
2
(x) l hai hm ty : p
2
(ax) = p
2
(x); q
2
(ax) = q
2
(x)
c) Trng hp < 0 Bin i tng t nh trng hp a1) ta c:
f(x) =
h(x)
1 +
+
g(x)
1 + +
+
[
o
[
ln|x|
ln|a|
q
_
cos
ln[x[
ln [a[
n(x) + sin
ln[x[
ln[a[
m(x)
_
trong :

1
= p iq;
2
= p + iq l nghim phng trnh:
2
+ + = 0.
[
o
[ = [
1
[ = [
2
[ =
_
p
2
+ q
2
tan =
q
p
; arg
2
= arg
1
= ; m(x), n(x) l hai hm ty tha mn: m(ax) =
m(x); n(ax) = n(x).
4.3.3 Mt s v d p dng
V d 4.3. Cho g(x) l hm tun hon nhn tnh chu k 3, (g(3x) = g(x)); h(x)
l hm phn tun hon nhn tnh chu k 3, (h(3x) = h(x)). Xc nh tt c cc
hm f : R

R sao cho:
f(9x) 7f(3x) + 10f(x) = 5g(x) + 21h(x).
Li gii:
Xt phng trnh c trng:
2
7 + 10 = 0
1
= 2;
2
= 5.
p dng Bi ton 2.6 (trng hp a4) ta c:
f(x) =
h(x)
18
+
g(x)
4
+
1
3
_
[x[
log
3
5
h
1
x [x[
log
3
2
h
2
x

trong h
1
, h
2
l hai hm ty : h
1
(3x) = h
1
(x); h
2
(3x) = h
2
(x)
V d 4.4. Cho g(x) l hm tun hon nhn tnh chu k
1
3
, (g(
1
3
x) =
g(x)); h(x) l hm phn tun hon nhn tnh chu k
1
3
, (h(
1
3
x) = h(x)). Xc
nh tt c cc hm: f : R0 R sao cho:
f(
1
9
x) + 2f(
1
3
x) +f(x) = 4g(x) 7h(x) + 13.
Li gii:
t g
1
(x) = 4g(x) + 13 g
1
(
1
3
x) = g
1
(x).
Xt phng trnh c trng:
2
+ 2 + 1 = 0, = 0
1
=
2
= 1 hn na
= 2, p dng bi ton 2.6 trng hp b2) ta c:
f(x) =
4g(x) + 13
4

ln 9 ln[3x[ (ln[x[)
2
2(ln3)
2
7h(x) +
ln[3x[g
1
(x)
ln3
+g
2
(x),
g
1
(x), g
2
(x) l cc hm ty tha mn: g
1
(
1
3
x) = g
1
(x); g
2
(
1
3
x) = g
2
(x).
4.3. Phng trnh vi hm s tun hon, phn tun hon nhn tnh 126
V d 4.5. Cho hm g(x) l hm tun hon nhn tnh chu k e, (g(ex) =
g(x)); h(x) l hm phn tun hon nhn tnh chu k e, (h(ex) = h(x)). Xc
nh tt c cc hm: f : R0 R sao cho:
f(e
2
x) 2

3f(ex) + 4f(x) = h(x) 3g(x).


Li gii:
Xt phng trnh c trng:
2
2

3+4 = 0, = 1 < 0 phng trnh c cc


nghim:

1
=

3 i;
2
=

3 +i; r = [
1
[ = [
2
[ = 2; q = 1; cos =

3
2
=

6
.
p dng Bi ton 2.6 (phn c) ) ta c:
f(x) =
h(x)
5 + 2

3

3g(x)
5 2

3
+ 2
ln|x|
_
cos
ln[x[
6
n(x) + sin
ln[x[
6
m(x)

,
trong m(x), n(x) l hai hm tha mn: m(ex) = m(x); n(ex) = n(x).
Ti liu tham kho
[1] Nguyn Thy Thanh (1990), L thuyt hm bin phc mt bin, NXB i
hc v Trung hc Chuyn nghip.
[2] Nguyn Vn Mu (2004, 2006), a thc i s v phn thc hu t, NXB
Gio dc.
[3] Nguyn Vn Mu (2003), Phng trnh hm, NXB Gio dc.
[4] L nh Thnh v cc tc gi khc, Phng trnh sai phn v mt s ng
dng, NXB Gio dc.
[5] Nguyn Trng Tun, Bi ton hm s qua cc k thi Olimpic, NXB Gio
dc.
[6] Kuczma Marek (1968), Functional equations in a single variable, PWN- Pol-
ish scientific publishers.
127
Chng 5
Dy s sinh bi hm s
5.1 Hm s chuyn i php tnh s hc v i s
Trong mc ny, ta kho st mt s tnh cht c bn ca mt s dng hm s
thng qua cc h thc hm n gin. Ta cng kho st mt s dng hm bo ton
v chuyn i cc tnh cht c bn ca php tnh i s nh giao hon, phn b
v kt hp.
Bi ton 1. Xc nh cc hm s f(x) xc nh v lin tc trn R tho mn
iu kin
f(x +y) = f(x) + f(y) + f(x)f(y), x, y R. (1)
Gii.
t f(x) = g(x) 1, ta thu c
g(x +y) 1 = g(x) 1 +g(y) 1 + [g(x) 1][g(y) 1], x, y R
hay
g(x +y) = g(x)g(y), x, y R. (2)
Do f(x) lin tc trn R nn g(x) cng l hm lin tc trn R. Suy ra (2) c
nghim g(x) = e
ax
, a R v (1) c nghim
f(x) = e
ax
1, a R.
Bi ton 2. Cho hm s F(u, v) (u, v R). Gi s phng trnh hm:
f(x +y) = F[f(x), f(y)], x, y R (3)
c nghim f(x) xc nh v lin tc trn R. Chng minh rng F(u, v) l hm i
xng (F(u, v) = F(v, u) v c tnh kt hp
F[F(u, v), w] = F[u, F(v, w)], u, v, w f. (4)
128
5.1. Hm s chuyn i php tnh s hc v i s 129
Gii.
Nhn xt rng tnh i xng ca F(u, v) c suy trc tip t (3). Mt khc,
theo (3), ta c
f(x +y +z) = f[(x +y) + z] = FF[f(x), f(y)], f(z), x, y, z R (5)
v
f(x +y +z) = f[x + (y + z)] = f[(y + z) +x] = FF[f(y), f(z)], f(x)
= Ff(x), F[f(y), f(z)], x, y, z R. (6)
T (5) v (6) suy ra (4):
F[F(u, v), w] = F[u, F(v, w)], u, v, w f.
Bi ton 3. Gi s phng trnh hm:
f(x +y) = F[f(x), f(y)], x, y R
vi hm s F(u, v) (u, v R) l mt a thc (khc hng), c nghim f(x) xc
nh v lin tc (khc hng) trn R. Chng minh rng F(u, v) c dng
F(u, v) = auv + bu +bv + c. (7)
Gii.
Gi s F(u, v) l a thc bc m theo u v bc n theo v. Khi , do F(u, v)
i xng nn m = n. Theo (4) th
F[F(u, v), w] = F[u, F(v, w)], u, v, w f
nn v tri l mt a thc bc n theo w cn v phi l a thc bc n
2
theo w.
Suy ra n
2
= n hay n = 1. Vy F(u, v) c dng
F(u, v) = auv +b
1
u +b
2
v +c.
Do F(u, v) l a thc i xng nn b
1
= b
2
v
F(u, v) = auv + bu +bv + c.
Nhn xt rng, vi F(u, v) = auv +bu +bv +c v F(u, v) tho mn iu kin
(4) th
ac = b
2
b.
Vy vi a ,= 0 th
ac = b
2
b c =
b
2
b
a
, a ,= 0. (8)
5.1. Hm s chuyn i php tnh s hc v i s 130
By gi, ta chuyn sang xt cc dng c bit ca (7).
Bi ton 4. Cho a thc F(u, v) = bu+bv +c, b ,= 0. Xc nh cc hm s f(x)
xc nh v lin tc trn R tho mn iu kin
f(x +y) = F[f(x), f(y)], x, y R
tc l
f(x +y) = bf(x) +bf(y) + c, x, y R. (9)
Gii.
Nhn xt rng, nu b ,= 1 th t (9) vi y = 0, ta c ngay f(x) = const. Khi
b =
1
2
v c = 0 th mi hm hng u tho mn (8). Khi b =
1
2
v c ,= 0 th (9) v
nghim. Cc trng hp khc (b ,= 1, b ,=
1
2
th nghim ca (9) l f(x) =
c
1 2b
.
Xt trng hp b = 1. Khi (9) c dng
f(x +y) = f(x) + f(y) +c, x, y R
v phng trnh hm ny c nghim f(x) = x c.
Bi ton 5. Cho a thc F(u, v) = auv +bu +bv +
b
2
b
a
, a ,= 0. Xc nh cc
hm s f(x) xc nh v lin tc trn R tho mn iu kin
f(x +y) = F[f(x), f(y)], x, y R
tc l
f(x + y) = af(x)f(y) + bf(x) +bf(y) +
b
2
b
a
, x, y R. (10)
Gii.
Nhn xt rng, nu t
f(x) =
h(x) b
a
th th t (10) ta nhn c
h(x +y) = h(x)h(y), x, y R
v phng trnh hm ny c nghim h(x) = e
x
. Suy ra nghim ca (10) c dng
f(x) =
e
x
b
a
.
Bi ton 6. Gi s f(x) l nghim ca phng trnh hm:
f(ax +by +c) = Af(x) +Bf(y) + C (abAB ,= 0), x, y R (11)
5.1. Hm s chuyn i php tnh s hc v i s 131
Chng minh rng hm s g(x) = f(x) f(0) tho mn phng trnh Cauchy
g(x +y) = g(x) + g(y), x, y R.
Gii.
Ln lt t x =
u
a
, y =
v c
b
; x =
u
a
, y =
c
b
; x = 0, y =
v c
b
; x = 0,
y =
c
b
vo (11), ta thu c cc ng thc
f(u +v) = Af
_
u
a
_
+Bf
_

v c
b
_
+C,
f(u) = Af
_
u
a
_
+Bf
_

c
b
_
+C,
f(v) = Af(0) +Bf
_
v c
b
_
+ C,
f(0) = Af(0) + Bf
_

c
b
_
+ C.
Suy ra
f(u +v) = f(u) +f(v) f(0).
T y suy ra iu phi chng minh.
Bi ton 7. Gi s hm s f(x) lin tc trn R l nghim ca phng trnh
hm:
f(ax + by + c) = Af(x) + Bf(y) + C (abAB ,= 0), x, y R. (11)
Chng minh rng khi A = a, B = b.
Gii.
Tht vy, nghim ca
g(x + y) = g(x) +g(y), x, y R
trong lp cc hm lin tc l hm tuyn tnh g(x) = x. Do vy, nghim f(x) c
dng f(x) = x +. Th vo (11), ta thu c A = a, B = b v
c C = (a + b 1). (12)
Bi ton 8. Gii v bin lun phng trnh hm sau trong lp cc hm s f(x)
lin tc trn R:
f(ax + by + c) = Af(x) + Bf(y) + C (abAB ,= 0), x, y R. (11)
Gii.
5.1. Hm s chuyn i php tnh s hc v i s 132
Theo Bi ton 7, th iu kin cn phng trnh hm (11) c nghim l
a = A, b = B.
Gi s iu kin ny c tho mn. Theo (12), ta chia cc trng hp ring
kho st.
Xt cc trng hp sau:
Trng hp b + a = 1, c = 0.
Khi , (11) c dng
f(ax + (1 a)y) = af(x) + (1 a)f(y) (abAB ,= 0), x, y R. (13)
Ta thu c (13) thuc lp hm chuyn tip cc i lng trung bnh cng. V
vy (13) c nghim f(x) = x + , , R.
Trng hp b + a = 1, c ,= 0.
Khi , (11) c dng
f(ax + (1 a)y +c) = af(x) + (1 a)f(y) + C (abAB ,= 0), x, y R. (13)
t f(x) =
C
c
x + h(x). Ta thu c (13) di dng
h(ax + (1 a)y + c) = ah(x) + (1 a)h(y), x, y R. (14)
D kim tra, phng trnh (14) ch c nghim hng tu (xem (12)) v v vy,
(13) c nghim f(x) =
C
c
x + , R.
Trng hp b + a ,= 1. Theo Bi ton 6 th nghim ca (13) c dng f(x) =
x +. Theo (12) th c C = (a+b 1). Vy nu cho R gi tr tu th
=
c C
a + b 1
.
Ch
Nu khng i hi nghim ca (11) l hm s lin tc trn R th cc ng
thc a = A, b = B v (12) c th khng tho mn. Tuy nhin, ta vn c cc tnh
cht i s sau y.
Bi ton 9. Gi s phng trnh hm
f(ax + y) = Af(x) +f(y) (aA ,= 0), x, y R (15)
c nghim khc hng. Chng minh rng nu a (hoc A) l s i s vi a thc
ti tiu P
a
(t)(tng ng P
A
(t)) th A (tng ng a) l s i s v
P
a
(t) P
A
(t). (16)
5.1. Hm s chuyn i php tnh s hc v i s 133
Gii.
Ta thy f(0) = 0 nn f(ax) = af(x) v bng quy np ton hc, d dng chng
minh
f(a
k
x) = A
k
f(x), k N. (17)
Gi s
P
a
(t) = t
n
+
n1

i=0
r
i
t
i
, r
0
, . . . , r
n1
Q.
Khi , theo (17) th
f
_
(a
n
+
n1

i=0
r
i
a
)x
_
= f(a
n
x) +
n1

i=0
r
i
f(a
i
x)
_
A
n
+
n1

i=0
r
i
A
_
f(x).
V f(x) khc hng nn
A
n
+
n1

i=0
r
i
A
= 0 (18)
v v vy A l s i s. Suy ra P
a
(t) l c ca P
A
(t) v do P
A
(t) l a thc
ti tiu nn c (16).
Ngc li, nu A l s i s tho mn (18) th thc hin quy trnh ngc li,
ta thu c
a
n
+
n1

i=0
r
i
a
= 0 (19)
v t suy ra (16).
Bi ton 10. Gi s phng trnh hm
f(ax +y) = Af(x) + f(y) (aA ,= 0, a Q), x, y R (20)
c nghim khc hng. Chng minh rng khi a = A.
Gii.
Tht vy, theo Bi ton 9 th P
a
(t) l a thc bc nht v v vy P
A
(t) cng
l a thc bc nht (vi h s bc cao nht u bng 1) nn a = A.
Bi ton 11. Gii phng trnh hm sau trong lp cc hm s f(x) lin tc trn
R:
f(x + y) = a
xy
f(x)f(y) (a > 0), x, y R. (21)
Gii.
5.1. Hm s chuyn i php tnh s hc v i s 134
D thy f(1) 0. Nu f(1) = 0 th t (21) ta c ngay f(x) 0. Xt trng
hp f(1) > 0. Bng quy np, d dng kim chng h thc
f(nx) = a
(n
2
n)x
2
2
[f(x)]
n
, n N

.
Vy vi x = 1 th
f(n) = a
n
2
n
2
[f(1)]
n
, n N

.
Vi x =
m
n
, ta thu c
f(m) = a
(n
2
n)(
m
n
)
2
2
_
f(
m
n
)
_
n
, m, n N

.
v
f(m) = a
m
2
m
2
[f(1)]
m
, m N

.
Suy ra
f
_
m
n
_
= a
1
2
(
m
n
)
2
_
a

1
2
f(1)
_m
n
. (22)
Do f(1) > 0 nn c th vit
c =
1
2
+ log
a
f(1).
T (21) suy ra
f(x) = a
1
2
x
2
+cx
, x Q
+
. (23)
Do f(x) lin tc nn (16) tho mn vi mi x R
+
. Vi x < 0, ta t x = y
v do f(0) = 1 nn t gi thit (21) ta nhn c
1 = a
x
2
f(x)a
(x
2
/2)cx
,
hay
f(x) = a
1
2
x
2
+cx
, x R.
Nhn xt.
Bng cch t
f(x) = a
x
2
/2
g(x)
ta a (15) v dng quen bit
g(x +y) = g(x)g(y), x R.
Bi ton 12. Xc nh cc hm s f xc nh v lin tc trn R tho mn iu
kin
f(x +y) +f(z) = f(x) +f(y + z), x, y, z R. (1)
5.2. V cc dy s xc nh bi dy cc phng trnh 135
Gii.
t f(0) = a th vi z = 0 trong (1) ta thu c
f(x + y) + a = f(x) + f(y), x, y R. (2)
t f(x) = g(x) + a. T (2) ta nhn c
g(x +y) = g(x) + g(y), x, y R. (3)
Phng trnh (3) c nghim g(x) = x, R.
Suy ra phng trnh (1) c nghim
f(x) = x + , , R.
Th li, ta thy hm f(x) = x + tho mn iu kin bi ra.
Bi ton 13. Xc nh cc hm s f xc nh v lin tc trn R tho mn iu
kin
f(x + y)f(z) = f(x)[f(y) + f(z)], x, y, z R. (4)
Gii.
Thay y = z = 0 trong (4), ta thu c f(0)f(x) = 0. Vy f(0) = 0. Vi z = 0
th
f(x +y)f(0) = f(x)[f(y) + f(0)], x, y R
hay
f(x)f(y) = 0, x, y R.
Suy ra f(x) 0.
5.2 V cc dy s xc nh bi dy cc phng trnh
Trong ton hc, c rt nhiu trng hp ta khng xc nh c gi tr c th
i tng m chng ta ang xt (v d s, hm s) nhng vn c th thc hin
cc php ton trn cc i tng . V d ta c th khng bit gi tr cc nghim
ca mt phng trnh, nhng vn bit c tng ca chng:
V d 5.1. Tm tng cc nghim ca phng trnh cos
5
x5 cos
3
x+3 cos x1 = 0
trn on [0, 2].
i khi ta cn tnh tch phn ca mt hm m ta khng c biu thc tng
minh:
V d 5.2. Chng minh rng vi mi t 0, phng trnh x
3
+tx 8 = 0 lun
c 1 nghim dng duy nht, k hiu l x(t). Tnh
_
7
0
[x(t)]
2
dt.
5.2. V cc dy s xc nh bi dy cc phng trnh 136
Trong bi vit nh ny, chng ta s cp n mt tnh hung cn bn khc,
l kho st nhng dy s xc nh bi dy cc phng trnh.
Bi ton 5.1. Cho dy cc hm s f
n
(x) xc nh bi cng thc tng minh
hoc truy hi tho mn iu kin: cc phng trnh f
n
(x) = 0 c nghim duy
nht x
n
D. Cn kho st cc tnh cht ca x
n
nh kho st s hi t, tm gii
hn...
Chng ta bt u t mt bi ton thi tuyn sinh vo khoa Ton trng i
hc c lp Matxcva nm 2000.
Bi ton 5.2. K hiu x
n
l nghim ca phng trnh
1
x
+
1
x 1
+ +
1
x n
= 0
thuc khong (0, 1)
a) Chng minh dy x
n
hi t;
b) Hy tm gii hn .
Bnh lun: Dy x
n
c xc nh duy nht v hm s
f
n
(x) =
1
x
+
1
x 1
+ +
1
x n
lin tc v n iu trn (0, 1). Tuy nhin, ta khng th xc nh c gi tr c
th ca x
n
. Rt may mn, chng minh tnh hi t ca x
n
, ta khng cn n
iu . Ch cn chng minh tnh n iu v b chn l . Vi tnh b chn,
mi th u n v 0 < x
n
< 1. Vi tnh n iu, ta ch mt cht n mi lin
h gia f
n
(x) v f
n+1
(x) trong
f
n+1
(x) = f
n
(x) +f
n+1
(x) = f
n
(x) +
1
x n 1
.
y chnh l cha kho chng minh tnh n iu ca x
n
.
Li gii. R rng x
n
c xc nh mt cch duy nht, 0 < x
n
< 1. Ta c
f
n+1
(x
n
) = f
n
(x
n
) + 1/(x
n
n 1) = 1/(x
n
n 1) < 0,
trong khi f
n+1
(0
+
) > 0. Theo tnh cht ca hm lin tc, trn khong (0, x
n
)
c t nht mt nghim ca f
n+1
(x). Nghim chnh l x
n+1
. Nh th ta
chng minh c x
n+1
< x
n
, tc l dy s x
n
n iu gim. Do dy ny b
chn di bi 0 nn dy s cho c gii hn.
5.2. V cc dy s xc nh bi dy cc phng trnh 137
Ta s chng minh gii hn ni trn bng 0. chng minh iu ny, ta cn
n kt qu quen thuc sau:
1 +
1
2
+
1
3
+ +
1
n
> ln n
(C th chng minh d dng bng cch s dng nh gi ln
_
1 +
1
n
_
<
1
n
).
Tht vy, gi s limx
n
= a > 0. Khi , do dy s gim nn ta c x
n
a
vi mi n.
Do 1+
1
2
+
1
3
+ +
1
n
khi n nn tn ti N sao cho vi mi n N
ta c 1 +
1
2
+
1
3
+ +
1
n
>
1
a
.
Khi vi n N ta c
0 =
1
x
n
+
1
x
n
1
+ +
1
x
n
n
<
1
x
n
+
1
1
+
1
2
+ +
1
x
n
<
1
a
+
1
a
= 0,
mu thun. Vy ta phi c limx
n
= 0.
Bi ton 5.3. Cho n l mt s nguyn dng (n > 1). Chng minh rng phng
trnh x
n
= x+1 c mt nghim dng duy nht, k hiu l x
n
. Chng minh rng
x
n
dn v 1 khi n dn n v cng v tm
lim
n
n(x
n
) 1).
Li gii. R rng x
n
> 1. t f
n
(x) = x
n
x 1. Khi f
n+1
(1) = 1 < 0 v
f
n+1
(x
n
) = x
n
n+1
x
n
1 > x
n
n
x
n
1 = f
n
(x
n
) = 0.
T ta suy ra 1 < x
n+1
< x
n
. Suy ra dy x
n
c gii hn hu hn a. Ta
chng minh a = 1. Tht vy, gi s a > 1. Khi x
n
a vi mi n v ta tm
c n ln sao cho: x
n
n
a
n
> 3 v x
n
+ 1 < 3, mu thun vi f
n
(x
n
) = 0.
gii phn cui ca bi ton, ta t x
n
= 1 + y
n
vi limy
n
= 0. Thay vo
phng trnh f
n
(x
n
) = 0, ta c (1 +y
n
)
n
= 2 +y
n
. Ly logarith hai v, ta c
nln(1 +y
n
) = ln(2 +y
n
)
T suy ra
limnln(1 +y
n
) = ln 2
Nhng limln(1 +y
n
)/y
n
= 1 nn t y ta suy ra limny
n
= ln2, tc l
lim
n
n(x
n
1) = ln 2.
5.2. V cc dy s xc nh bi dy cc phng trnh 138
Bi ton 5.4 (VMO 2007). Cho s thc a > 2 v f
n
(x) = a
10
x
n+10
+x
n
+ +
x + 1.
a) Chng minh rng vi mi s nguyn dng n, phng trnh f
n
(x) = a lun
c ng mt nghim dng duy nht.
b) Gi nghim l x
n
, chng minh rng dy x
n
c gii hn hu hn khi
n dn n v cng.
Li gii. Kt qu ca cu a) l hin nhin v hm f
n
(x) tng trn (0, +). D
dng nhn thy 0 < x
n
< 1. Ta s chng minh dy x
n
tng, tc l x
n+1
> x
n
.
Tng t nh nhng li gii trn, ta xt
f
n+1
(x
n
) = a
10
x
n+11
n
+ x
n+1
n
+ x
n
n
+ +x + 1 = x
n
f
n
(x
n
) + 1 = ax
n
+ 1
V ta c f
n+1
(1) = a
10
+n +1 > a nn ta ch cn chng minh ax
n
+1 < a l
s suy ra x
n
< x
n+1
< 1. Nh vy, cn chng minh x
n
<
a 1
a
. Tht vy, nu
x
n

a 1
a
th
f
n
(x
n
) a
10
_
a 1
a
_
n+10
+
1
_
a 1
a
_
n+1
1
a 1
a
=
= (a 1)
10
_
a 1
a
_
n
+a (a 1)
_
a 1
a
_
n
> a
(do a 1 > 1). Vy dy s tng x
n
tng v b chn bi 1 nn hi t.
Nhn xt 5.1. Mt ln na mi lin h f
n+1
(x) = xf
n
(x) +1 li gip chng ta
tm c mi quan h gia x
n
v x
n+1
. T li gii trn, ta c th chng minh
c rng limx
n
=
a 1
a
. Tht vy, t c =
a 1
a
< 1, theo tnh ton trn th
f
n
(c) f
n
(x
n
) = kc
n
(vi k = (a 1)((a 1)
9
1) > 0)
Theo nh l Lagrange th
f
n
(c) f
n
(x
n
) = f

()(c x
n
) vi thuc (x
n
, c)
Nhng f

() = (n + 10)a
10

n+9
+n
n1
+ + 1 > 1 nn t y suy ra
kc
n
> c x
n
T ta c
c kc
n
< x
n
< c
V c ngha lm limx
n
= c.
5.2. V cc dy s xc nh bi dy cc phng trnh 139
Bi ton 4. (VMO 2002) Cho n l mt s nguyn dng. Chng minh rng
phng trnh
1
x 1
+
1
4x 1
+ +
1
n
2
x 1
=
1
2
c mt nghim duy nht x
n
> 1. Chng minh rng khi n dn n v cng, x
n
dn n 4.
Bnh lun: Vic chng minh phng trnh c nghim duy nht x
n
> 1 l
hin nhin. Mi lin h f
n+1
(x) = f
n
(x) +
1
((n + 1)
2
x 1)
cho thy x
n
l dy
s tng ( y f
n
(x) =
1
x 1
+
1
4x 1
+ +
1
n
2
x 1

1
2
). bi cho sn
gii hn ca x
n
l 4 lm cho bi ton tr nn d hn nhiu. Tng t nh
cch chng minh limx
n
= c nhn xt trn, ta s dng nh l Lagrange
nh gi khong cch gia x
n
v 4. lm iu ny, ta cn tnh f
n
(4), vi
f
n
(x) =
1
x 1
+
1
4x 1
+ +
1
n
2
x 1

1
2
. Rt may mn, bi tnh f
n
(4) ny
lin quan n 1 dng tng quen thuc.
Li gii: t f
n
(x) nh trn v gi x
n
l nghim > 1 duy nht ca phng trnh
f
n
(x) = 0. Ta c
f
n
(4) =
1
4 1
+
1
16 1
+ +
1
4n
2
1

1
2
=
1
1.3
+
1
3.5
+ +
1
(2n 1)(2n + 1)

1
2
=
1
2
_
1
1

1
3
+
1
3

1
5
+ +
1
2n 1

1
2n
_

1
2
=
1
4n
p dng nh l Lagrange, ta c
1
4n
= [f
n
(x
n
) f(4)[ = [f

(c)[[x
n
4[
vi c thuc (x
n
, 4)
Nhng do
[f

(c)[ =
1
(c 1)
2
+
1
(4c 1)
2
+ >
1
9
Nn t y [x
n
4[ <
9
4n
, suy ra limx
n
= 4.
Trong v d trn (v trong phn nhn xt bi ton 3) chng ta s dng
nh l Lagrange nh gi hiu s gia x
n
v gi tr gii hn. v d cui
cng ca bi vit ny, ta tip tc nu ra ng dng dng nh l ny trong mt
tnh hung phc tp hn.
5.2. V cc dy s xc nh bi dy cc phng trnh 140
Bi ton 5. Cho n l mt s nguyn dng > 1. Chng minh rng phng
trnh x
n
= x
2
+x + 1 c mt nghim dng duy nht, k hiu l x
n
. Hy tm s
thc a sao cho gii hn lim
n
n
a
(x
n
x
n+1
) tn ti, hu hn v khc 0.
Bnh lun. D thy gi tr a, nu tn ti, l duy nht. Tng t nh bi
ton 2, c th chng minh c rng x
n
1 +
ln(3)
n
. T c d on l a = 2.
nh l Lagrange s gip chng ta nh gi hiu x
n
x
n+1
v chng minh d
on ny.
Li gii. t
P
n
(x) = x
n
x
2
x 1.
Ta c
P
n+1
(x) = x
n+1
x
2
x 1 = x
n+1
x
n
+P
n
(x) = x
n
(x 1) +P
n
(x).
T
P
n+1
(x
n
) = x
n
n
(x
n
1) +P
n
(x
n
) = (x
2
n
+ x
n
+ 1)(x
n
1) = x
3
n
1.
p dng nh l Lagrange, ta c
(x
2
n
+x
n
+ 1)(x
n
1) = P
n+1
(x
n
) P
n+1
(x
n+1
) = (x
n
x
n+1
)P

n+1
(c)
vi c thuc (x
n+1
, x
n
), P

n+1
(x) = (n + 1)x
n
2x 1.
T
(n + 1)
_
x
n+1
+ 1 +
1
x
n+1
_
2x
n+1
1 = P

n+1
(x
n+1
) < P

n+1
(c) <
< P

n+1
(x
n
) = (n + 1)(x
2
n
+x
n
+ 1) 2x
n
1.
T y, vi lu limx
n
= 1, ta suy ra
lim
n
P

n+1
(c)
n
= 3
Tip tc s dng limn(x
n
1) = 3, ta suy ra
lim
n
nP

n+1
(c)(x
n
x
n+1
) = lim
n
n(x
2
n
+x
n
+ 1)(x
n
1) = 3 ln(3)
lim
n
n
2
(x
n
x
n+1
)
P

n+1
(c)
n
= 3 ln(3)
lim
n
n
2
(x
n
x
n+1
) lim
n
P

n+1
(c)
n
= 3 ln(3)
lim
n
n
2
(x
n
x
n+1
)3 = 3 ln(3)
lim
n
n
2
(x
n
x
n+1
) = ln(3)
5.3. nh l v ba mnh tng ng 141
Vy vi c = 2 th gii hn cho tn ti, hu hn v khc 0. D thy vi
c > 2 th gii hn cho bng v cng v vi c < 2 th gii hn cho bng 0.
Vy c = 2 l p s duy nht ca bi ton.
Qua cc v d trn, chng ta thy cng c c bn kho st cc dy s cho
bi dy cc phng trnh l cc nh l c bn ca gii tch (v hm lin tc, hm
n iu, nh l v s hi t ca dy s n iu v b chn, nh l Lagrange)
v mi lin h mang tnh truy hi gia cc phng trnh. Hy vng rng vic phn
tch cc tnh hung 5 v d trn y s gip chng ta c mt cch nhn tng
qut cho cc bi ton dng ny.
5.3 nh l v ba mnh tng ng
nh l 5.1 (V ba mnh tng ng). . Cho dy s c
k
vi 0 < c
k
< 1,
k = 1, 2, 3, . . . . Xt cc dy s
X
n
=
n

i=1
(1 + c
i
); Y
n
=
n

i=1
(1 c
i
).
Khi ba khng nh sau l tng ng
(i) lim
n+
X
n
= +,
(ii) lim
n+
Y
n
= 0,
(iii) lim
n+
_
n

i=1
c
i
_
= +.
Chng minh.
Xt khng nh (i) (iii).
Gi s
n

i=1
c
i
< M, vi 0 < M < +. Khi
n

i=1
(1 + c
i
) <
_
1 +
1
n
n

i=1
c
i
_
n
<
_
1 +
M
n
_
n
< e
M
,
v l v rng lim
n+
X
n
= +. Do lim
n+
_
n

i=1
c
i
_
= +.
Xt khng nh (iii) (i). iu ny l hin nhin ng v rng
n

i=1
(1 + c
i
) >
n

i=1
c
i
.
5.4. Mt s bi ton v c lng tng v tch 142
Xt khng nh (ii)(iii). Nhn xt rng, ng vi b n s bt k a
1
, a
2
, . . . , a
n
vi 0 < a
i
< 1, th
n

i=1
a
i
> 1
n

i=1
(1 a
i
).
D dng kim tra tnh ng n ca bt ng thc ny bng qui np.
Do lim
n+
n

i=1
(1c
i
) = 0 nn ng vi mi m lun tn ti n sao cho
n

i=1
(1c
i
) <
1
2
. T ta c
n

i=1
c
i
> 1
n

i=1
(1 c
i
) >
1
2
.
Suy ra
+

i=1
c
i
= +.
Xt khng nh (i) (ii). Ta c
1 >
n

i=1
(1 c
2
i
) =
n

i=1
(1 + c
i
)
n

i=1
(1 c
i
).
Nhng v lim
n+
n

i=1
(1 +c
i
) = + nn lim
n+
n

i=1
(1 c
i
) = 0 (theo nguyn l kp).
Do lim
n+
Y
n
= 0.

By gi ta chuyn sang phn p dng nh l trn gii quyt mt s bi


ton.
5.4 Mt s bi ton v c lng tng v tch
Bi ton 5.5. Cho dy s thc tng u
n
c tnh cht
lim
n+
u
n
= +.
Chng minh rng lun tn ti k N sao cho
u
1
u
2
+
u
2
u
3
+ +
u
k
u
k+1
< k 2007
(ta gi s u
1
> 0).
5.4. Mt s bi ton v c lng tng v tch 143
Gii.
Ta s dng bin i tng ng sau
k
_
u
1
u
2
+
u
2
u
3
+ +
u
k
u
k+1
_
> 2007
k

i=1
_
1
u
i
u
i+1
_
> 2007.
Do u
n
l dy tng nn 0 < 1
u
i
u
i+1
< 1.
t c
i
= 1
u
i
u
i+1
. Suy ra 0 < c
i
< 1. Mt khc, ta c
n

i=1
(1 c
i
) =
n

i=1
u
i
u
i+1
=
u
1
u
n+1
tin dn ti 0 khi n +. Vy nn
n

i=1
c
i
= + (T 2) 3). Do k N
k

i=1
_
1
u
i
u
i+1
_
=
k

i=1
c
i
> 2007.
Bi ton 5.6. Cho dy s a
n
dng c tnh cht lim
n+
a
n
= +. Chng minh
rng lun tn ti k N sao cho
k

i=1
a
i
a
1
+a
2
+ +a
i
> 26
3
2007
.
Gii.
t c
i
=
a
i
a
1
+a
2
+ +a
i
V a
i
> 0 nn 0 < c
i
< 1 v
1 c
i
=
a
1
+a
2
+ +a
i1
a
1
+ a
2
+ + a
i
, vi i 2.
Suy ra
n

i=2
(1 c
i
) =
a
1
a
1
+ a
2
+ +a
n
tin dn ti 0 khi n +.
V a
i
> 0 v lim
n+
a
n
= +, nn
n

i=1
c
i
= +hay k N
n

i=1
c
i
> 26
3
2007
,
iu phi chng minh.
5.5. Bi tp 144
Bi ton 5.7. Xt dy tt c cc s nguyn t p
n
, 2 = p
1
< p
2
< p
3
< .
Chng minh rng
+

n=1
1
p
n
= +.
Gii.
Bi ton 5.8. Xt dy s a
n
xc nh bi cng thc
a
1
=
1
2
, a
n+1
=
2n 1
2n + 2
a
n
, n = 1, 2, . . . .
Chng minh rng
+

n=1
a
n
= 1.
Gii.
Bi ton 5.9. Cho dy s a
n
dng, tng v khng b chn. Chng minh rng
+

n=1
_
arccos
_
a
n
a
n+1
__
2
= +.
Gii.
5.5 Bi tp
Bi 5.1. Cho dy s a
n
dng, tng v khng b chn. Chng minh rng
+

n=1
arccos
_
a
n
a
n+1
_
= +.
Chng 6
Mt s lp hm chuyn i cc
cp s
Trong chng ny s m t mt s lp hm s chuyn i cc cp s.
6.1 Cp s cng, cp s nhn v cp s iu ho
Trong chng trnh ton bc trung hc, cc bi ton v cp s cng v
cp s nhn c cp kh y . c bit, trong cc sch gio khoa v
sch bi dng, nng cao c mt s lng rt ln cc bi ton v tnh tng, xc
nh s hng tng qut, iu kin mt dy lp thnh mt cp s,... . V vy,
trong mc ny chng ta ch yu tp trung kho st mt s c trng c lin quan
trc tip n dy s l cc cp s cng, cp s nhn v mt vi dng cp s m
rng.
nh ngha 6.1. (i) Dy s u
n
(hoc (u
n
)) tho mn iu kin
u
1
u
0
= u
2
u
1
= = u
n+1
u
n
=
c gi l mt cp s cng.
(ii) Khi dy s u
n
lp thnh mt cp s cng th hiu d = u
1
u
0
c gi
l cng sai ca cp s cho.
Nhn xt rng khi cho mt dy hu hn s u
0
, u
1
, . . . , u
s
tho mn iu
kin
u
1
u
0
= u
2
u
1
= = u
s
u
s1
th ta cng ni rng dy hu hn cho lp thnh mt cp s cng vi cng sai
d = u
1
u
0
.
145
6.2. Dy s tun hon 146
nh ngha 6.2. (i) Dy s u
n
(u
n
,= 0 vi mi n N) tho mn iu kin
u
1
u
0
=
u
2
u
1
= =
u
n+1
u
n
=
c gi l mt cp s nhn.
(ii) Khi dy s u
n
lp thnh mt cp s nhn th thng q =
u
1
u
0
c gi
l cng bi ca cp s cho.
Nhn xt rng khi cho mt dy hu hn s khc 0 : u
0
, u
1
, . . . , u
s
tho mn
iu kin
u
1
u
0
=
u
2
u
1
= =
u
s
u
s1
th ta cng ni rng dy hu hn cho lp thnh mt cp s nhn vi cng bi
p =
u
1
u
0
.
Ta lun c mi lin h gia cp s cng v cp s nhn sau y.
Bi ton 6.1. (i) Nu dy s u
n
l mt cp s cng th dy s v
n
vi
v
n
= a
un
, n N, a > 0
s lp thnh mt cp s nhn.
(ii) Ngc li, nu dy s u
n
l mt cp s nhn vi cc s hng dng th
dy s v
n
vi
v
n
= log
a
u
n
, n, N, 0 < a ,= 1
s lp thnh mt cp s cng.
Gii Chng minh c suy ra trc tip t nh ngha 6.1 v 6.2
nh ngha 6.3. Dy s u
n
(u
n
,= 0 vi mi n N) tho mn iu kin
u
n
=
2u
n1
u
n+1
u
n1
+u
n+1
, n N

c gi l cp s iu ho.
6.2 Dy s tun hon
Tng t nh i vi hm s thng thng, ta c th coi dy s x
n
nh
mt hm f(n) = x
n
xc nh trn tp N v nhn gi tr trong R. Ta ch quan
tm n hai loi dy tun hon c bn l tun hon cng tnh v tun hon nhn
tnh.
6.2. Dy s tun hon 147
nh ngha 6.4. Dy s u
n
c gi l mt dy tun hon (cng tnh) nu
tn ti s nguyn dng l sao cho
u
n+l
= u
n
, n N. (1)
S nguyn dng l nh nht dy u
n
tho mn (1) c gi l chu k c s
ca dy.
Trong thc hnh, chng minh mt dy cho l tun hon, khng nht
thit phi xc nh chu k c s ca n.
Nhn xt 6.1. Dy tun hon chu k 1 khi v ch khi dy l mt dy hng.
Tng t, ta cng c nh ngha v dy tun hon nhn tnh.
nh ngha 6.5. Dy s u
n
c gi l mt dy tun hon nhn tnh nu tn
ti s nguyn dng s (s > 1) sao cho
u
sn
= u
n
, n N. (2)
S nguyn dng s nh nht dy u
n
tho mn (2) c gi l chu k c s
ca dy.
Bi ton 6.2. Chng minh rng dy u
n
tun hon (cng tnh) chu k 2 khi
v ch khi dy c dng
u
n
=
1
2
[ + + ( )(1)
n+1
], , R. (3)
Gii Gi s u
0
= , u
1
= v u
n+2
= u
n
, n N. Khi ta thy ngay (bng
quy np ton hc) dy u
n
c dng (3). Ngc li, mi dy xc nh theo (3) l
mt dy tun hon chu k 2.
Bi ton 6.3. Chng minh rng dy u
n
tun hon nhn tnh chu k 2 khi v
ch khi dy c dng
u
n
=
_
tu vi nl,
u
2k+1
vi n = 2
m
(2k + 1), m N

, k N.
Gii Chng minh c suy trc tip t h thc truy hi.
Bi ton 6.4. Chng minh rng dy u
n
tun hon chu k 3 khi v ch khi dy
c dng
u
n
=
1
3
[+++(+2)] cos
2n
3
+

3
2
() sin
2n
3
, , , R. (4)
6.2. Dy s tun hon 148
Gii. Gi s u
0
= , u
1
= , u
2
= v u
n+3
= u
n
, n N. Khi , ta thy
ngay (bng quy np ton hc) dy u
n
c dng (4).
Ngc li, mi dy xc nh theo (4) l mt dy tun hon chu k 3 :
, , , , , , . . . .
Bi ton 6.5. Cho k Q Z. Chng minh rng dy s u
n
xc nh theo cng
thc
u
0
= 1, u
1
= 1, u
n+1
= ku
n
u
n1
, n N

khng l mt dy tun hon.


Gii. Khi [k[ > 2 th
[u
n+1
[ [k[[u
n
[ [u
n1
[ > 2[u
n
[ [u
n1
[.
Nu lun lun xy ra [u
n
[ < [u
n1
[ vi mi n N

th ta c ngay iu phi chng


minh. Nu xy ra [u
m
[ [u
m1
[ > 0 th suy ra
[u
m
[ < [u
m+1
[ <
v do dy u
n
khng l mt dy s tun hon.
Xt [k[ 2 vi k =
p
q
, (p, q) = 1, 2 q Z

, p Z. Bng quy np theo n ta


thu c
u
j
=
p
j
q
j1
, p
j
Z, (p
j
, q) = 1, j 1, . . . , n.
T suy ra
u
n+1
=
p
q
u
n
u
n1
=
p
n+1
q
n
,
trong
p
n+1
= pp
n
q
2
p
n1
Z
v (p
n+1
, q) = 1. Do q 2 nn u
n
,= u
m
khi n ,= m v dy u
n
khng l dy s
tun hon.
Bi ton 6.6. Xc nh cc gi tr ca k Q dy s u
n
xc nh theo cng
thc
u
0
= 1, u
1
= 1, u
n+1
= ku
n
u
n1
, n N

l mt dy s tun hon.
6.2. Dy s tun hon 149
Gii.
Theo kt qu ca Bi ton 4, khi [k[ > 2 v [k[ 2, k =
p
q
vi (p, q) = 1,
2 q Z

th dy u
n
khng l dy s tun hon.
Xt [k[ 2 v k Z.
Vi k = 2 th u
n
l mt cp s cng vi cng sai bng 2 nn hin nhin
dy u
n
khng l dy tun hon.
Vi k = 1 th u
n
l dy tun hon chu k 6 :
u
2
= 2, u
3
= 1, u
4
= 1, u
5
= 2, u
6
= 1, u
7
= 1, . . . .
Vi k = 0 th u
n
l dy tun hon chu k 4 :
u
0
= 1, u
1
= 1, u
2
= 1, u
3
= 1, u
4
= 1, u
5
= 1, . . . .
Vi k = 1 th u
n
l dy tun hon chu k 3 :
u
0
= 1, u
1
= 1, u
2
= 0, u
3
= 1, u
4
= 1, . . . .
Vi k = 2 th u
n
l dy tun hon chu k 2 :
u
0
= 1, u
1
= 1, u
2
= 1, u
3
= 1, u
4
= 1, . . . .

nh ngha 6.6. a) Dy s u
n
c gi l mt dy phn tun hon (cng
tnh) nu tn ti s nguyn dng l sao cho
u
n+l
= u
n
, n N. (5)
S nguyn dng l nh nht dy u
n
tho mn (5) c gi l chu k c s
ca dy.
b) Dy s v
n
c gi l mt dy phn tun hon nhn tnh nu tn ti s
nguyn dng s (s > 1) sao cho
v
sn
= v
n
, n N. (6)
S nguyn dng s (s > 1) nh nht dy v
n
tho mn (6) c gi l chu
k c s ca dy.
Nhn xt 6.2. a) Dy phn tun hon vi chu k l l mt dy tun hon chu
k 2l.
b) Dy phn tun hon nhn tnh chu k s l mt dy tun hon nhn tnh chu
k 2s.
6.2. Dy s tun hon 150
Bi ton 6.7. Chng minh rng mi dy u
n
phn tun hon chu k r u c
dng
u
n
=
1
2
(v
n
v
n+r
) vi v
n+2r
= v
n
. (7)
Gii. Gi s u
n+r
= u
n
, n N. Khi , ta thy ngay rng dy u
n
tun
hon chu k 2r v
u
n
=
1
2
(u
n
u
n+r
),
tc l c dng (7).
Ngc li, kim tra trc tip, ta thy mi dy xc nh theo (7) u l dy
phn tun hon chu k r.
Bi ton 6.8. Cho f(x) l mt a thc vi deg f = k 1, f(x) Z ng vi
mi x Z. K hiu r(k) = min2
s
[ s N

, 2
s
> k. Chng minh rng dy s
(1)
f(k)
(k = 1, 2, . . .) l dy tun hon vi chu k r(k).
Gii. Ta c k!f(x) Z[x]. Biu din f(x) di dng
f(x) = a
0
+a
1
_
x
1
_
+ + a
k
_
x
k
_
,
trong
_
x
k
_
=
x(x 1) (x k + 1)
k!
.
Ta cn chng minh f(x +r(k)) f(x) chia ht cho 2 vi mi x Z.
Nhn xt rng
M
i
=
_
x + 2
s
i
_

_
x
i
_
chia ht cho 2 vi mi i N

, 2
s
i, x Z. Tht vy, ta c
M
i
=
1
i!
_
(2
s
+x)(2
s
+x 1) . . . (2
s
+ x i + 1) x(x 1) . . . (x i + 1)

.
T s hin nhin chia ht cho 2
s
. Mt khc, s m ca 2 trong khai trin ca i!
l

j=1
_
i
2
j

<

j=1
i
2
j
= i 2
s
,
nn M
i
chia ht cho 2 vi mi i N

, i 2
s
, x Z. T suy ra
T
i
=
_
x + r(k)
i
_

_
x
i
_
6.2. Dy s tun hon 151
chia ht cho 2 vi mi i Z, i k, x Z. Do a
j
Z nn
f(x +r(k)) f(x) =
k

j=0
a
j
T
j
chia ht cho 2, iu phi chng minh.
Bi ton 6.9. Xc nh dy s u
n
tho mn iu kin
u
2n+1
= 3u
n
, n N. (8)
Gii. t n + 1 = m, m = 1, 2, . . .. Khi c th vit (8) di dng
u
2m1
= 3u
m1
, m N

hay
v
2m
= 3v
m
, m N

(9)
vi
v
m
= u
m1
, m N

. (10)
T (9) ta c v
0
= 0. t v
m
= m
log
2
3
y
m
, m N

. Khi (9) c dng


y
2m
= y
m
, m N

.
Vy y
m
l mt dy tun hon nhn tnh chu k 2. Khi theo Bi ton 2 ta c
y
n
=
_
tu vi nl,
y
2k+1
vi nc dng 2
m
(2k + 1), m N

, k N.
T suy ra
u
m
= v
m+1
= m
log
2
3
y
m+1
,
vi
y
n
=
_
tu vi nl,
y
2k+1
vi nc dng 2
m
(2k + 1), m N

, k N.

Bi ton 6.10. Xc nh dy u
n
tho mn iu kin
u
2n+1
= 3u
n
+ 4, n N. (11)
6.3. Hm s chuyn i cp s cng 152
Gii. t n + 1 = m, m = 1, 2, . . .. Khi c th vit (11) di dng
u
2m1
= 3u
m1
+ 4, m N

hay
v
2m
= 3v
m
+ 4, m N

(12)
vi v
m
= u
m1
.
t v
m
= 1 +x
m
. Khi (12) c dng
x
2m
= 3x
m
, m N

. (13)
t x
m
= m
log
2
3
y
m
, m N

. Khi (13) c dng


y
2m
= y
m
, m N

.
Vy y
m
l mt dy phn tun hon nhn tnh chu k 2.
Khi , theo Bi ton 2, ta c
y
n
=
_

_
tu vi nl,
y
2k+1
vi nc dng 2
2m+1
(2k + 1), m, k N,
y
2k+1
vi nc dng 2
2m
(2k + 1), m N

, k N.
T suy ra
u
m
= v
m+1
= 1 + (m+ 1)
log
2
3
y
m+1
,
vi
y
n
=
_

_
tu vi nl,
y
2k+1
vi nc dng 2
2m+1
(2k + 1), m, k N,
y
2k+1
vi nc dng 2
2m
(2k + 1), m N

, k N.
6.3 Hm s chuyn i cp s cng
Bi ton 6.11. Nu dy s u
n
l mt cp s cng th dy s v
n

vi v
n
= au
n
+b, n N s lp thnh mt cp s cng.
Gii. Gi s u
n
l cp s cng vi cng sai bng d.
Xt dy s v
n
vi v
n
= au
n
+ b, n N.
Ta c v
0
= au
0
+b, v
1
= au
1
+ b . . . v
n
= au
n
+b, v
n+1
= a(n + 1) + b.
Khi
v
1
v
0
= v
2
v
1
= v
3
v
2
= v
n+1
v
n
= ad
Vy dy v
n
l cp s nhn vi cng sai bng ad
Vn t ra l ta i tm tt c cc hm s c tnh cht chuyn cp s cng
thnh cp s cng. Xt b sau.
6.3. Hm s chuyn i cp s cng 153
B 6.1. Cho cp s cng a
n
v hm s f : R R
+
tho mn iu kin
f
_
x +y
2
_
=
f(x) + f(y)
2
, x, y > 0.
Khi dy f(a
n
) l mt cp s cng.
Chng minh. T gi thit, ta c cc h thc
a
1
a
0
= = a
n
a
n1
= a
n+1
a
n
= . . .
Suy ra
2a
n
= a
n1
+ a
n+1
, n N

Khi
f(a
n
) = f(
a
n1
+ a
n+1
2
) =
f(a
n1
) +f(a
n+1
)
2
.
T ta c f(a
n
) l mt cp s cng.
Bi ton 6.12. Tm hm s f(x) xc nh v lin tc trn R tho mn iu
kin:
f
_
x + y
2
_
=
f(x) + f(y)
2
, x, y R
Gii. t f(x) f(0) = g(x), ta c g(x) lin tc trn R, vi g(0) = 0 v
g
_
x + y
2
_
=
g(x) + g(y)
2
, x, y R
Ln lt cho y = 0 v x = 0, th
g(
x
2
) =
g(x)
2
v
g(
y
2
) =
g(y)
2
, x, y R.
Do
g
_
x + y
2
_
= g
_
x
2
_
+ g
_
y
2
_
, x, y R
Vy
g(x + y) = g(x) +g(y), x, y R
Vg(x) lin tc trnR,nn phng trnh trn l phng trnh Cauchy v do
g(x) = ax.Suy ra f(x) = ax + b, (a, b R).
6.4. Hm s chuyn i cp s cng vo cp s nhn 154
6.4 Hm s chuyn i cp s cng vo cp s nhn
Bi ton 6.13. Nu dy s u
n
l mt cp s cng th dy s v
n

vi v
n
= a
un
, n N, a > 0 s lp thnh mt cp s nhn.
Gii. Gi s u
n
l cp s cng vi cng sai bng d.
Xt dy s v
n
vi v
n
= a
un
, n N, a > 0.
Ta c v
0
= a
u
0
, v
1
= a
u
1
. . . v
n
= a
n
, v
n+1
= a
n+1
.
Khi
v
1
v
0
=
v
2
v
1
=
v
3
v
2
=
v
n+1
v
n
= a
d
Vy dyv
n
l cp s nhn vi cng bi bng a
d
B 6.2. Cho cp s cng a
n
v hm s f : R R
+
tho mn iu kin
f
_
x +y
2
_
=
_
f(x)f(y), x, y > 0.
Khi dy f(a
n
) l mt cp s nhn.
Chng minh. T gi thit, ta c cc h thc
a
1
a
0
= = a
n
a
n1
= a
n+1
a
n
= . . .
Suy ra
2a
n
= a
n1
+ a
n+1
, n N

Khi
f(a
n
) = f(
a
n1
+ a
n+1
2
) =
_
f(a
n1
)f(a
n+1
).
T ta c f(a
n
) l mt cp s nhn.
Nh vy ta c hai hm s trn chuyn cp s cng thnh cp s nhn, vn
t ra l ta i tm tt c cc hm s c tnh cht chuyn mt cp s cng bt k
thnh mt cp s nhn.Trc ht ta xt bi ton sau.
Bi ton 6.14. Tm hm f(x) xc nh v lin tc trn R tho mn iu kin
f(
x +y
2
) =
_
f(x)f(y), x, y R
Gii. Theo iu kin bi ton ta suy raf(x) 0, x R.
Nu tn ti x
0
f(x
0
) = 0 th
f(
x
0
+y
2
) =
_
f(x
0
)f(y) = 0, y R,
tc l f(x) 0
Xt trng hp f(x) > 0, x R. Khi ta c
6.5. Hm s chuyn i cp s nhn vo cp s cng 155
lnf(
x + y
2
) =
lnf(x) + ln f(y)
2
, x, y R
hay
g
_
x + y
2
_
=
g(x) + g(y)
2
, x, y R
trong g(x) = lnf(x). Theo kt qu Bi ton 4 th g(x) = ax + b.
Vy f(x) = e
ax+b
, a, b R tu .
6.5 Hm s chuyn i cp s nhn vo cp s cng
Bi ton 6.15. Nu dy s u
n
l mt cp s nhn vi cc s hng dng th
dy s v
n

vi v
n
= log
un
a
, n N, 0 < a ,= 1 s lp thnh mt cp s cng.
Gii. Gi s u
n
l cp s nhn vi cng bi bng q. Xt dy s v
n
viv
n
=
log
un
a
, n N, 0 < a ,= 1.
Ta c
v
0
= log
u
0
a
, v
1
= log
u
1
a
, v
2
= log
u
2
a
. . . , v
n
= log
un
a
Khi
v
1
v
0
= v
2
v
1
= v
3
v
2
= v
n
v
n1
= log
d
a
Vyv
n
l cp s cng vi cng sai bng log
d
a

B 6.3. Cho cp s nhn a
n
vi a
n
> 0n N v hm s f(x) tho mn
iu kin
f(

xy) =
f(x) +f(y
2
, x, y > 0.
Khi dy f(a
n
) l mt cp s cng.
Chng minh. T gi thit, ta c cc h thc
a
0
a
1
= =
a
n1
a
n
=
a
n
a
n+1
= . . .
Suy ra
a
2
n
= a
n1
a
n+1
, n N

Khi
f(a
n
) = f(

a
n1
a
n+1
) = f(
a
n1
+a
n+1
2
).
T ta c f(a
n
) l mt cp s cng.
Vn t ra ta i tm tt c cc hm s chuyn i mt cp s nhn bt k
thnh mt cp s cng. Trc ht ta xt bi ton sau.
6.6. Hm s chuyn i cp s nhn vo cp s iu ho 156
Bi ton 6.16. Tm hm s f(x) xc nh v lin tc trn R tho mn iu kin
f(

xy) =
f(x) + f(y)
2
, x, y R
+
Gii V x > 0, y > o nn c th t x = e
u
, y = e
v
v f(e
u
) = g(u).
Khi g(u) lin tc tn R v c dng
g
_
u + v
2
_
=
g(u) +g(v)
2
, u, v R.
Theo kt qu ca bi ton 8.52 thg(u) = au +b.
Vy ta c kt qu f(x) = a ln x +b, a, b R tu .
Theo b 6.3 trn ta c hm s f(x) = a lnx + b, a, b R chuyn i mi
cp s nhn thnh cp s cng.
6.6 Hm s chuyn i cp s nhn vo cp s iu
ho
Ta xt bi ton sau.
Bi ton 6.17. Cho cp s nhn a
n
vi a
n
> 0, n N v cho hm s
f : R
+
R
+
tho mn iu kin
f(

xy) =
2f(x)f(y)
f(x) + f(y)
, x, y R
+
Chng minh rng dy f(a
n
) l mt cp s iu ho.
Gii. T gi thit, ta c cc h thc
a
1
a
0
= =
a
n
a
n1
=
a
n+1
a
n
= . . .
Suy ra
a
2
n
= a
n1
a
n+1
, n N

Khi
f(a
n
) = f(

a
n1
a
n+1
) =
2f(a
n1
)f(a
n+1
)
f(a
n1
) + f(a
n+1
)
).
T ta c f(a
n
) l mt cp s iu ho.
By gi ta i tm tt c cc hm s c tnh cht chuyn mt cp s cng bt
k thnh cp s iu ho thng qua vic tm tt c cc hm s c tnh cht sau.
6.6. Hm s chuyn i cp s nhn vo cp s iu ho 157
Bi ton 6.18. Tm hm f(x) xc nh v lin tc trn R
+
tho mn iu kin
f(

xy) =
2f(x)f(y)
f(x) + f(y)
, x, y R
+
Gii. Ta c
f(

xy) =
2f(x)f(y)
f(x) +f(y)
, x, y R
+
Suy ra
f(

xy) =
2
1
f(x)
+
1
f(y)
, x, y R
+
Hay
1
f(

xy)
=
1
f(x)
+
1
f(y)
2
, x, y R
+
t g(x) =
1
f(x)
. Khi ta c
g(

xy) =
g(x) + g(y)
2
, x, y R
+
Theo kt qu ca Bi ton 6.16 th g(x) = a lnx + b. f(x) lin tc trong R
+
th g(x) ,= 0 vi mi x R
+
. iu tng ng vi a = 0, b ,= 0
Vy
f(x) b R0 tu .

Chng 7
Mt s lp hm chuyn i cc
cp s trong tp ri rc
Trong chng ny s m t mt s lp hm s chuyn i cc cp s
trong tp hp Z, N.
7.1 Hm s chuyn i cp s cng thnh cp s cng
Trc ht ta xt bi ton sau.
Bi ton 7.1. Tm cc hm s f(x) xc nh trn Z tho mn tnh cht
f(x +y) = f(x) +f(y), x, y Z
Gii. Trc ht ta kho st hm s f(x) trong tp hp N
Ti x = 0, y = 0, ta c f(0) = 0
Ti x = 1, y = 1, ta c f(2) = 2f(1) t f(1) = a ta c f(2) = 2a
Ti x = 2, y = 1, ta c f(3) = f(2) + f(1) f(3) = 3f(1) hay f(3) = 3a
Bng php qui np ta chng minh c f(n) = nf(1) hay f(n) = na, N

Vix, y Z
Thay x = y ta c f(0) = f(x) + f(x) f(x) = f(x). Khi ta c
hm f(x) l hm l.
Xt n Z, n < 0 n > 0, khi theo chng minh phn trn ta c
f(n) = na
m f(n) = f(n) f(n) = na
Vy hm s cn tm l f(x) = axx Z.
Bi ton 7.2. Tm hm s f(x) xc nh trn Z tho mn iu kin:
f
_
x + y
2
_
=
f(x) +f(y)
2
, x, y Z, x +y = 2k, k Z
158
7.1. Hm s chuyn i cp s cng thnh cp s cng 159
Gii. t f(0) = b, f(x) = b + g(x) th g(0) = 0, thay vo cng thc trn ta c
b +g(
x + y
2
) =
b +g(x) +b +g(y)
2
g(
x +y
2
) =
g(x) + g(y)
2
Ln lt chn x = 2k, y = 0 , hoc x = 0, y = 2k ta c
g(x +y)
2
=
g(x) + g(y)
2
g(x + y) = g(x) +g(y)x, y Z
Theo kt qu Bi ton 7.1 ta c g(x) = ax, x Z
Vy f(x) = ax +b
Bi ton 7.3. Chng minh rng iu kin cn v dy s a
n
lp thnh
mt cp s cng l dy cho phi tho mn h thc
2a
m+n
= a
2m
+ a
2n
, m, n N. (7.1)
Gii.
iu kin cn.
Gi s dy a
n
l mt cp s cng vi cng sai bng d.
Khi
a
n
= a
o
+ (n 1)d, n N

.
Vy nn
a
2n
+ a
2m
= 2a
o
+ (2m+ 2n 2)d
V
2a
m+n
= 2 [a
o
+ (m+n 1)d]
T ta c ngay cng thc 7.1
iu kin .
Gi s dy a
n
tho mn iu kin 7.1. Ta chng minh dy a
n
l mt cp
s cng vi cng sai bng d = a
1
a
o
Thay m = 0 vo cng thc 7.1 ta c
2a
n
= a
o
+ a
2n
Thay n = 0 vo cng thc 7.1 ta c
2a
m
= a
o
+ a
2m
7.1. Hm s chuyn i cp s cng thnh cp s cng 160
Thay kt qu trn vo cng thc 7.1 ta thu c
2a
m+n
= 2a
m
+ 2a
n
2a
o
a
m+n
= a
m
+a
n
a
o
(7.2)
Thay m = 1 vo cng thc 7.2 ,ta c
a
n+1
= a
n
+d, d = a
1
a
o
Vy dy a
n
l mt cp s cng.
B 7.1. iu kin cn v mt hm s chuyn mi cp s cng nguyn
dng thnh cp s cng l hm chuyn tp cc s t nhin thnh cp s cng.
Chng minh iu kin cn.
Nu hm f chuyn mi cp s cng thnh cp s cng th hin nhin hm f
chuyn tp cc s t nhin thnh mt cp s cng v tp cc s t nhin l cp
s cng vi cng sai nh nht l 1.
iu kin .
Hm f chuyn tp cc s t nhin thnh cp s cng,tc l dy f(n) l cp
s cng n N.Dy a
n
l cp s cng nguyn dng, vi cng bi l d N ta
phi chng minh dy f(a
n
) l cp s cng.
V dy f(n) l cp s cng nn theo cng thc 7.2 ta c
f(m+ n) = f(m) +f(n) f(o), m, n N
Dy a
n
l cp s cng nguyn dng, vi cng bi l d N suy ra a
n+1
=
a
n
+d
Khi
f(a
n+1
) = f(a
n
+d) = f(a
n
) + f(d) f(0)
hay f(a
n+1
) f(a
n
+d) = f(d) f(0) khng i.
Vy dy f(a
n
) l cp s cng vi cng sai l f(d) f(0)

Bi ton 7.4. Xc nh cc hm s f : Z R
+
chuyn mi cp s cng
a
n
, a
n
Z thnh cp s cng.
Gii. gii bi ton ny theo B 7.1 ta ch cn xc nh cc hm s chuyn
dy s t nhin thnh cp s cng.Hm f chuyn dy s t nhin thnh cp s
cng th ta c:
f(m+ n) = f(m) +f(n) f(o), m, n N
7.2. Hm s chuyn i cp s nhn thnh cp s nhn 161
f(m+n) f(0) = f(m) f(0) +f(n) f(0), m, n N.
t g(n) = f(n) f(0) ta c
g(m+n) = g(m) + g(n)
Khi theo bi ton 7.1 ta c g(x) = ax, x N trong a = g(1)
Do f(x) = g(x) + f(0) .t f(0) = b th f(x) = ax + bx N
Kt hp Bi ton 7.2 ta c:
Hm s chuyn i mi cp s cng thnh cp s cng trong tp cc s nguyn
l f(x) = ax +b, x Z.
Bi ton 7.5. Xc nh hm s f chuyn cp s cng nguyn dng a
n
cho
trc thnh cp s cng b
n
cho trc.
Gii. Ta xt hai trng hp sau:
(i) Nu a
n
N, theo kt qu Bi ton 7.4 ta c f(x) = ax + b, x
N, a, b R.
(ii) Nu a
n
N, ta c hm s f : N R c xc nh nh sau
f(n) =
_
b
n
nu n a
n

c
n
nu n , a
n

(7.3)
trong c
n
tu trong R. chuyn cp s cng nguyn dng a
n
cho trc thnh
cp s cng b
n
cho trc.
7.2 Hm s chuyn i cp s nhn thnh cp s nhn
Trn c s cc bi ton trn ta tm cc hm s chuyn cc cp s khc
trn tp hp s nguyn.Trc ht ta i tm nhng dy s thc hin php chuyn
tip mt i lng trung bnh ca cp phn t tng ng ca dy s.Cc bi ton
ny lin quan cht ch n vic chuyn tip cc cp s, n s phng on cc
cp s tng qut.
Bi ton 7.6. Xc nh dy s u
n
, sao cho
u
(
m+ n
2
) =
_
u(m)u(n), m, n,
m+ n
2
N

. (7.4)
Gii. Ta c
u(n) = u(
n + n
2
) =
_
u(n)u(n) =
_
[u(n)]
2
=[ u(n) [
7.2. Hm s chuyn i cp s nhn thnh cp s nhn 162
t u(1) = , u(2) = ( 0, o).
a) Nu = 0 th
u(n) = u(
1 + 2n 1
2
) =
_
u(1)u(2n 1) = 0, n N

Vy u(n) 0 l nghim duy nht ca phng trnh (7.4)


b) Nu > 0 v = 0 th
u(n) = u(
2 + 2n 2
2
) =
_
u(2)u(2n 2) = 0, n 2
Suy ra
u(n) =
_
nu n = 1
0 nu n 2
l nghim ca phng trnh (7.4)
c)Xt trng hp > 0 v > 0. Gi s tn ti n
o
3 sao cho u(n
o
) = 0
Th th
u(n
o
1) = u(
n
o
+ n
o
2
2
) =
_
u(n
o
)u(n
o
2) = 0.
Chn n
o
= 3 th u(n
o
1) = u(2) = 0, hay = 0, mu thun .
Do , ta c th gi thit rng u(n) > 0, vi mi n N

. Khi
u(2) = u(
3 + 1
2
) =
_
u(3)u(1) = 0.
Suy ra
u(3) =
u
2
(2)
u(1)
=

2

.
Mt khc
u(3) = u(
4 + 2
2
) =
_
u(4)u(2).
Suy ra
u(4) =
u
2
(3)
u(2)
=
(

)
2

=

3

2
Bng phng php quy np ton hc, ta chng minh c rng
u(n) =

n1

n2
, n 3.
7.2. Hm s chuyn i cp s nhn thnh cp s nhn 163
Ta c

n1

n2
= (

).(

)
n
t
_
= ab,
= ab
2
(a > 0, b > 0)
Suy ra

= a,

= b.
Vy nghim ca phng trnh (7.4) l
u(n) =
_
nu n = 1
0 nu n 2 ( 0)
hoc u(n) = ab
n
(a > 0, b > 0).
Bi ton 7.7. Xc nh dy s u
n
, sao cho
u(
m+n
2
) =
2u(m)u(n)
u(m) + u(n)
, m, n,
m+ n
2
N

(7.5)
Gii.
u(
m+ n
2
) =
2u(m)u(n)
u(m) +u(n)
u(
m+ n
2
) =
2
1
u(m)
+
1
u(n)
t
1
u(n)
, th phng trnh cho tng ng vi
v(
m+ n
2
) =
v(m) + v(n)
2
.
Theo Bi ton 7.2, ta c v(n) = an + b vi a, b 0, a + b > 0. Vy nghim ca
phng trnh (7.5) l
u(n) =
1
an + b
, a, b 0, a +b > 0.
7.2. Hm s chuyn i cp s nhn thnh cp s nhn 164
B 7.2. Chng minh rng iu kin cn v dy cc s dng a
n
lp
thnh mt cp s nhn l dy cho phi tho mn h thc
a
2
m+n
= a
2m
a
2n
, x, y N (7.6)
Chng minh. t lna
n
= b
n
vi mi n N. Khi a
n
= e
b
n
v (7.6) c dng
e
2
bm+n = e
b
2m
+ b
2n
, m, n N
Hay
2b
m+n
= b
2m
+ b
2n
(7.7)
Theo bi ton 7.3 th (7.7) l iu kin cn v dy s b
n
lp thnh
mt cp s cng vi cng sai d = b
1
b
0
.
T theo bi ton 6.13 suy ra iu phi chng minh.
Nhn xt:T cng thc (7.6) ta c
Xt m = 0 ta c a
2
n
= a
0
a
2n
Xt n = 0 ta c a
2
m
= a
0
a
2m
Suy ra a
2m
a
2n
=
a
2
n
a
2
m
a
2
0
Do a
2
m+n
=
a
2
n
a
2
m
a
2
0
Nn a
m+n
=
a
n
a
m
a
0

Bi ton 7.8. Xc nh dy cc s dng x


n
tho mn iu kin
x
mn
= x
m
x
n
, m, n N

Gii. Ta c x
1.n
= x
1
x
n
. Suy ra x
1
= 1. Gi s n = p l s nguyn t. Khi
bng qui np ta chng minh c x
p
k = (x
p
)
k
v nu n = p

1
1
p

2
2
. . . p
s
s
th:
x
n
= (x
p
1
)

1
(x
p
2
)

2
. . . (x
ps
)
s
x
p
c th nhn gi tr tu khi p l mt s nguyn t.
T ta c kt lun:
x
p
c th nhn gi tr tu khi p l mt s nguyn t v
x
n
= (x
p
1
)

1
(x
p
2
)

2
. . . (x
ps
)
s
khi n = p

1
1
p

2
2
. . . p
s
s
By gi ta xt tip bi ton sau:
Bi ton 7.9. Xc nh hm s f tho mn tnh cht f(mn) = f(m)f(n) trong
m, n N.
7.2. Hm s chuyn i cp s nhn thnh cp s nhn 165
Gii. Ta c f(1.n) = f(1)f(n). Suy ra f(1) = 1. Gi s n = p l s nguyn t.
Khi bng qui np ta chng minh c f(p
k
) = f(p)
k
v nu n = p

1
1
p

2
2
. . . p
s
s
th:
f(n) = f(p
1
)

1
f(p
2
)

2
. . . f(p
s
)
s
f(p) c th nhn gi tr tu khi p l mt s nguyn t.
T ta c kt lun:
f(p) c th nhn gi tr tu khi p l mt s nguyn t v
f(n) = f(p
1
)

1
f(p
2
)

2
. . . f(p
s
)
s
khi n = p

1
1
p

2
2
. . . p
s
s
Bi ton 7.10. Chng minh hm s f : N R xc nh nh sau vi n =
p

1
1
p

2
2
. . . p
s
s
, p
i
l cc s nguyn t th f(n) = f(p
1
)

1
f(p
2
)

2
. . . f(p
s
)
s
trong
f(p
i
) tu , i = 1, 2, 3, . . . chuyn cp s nhn c cng bi nguyn t thnh
cp s nhn.
Gii. Gi s c cp s nhn n
0
q
k
, k = 0; 1; 2 . . . , n
0
; q N

ta phi chng minh


f(n
0
q
k
) cng l cp s nhn. Tht vy theo kt qa bi ton 7.9 trn ta c ngay
iu phi chng minh.
Sau y ta xt bi ton
Bi ton 7.11. Chng minh rng hm s f chuyn mi cp s nhn thnh cp
s nhn khi v ch khi hm s chuyn cp s nhn c cng bi nguyn t thnh
cp s nhn.
Gii. iu kin cn.
Nu hm s f chuyn mi cp s nhn thnh cp s nhn th hin nhin n
chuyn cp s nhn c cng bi nguyn t thnh cp s nhn.
iu kin .
Nu hm s f : N R chuyn cp s nhn c cng bi nguyn t thnh cp
s nhn. Gi s u
n
l cp s nhn ta phi chng minh f(u
n
) cng l cp s
nhn. Vi u
n
= u
0
q
n
ta xt hai trng hp sau:
Nu q l s nguyn t th bi ton c chng minh.
Nu q khng l s nguyn t th q = p

1
1
p

2
2
. . . p
s
s
trong p
i
, i N

l cc
s nguyn t. Khi ta c
f(u
n
) = f(u
0
q
n
) = f(u
0
(p

1
1
p

2
2
. . . p
s
s
)
n
)
= f(u
0
(p
1
)

1
.n
(p
2
)

2
.n
. . . (p
s
)
s.n
)
= f(u
0
)f

1
.n
(p
1
)
f

2
.n
(p
2
)
. . . f
s.n
(ps)
7.2. Hm s chuyn i cp s nhn thnh cp s nhn 166
Theo B 7.6 ta chng minh f
2
(u
m+n
) = f(u
2m
)f(u
2n
)
Ta c f
2
(u
m+n
) = (f(u
0
)f

1
.(m+n)
(p
1
)
f

2
.(m+n)
(p
2
)
. . . f
s.(m+n)
(ps)
)
2
= f
2
(u
0
)f
2
1
.(m+n)
(p
1
)
f
2
2
.(m+n)
(p
2
)
. . . f
2s.(m+n)
(ps)
f(u
2m
)f(u
2n
) = f(u
0
)f

1
.2m
(p
1
)
f

2
.2m
(p
2
)
. . . f
s.2m
(ps)
f(u
0
)f

1
.2n
(p
1
)
f

2
.2n
(p
2
)
. . . f
s.2n
(ps)
= f
2
(u
0
)f
2
1
.(m+n)
(p
1
)
f
2
2
.(m+n)
(p
2
)
. . . f
2s.(m+n)
(ps)
Vy ta c iu phi chng minh.
Chng 8
Mt s bi ton xc nh dy
s trong lp dy tun hon
cng tnh v nhn tnh.
8.1 Mt s bi ton xc nh dy s trong lp dy
tun hon cng tnh
Bi ton 8.1. Xc nh dy x
n
sao cho x
n+3
= x
n
+ 1, n = 1; 2; 3; . . .
Gii. t x
n
=
n
3
+ y
n
. Khi ta c
y
n+3
+
n + 3
3
=
n
3
+y
n
+ 1
hay y
n+3
= y
n
, n N. Vy nn
_

_
y
0
= y
3
= y
6
=
y
1
= y
4
= y
7
=
y
2
= y
5
= y
8
=
y
n
=
_

_
a tu vi n = 3k, k N
b tu vi n = 3k + 1, k N
c tu vi n = 3k + 2, k N
Bi ton 8.2. Xc nh dy s x
n
sao cho x
n+3
= 2x
n
Gii. t x
n
= 2
n
3
y
n
. Suy ra
2
n + 3
3
y
n+3
= 2(2
n
3
y
n
) y
n+3
= y
n
y
n+3
= y
n
.
167
8.1. Mt s bi ton xc nh dy s trong lp dy tun hon cng tnh 168
Vy nn
_

_
y
0
= y
3
= y
6
=
y
1
= y
4
= y
7
=
y
2
= y
5
= y
8
=
y
n
=
_

_
a tu vi n = 3k, k N
b tu vi n = 3k + 1, k N
c tu vi n = 3k + 2, k N
Vy x
n
= 2
n
3
y
n
trong
y
n
=
_

_
a tu vi n = 3k, k N
b tu vi n = 3k + 1, k N
c tu vi n = 3k + 2, k N
Sau y ta xy dng bi ton tng qut sau.
Bi ton 8.3. Xc nh dy s u
n
tho mn iu kin
u
n+b
= u
n
+ d
Trong n, b N, d R
Gii. t u
n
=
d
b
n +v
n
. Thay vo cng thc u
n+b
= u
n
+ d, ta c
d
b
(n +b) + v
n+b
=
d
b
n + v
n
+d.
Suy ra v
n+b
= v
n
do v
n
l dy tun hon cng tnh chu k b
Vy u
n
=
d
b
n +v
n
vi v
n
l dy tun hon cng tnh chu k b.
Bi ton 8.4. Xc nh dy s u
n
tho mn iu kin
u
n+b
= c.u
n
,
trong n, b N

, c R
Gii. t u
n
= c
n
b
v
n
ta c
c
n+b
b
v
n+b
= cc
n
b
v
n
.
Suy ra v
n+b
= v
n
do v
n
l dy tun hon cng tnh chu k b Vy u
n
= c
n
b
v
n
vi v
n
l dy tun hon cng tnh chu k b.
Bi ton 8.5. Xc nh dy s u
n
tho mn iu kin
u
n+b
= cu
n
+d,
trong n, b N

, c, d R.
8.1. Mt s bi ton xc nh dy s trong lp dy tun hon cng tnh 169
Gii. Xt trng hp c = 1 theo kt qu bi ton 8.3, ta c
u
n
=
d
b
n +v
n
vi v
n
l dy tun hon cng tnh chu k b.
Xt trng hp c ,= 1. t u
n
= v
n
+
d
1 c
. Khi ta c
v
n+b
+
d
1 c
= c(v
n
+
d
1 c
) +d, n N
hay
v
n+b
= cv
n
.
Theo kt qu bi ton 8.4, ta c v
n
= [ c [
n
b
x
n
, trong
x
n+b
=
_
x
n
vi c > 0
x
n
vi c < 0
Vy nn
f(x) =
_

_
d
1 c
+ c
n
b
x
n
, vi x
n
tu sao cho x
n+b
= x
n
, vi c > 0
d
1 c
+ [ c [
n
b
x
n
, vi x
n
tu sao cho x
n+b
= x
n
, vi c < 0
Kt lun:
- Nu c = 1 th u
n
=
d
b
n +v
n
vi v
n
l dy tun hon cng tnh chu k b.
- Nu c ,= 1 th
f(x) =
_

_
d
1 c
+c
n
b
x
n
, vi x
n
tu sao cho x
n+b
= x
n
, vi c > 0
d
1 c
+[ c [
n
b
x
n
, vi x
n
tu sao cho x
n+b
= x
n
vi c < 0
Bi ton 8.6.
8.2. Hm s xc nh trn tp cc s nguyn 170
8.2 Hm s xc nh trn tp cc s nguyn
8.2.1 Hm s chuyn i cc php tnh s hc
Khi xt lp phng trnh hm vi cp ch s t do dng i xng quen bit, ta
thng s dng php th ch s bng cc bin mi a phng trnh hm
cho v mt dng phng trnh hm mi bit cch gii. Tuy nhin, trong cc
trng hp s dng php th ch s tng qut, nghim nhn c ca phng trnh
mi, nhn chung khng tho mn iu kin bi ra. V vy, nghim ca phng
trnh mi cn c th li thng qua cc d liu ca bi ra. Ta xt mt s v d
minh ho.
Bi ton 8.7. Xc nh hm s f : Z R tho mn cc iu kin
f(m+ n) = f(m) +f(n) +mn (m, n Z). (1)
Gii. T phng trnh (1) ta nhn c
f(n + 1) = f(1) + f(n) +n, n Z,
hay
f(n + 1) f(n) = a + n, a = f(1), n Z. (2)
Phng trnh f(n + 1) f(n) = a + n l mt phng trnh sai phn tuyn tnh
khng thun nht cp 1. Do phng trnh c trng tng ng c nghim = 1,
nn ta c nghim tng qut ca phng trnh thun nht f(n + 1) f(n) = 0 l

f(n) = c (3)
Ta vit
n =
1
2
(n + 1)
2

1
2
n
2

1
2
.
Khi , nghim ring ca (2) c dng
f(n)

= n(dn + e).
Thay f(n)

vo (2) ta c
f(n)

=
_
a
1
2
_
n. (4)
V f(n) =

f(n) +f(n)

nn t (3) v (4) ta c nghim ca (2) l


f(n) = c +
1
2
n
2
+
_
a
1
2
_
n. (5)
8.2. Hm s xc nh trn tp cc s nguyn 171
Do f(1) = a, t (5) ta c c = 0.
Thay c = 0 vo (5), ta thu c nghim ca (2)
f(n) =
1
2
n
2
+
_
a
1
2
_
. (6)
Th li ta thy nghim dng (6) tho mn iu kin ca u bi.
Bi ton 8.8. Tn ti hay khng tn ti mt hm s f : Z R tho mn iu
kin
f(m+ n) = f(m) +f(n) +m +n, (m, n Z). (7)
Gii. Lp li cch gii nh i vi Bi ton 1, t phng trnh (l) ta suy ra
f(n + 1) = f(1) + f(n) +n + 1
hay
f(n + 1) f(n) = a + n vi a = f(1) + 1. (8)
Phng trnh f(n + 1) f(n) = a + n l mt phng trnh sai phn tuyn tnh
khng thun nht cp 1. Do phng trnh c trng c nghim = 1 nn ta c
nghim tng qut ca phng trnh thun nht f(n + 1) f(n) = 0 l

f(n) = c (9)
Ta vit
n =
1
2
(n + 1)
2

1
2
n
2

1
2
.
Khi , nghim ring ca (8) c dng x

n
= n(dn + e). Thay f(n)

vo (8) ta
c
f(n)

=
1
2
n
2
+
_
a
1
2
_
n. (10)
V f(n) =

f(n) +f(n)

nn t (9) v (10) ta c nghim ca (8) l


f(n) = c +
1
2
n
2
+
_
a
1
2
_
n. (11)
Do f(1) = a 1, t (11) ta c c = 1.
Thay c = 1 vo (11), ta c nghim ca (8)
f(n) =
1
2
n
2
+
_
a
1
2
_
n + 1. (12)
Th li ta thy nghim dng (12) khng tho mn iu kin ca u bi. Vy
khng tn ti mt hm s f : Z R tho mn iu kin
f(m+n) = f(m). + f(n) + m+n (m, n Z).
8.2. Hm s xc nh trn tp cc s nguyn 172
Bi ton 8.9. Xc nh f : Z R
+
tho mn iu kin
f(mn) = f(m)f(n) (m, n Z).
Gii.
Ta c x
1.n
= f(1)f(n). Suy ra f(1) = 1. Gi s n = p l mt s nguyn t.
Khi f(p
k
) = (f(p))
k
(quy np) v nu n = p

1
1
. . . p
s
s
th
f(n) = (f(p
1
))

1
(f(p
s
))
s
.
Vy f(p) c th nhn gi tr tu khi p l mt s nguyn t.
Kt lun:
f(p) c th nhn gi tr tu khi p l mt s nguyn t v
f(n) = (f(p
1
))

1
(f(p
s
))
s
khi n = p

1
1
. . . tp
s
s
.
Bi ton 8.10. Xc nh dy f : Z R tho mn iu kin
F(m+ n) +f(n m) = f(3n) (m, n Z, n m).
Gii.
Cho m = 0, ta c 2f(n) = f(3n). Suy ra f(0) = 0. t m = n ta c
f(2n) = f(3n). Suy ra, mt mt th
f(4n) = f(6n) = f(9n)
v mt khc th
f(4n) + f(2n) = f(9n).
T suy ra
f(n) =
1
2
f(3n) =
1
2
f(2n) = 0
vi mi n Z.
8.2.2 Hm s chuyn tip cc i lng trung bnh
Trong mc ny, ta i tm nhng hm s thc hin php chuyn tip mt i
lng trung bnh ca cp ch s sang mt i lng trung bnh ca cp phn t
tng ng ca hm s. Cc bi ton ny lin quan cht ch n vic chuyn tip
cc cp s; n s m phng cc cp s tng qut, chng hn, ta c th chuyn
mt cp s cng sang mt cp s nhn, cp s iu ho,...
Di y ta xt mt s bi ton chuyn tip cc i lng trung bnh c bn
trong chng trnh ph thng.
1) Php chuyn cc i lng trung bnh cng
8.2. Hm s xc nh trn tp cc s nguyn 173
Bi ton 8.11. Xc nh hm s u(n), sao cho
u
_
m+ n
2
_
=
u(m) + u(n)
2
_
m, n,
m+ n
2
Z
_
Gii. t u(1) = , u(2) = . Ta c
u(2) = u
_
3 + 1
2
_
=
u(3) +u(1)
2
.
Suy ra
u(3) = 2u(2) u(1) = 2 .
Tip tc qu trnh nh vy, ta c
u(3) = u
_
4 + 2
2
_
=
u(4) +u(2)
2
.
Suy ra
u(4) = 2u(3) u(2) = 2(2 ) = 3 2.
Bng phng php quy np, ta thu c
u(n) = (n 1) (n 2), n.
Vy
_
u(n) = ( )n + 2 , n,
u(1) = , u(2) = .
t = a +b; = 2a +b, th a = v b = 2 .
Do , nghim ca phng trnh l u(n) = an +b; a, b tu .
2) Php chuyn i lng trung bnh cng sang trung bnh iu ho
Bi ton 8.12. Xc nh hm s u(n) Z sao cho
u
_
m+n
2
_
=
2u(m)u(n)
u(m) + u(n)
_
m, n,
m+ n
2
Z
_
.
Gii. Ta c
u
_
m+n
2
_
=
2u(m)u(n)
u(m) +u(n)
u
_
m+n
2
_
=
2
1
u(m)
+
1
u(n)
.
t
1
u(n)
= v(n), th phng trnh cho tng ng vi
v
_
m+n
2
_
=
v(m) +v(n)
2
.
8.2. Hm s xc nh trn tp cc s nguyn 174
Theo Bi ton 1, v(n) = an +b; a, b 0, a +b > 0.
Vy nghim ca phng trnh l
u(n) =
1
an +b
; a, b 0; a +b > 0.
3) Php chuyn i lng trung bnh cng sang trung buh nhn
Bi ton 8.13. Xc nh hm s u(n) sao cho
u
_
m+ n
2
_
=
_
u(m)u(n);
_
m, n,
m+ n
2
_
Z.
Gii. Ta c
u(n) = u
_
n +n
2
_
=
_
u(n)u(n) =
_
[u(n)]
2
= [u(n)[ 0.
t u(1) = , u(2) = ( 0, 0).
a) Nu = 0 th
u(n) = u
_
1 + 2n 1
2
_
=
_
u(1)u(2n 1) = 0, n Z.
Vy u(n) 0 l nghim duy nht ca phng trnh .
b) Nu > 0 v = 0 th
u(n) = u
_
2 + 2n 2
2
_
=
_
u(2)u(2n 2) = 0, n 2.
Suy ra
u(n) =
_
, nu n = 1
0 nu n 2
l nghim ca phng trnh.
c) Xt trng hp > 0 v > 0. Gi s tn ti n
0
3 sao cho u(n
0
) = 0.
Th th
u(n
0
1) = u
_
n
0
+n
0
2
2
_
=
_
u(n
0
)u(n
0
2) = 0.
Chn n
0
= 3 th u(n
0
1) = u(2) = 0, hay = 0, mu thun.
Do , c th gi thit rng u(n) > 0 vi mi n Z. Ta c
u(2) = u
_
3 + 1
2
_
=
_
u(3)u(1).
8.2. Hm s xc nh trn tp cc s nguyn 175
Suy ra
u(3) =
u
2
(2)
u(1)
=

2

.
Mt khc
u(3) = u
_
4 + 2
2
_
=
_
u(4)u(2).
Suy ra
u(4) =
u
2
(3)
u(2)
=
_

_
2

=

3

2
.
Bng phng php quy np ton hc, ta chng minh c rng
u(n) =

n1

n2
, n 3.
M

n1

n2
=
_

__

_
n
t
_
= ab,
= ab
2
(a > 0, b > 0).
Suy ra

= a,

= b.
Vy nghim ca phng trnh l
u(n) =
_
nu n = 1
0 nu n 2
( 0)
hoc u(n) = a.b
n
(a > 0, b > 0).
4) Php chuyn i lng trung bnh cng sang trung bnh bc hai
Bi ton 8.14. Xc nh hm s u(n), sao cho
u
_
m+n
2
_
=
_
u
2
(m) +u
2
(n)
2
_
m, n,
m+n
2
Z
_
.
8.2. Hm s xc nh trn tp cc s nguyn 176
Gii. Ta c
u(n) = u
_
n +n
2
_
=
_
u
2
(n) +u
2
(n)
2
=
_
u
2
(n) = [u(n)[ 0, n Z.
t u(1) = 0 ; u(2) = 0. Ta c
u(2) = u
_
3 + 1
2
_
=
_
u
2
(3) +u
2
(1)
2
.
Suy ra
u
2
(3) = 2u
2
(2) u
2
(1) = 2
2

2
u(3) =
_
2
2

2
(

2).
Tng t
u(3) = u
_
4 + 2
2
_
=
_
u
2
(4) +u
2
(2)
2
.
Suy ra
u
2
(4) = 2u
2
(3) u
2
(2) = 2(2
2

2
)
2
= 3
2
2
2
hay
u(4) =
_
3
2
2
2
_

_
3
2
_
.
Bng quy np ton hc, ta chng minh c h thc
u(n) =
_
(n 1)
2
(n 2)
2
, n 3.
Nhn xt rng, ta lun c
_
(n 1)
2
(n 2)
2
=
_
(
2

2
)n + 2
2

2
.
t
_

2
= a +b

2
= 2a +b.
Suy ra
_
a =
2

2
b = 2
2

2
.
Vy nghim ca phng trnh l u(n) =

an +b ; a 0, a +b 0.
Nhn xt 8.1. Trong c bn bi ton nu trn, nu ta thay m bi (n + 1)
v n bi (n 1) th ta c th a chng c v cc phng trnh sai phn quen
bit.
8.2. Hm s xc nh trn tp cc s nguyn 177
8.2.3 Phng trnh trong hm s vi cp bin t do
Trong mc ny, ta i tm nhng hm s thc hin php chuyn tip mt biu
thc i s ca cp ch s sang mt i lng khc ca cp phn t tng ng
ca dy s. Cc bi ton ny lin quan cht ch n vic chuyn tip cc hm
s; n s m phng cc hm s c bit trong s hc, i s,...
Bi ton 8.15. Tm hm f : Z Z tha mn cc iu kin f(1) = a Z v
f(m+ n) +f(m n) = 2f(m)f(n), m, n Z.
Gii. Cho m = n = 0 ta c f(0) = 0 hoc f(0) = 1. Nu f(0) = 0 th thay
n = 0 ta c 2f(m) = 0 vi mi m Z. Do vy f(m) 0 v ng vi a = 0.
Nu f(0) = 1, cho m = n = 1 ta thu c f(2) = 2a
2
1.
Tip tc thay m = 2; n = 1 vo iu kin bi ra ta c f(3) = 4a
3
3a. T
ta c d on f(n) = T
n
(a) vi mi n 1.
D on c chng minh d dng bng phng php quy np.
Mt khc, cho m = 0 ta c f(n)+f(n) = 2f(0)f(n) = 2f(n) nn f(n) =
f(n). Vy f(n) l hm chn. Vy ta c
f(m) =
_

_
1 khi m = 0,
a khi m = 1,
T
|m|
(a) khi [m[ 1, m Z.
Bi ton 8.16. Tm hm f : Z R tha mn cc iu kin f(0) ,= 0, f(1) =
5
2
v
f(m+n) + f(mn) = f(m)f(n), m, n Z.
Gii. Cho m = n = 0 ta c, do f(0) ,= 0, f(0) = 2. Tip theo, theo quy np
ta c
f(n) = 2
n
+ 2
n
, n Z.
Th li ta thy hm ny tho mn iu kin bi ra.
Bi ton 8.17. Tm hm f : Z [0, +) tha mn cc iu kin f(1) = 1 v
f(m+ n) +f(m n) =
1
2
[f(2m) +f(2n)], m, n Z, m n.
Gii. Cho m = n = 0 ta c f(0) = 0. Cho m = 1, n = 0 th
f(1) + f(1) =
1
2
[f(2) + f(0)].
Suy ra f(2) = 4.
8.2. Hm s xc nh trn tp cc s nguyn 178
Chng minh bng quy np ta c f(n) = n
2
.
Tht vy, do f(k) + f(k) =
1
2
[f(2k) + f(0)] nn c ngay f(2k) = 4k
2
.
Cng vy, do f(k + 1) +f(k 1) =
1
2
[f(2k) + f(2)] nn ta c
f(k + 1) =
1
2
f(2k) + 2 f(k 1) = (k + 1)
2
.
Bi ton 8.18. Tm cc a thc hai bin P(m, n) (m, n Z) tho mn iu
kin
a) P(am, an) = a
2
P(m, n) vi mi m, n, a Z,
b) P(b + c, a) +P(c +a, b) +P(a +b, c) = 0 vi mi a, b, c Z,
c) P(1, 0) = 1.
Gii.
Trong b) t b = 1 a; c = 0 ta c
P(1 a, a) = 1 P(a, 1 a). (1)
Li t c = 1 a b v kt hp vi a) ta c
P(a +b, 1 a b) = P(a, 1 a) + P(b, 1 b) + 2. (2)
t f(m) = P(m, 1 m) +2. Khi f(1) = P(1, 0) +2 = 3 v (5) tr thnh
f(m+ n) = f(m) +f(n). l phng trnh dy chuyn i php cng
_
f(m+ n) = f(m) +f(n),
f(1) = 3.
(3)
Phng trnh (3) c nghim duy nht f(n) = 3n. Vy nn
P(n, 1 n) = 3n 2. (4)
Bng phng php quy np ta s thu c
P(a, b) = (a +b)
2
_
3
a
a +b
2
_
= (a +b)(a 2b), a, b Z.
Tm li P(m, n) = (m+n)
2
(m 2n).
Bi ton 8.19. Cho a thc Chebyshev T
n
(x) = cos(narccos x). Chng minh
rng vi m, n Z; n m v x R th T
n
(x) l nghim ca phng trnh dy
sau
T
n+m
(x) +T
nm
(x) = 2T
n
(x)T
m
(x).
8.2. Hm s xc nh trn tp cc s nguyn 179
Gii. S ng nh ngha T
n
(x) v phng php quy np hoc s dng cc cng
thc
cos(n +m)x + cos(n m)x = 2 cos nx cos mx
v
cosh(n + m)x + cosh(n m)x = 2 cosh(nx) cosh(mx),
ta c ngay iu phi chng minh.
Bi ton 8.20. Tm hm f Z Z tha mn cc iu kin
N Z : N < f(n) < N n Z,
f(m+ n) +f(m n) = 2f(m)f(n) m, n Z. (1)
Gii. Cho m = n = 0 ta c f(0) 0, 1. Gi s f(0) = 0. Cho n = 0 trong
(1) ta c 2f(m) = 2f(m)f(0) = 0 v f 0.
Gi s f(0) = 1. Cho m = 0 trong (1) ta thu c f(n) = f(n) vi mi
n Z. Vy ch cn xt n Z. Cho n = 1 trong (1), ta c
f(m+ 1) = 2f(m)f(1) f(m1)
v thu c cng thc truy hi theo f(1). Nu [f(1)[ 2 th t gi thit ta c
f(2n) = 2[f(n)]
2
1
tng v khng gii ni, tri vi gi thit. Vy f(1) 1, 0, 1.
Vi f(1) = 1 th f(n) = (1)
n
(quy np).
Vi f(1) = 1 th f(n) 1.
Vi f(1) = 0 ta c dy tun hon (quy np)
f(4m) = 1, f(4m+ 1) = 0, f(4m+ 2) = 1, f(4m+ 3) = 0.
Suy ra f(2) = 4. Chng minh bng quy np ta c f(n) = n
2
. Tht vy, do
f(k) +f(k) = (1/2)[f(2k) +f(0)] nn c ngay f(2k) = 4k
2
. Cng vy, do f(k +
1) +f(k 1) = (1/2)[f(2k) + f(2)] nn ta c
f(k + 1) =
1
2
f(2k) + 2 f(k 1) = (k + 1)
2
.
8.2. Hm s xc nh trn tp cc s nguyn 180
8.2.4 Mt s dng ton lin quan n dy truy hi
Bi ton 8.21. K hiu
u
n
=
_
2
0
sin
n
xdx, n Z.
Xc nh hm s f : Z R theo cng thc
f(n) = (n + 1)u
n
g(n + 1), n Z.
Gii. S dng cng thc tch phn tng phn, ta thu c
u
n
= cos x sin
n1
x

2
0
+
_
2
0
(n 1) sin
n2
x cos
2
xdx
=
_
2
0
(n 1) sin
n2
x(1 sin
2
x)dx
= (n 1)(u
n1
u
n
).
T y suy ra
g(n + 2) =
n + 1
n + 2
u
n
, n Z. (1)
T (1) ta nhn c
f(n + 1) = (n + 2)g(n + 1)g(n + 2)
= (n + 2)g(n + 1)
n + 1
n + 2
u
n
= (n + 1)g(n + 1)u
n
= f(n).
Vy nn
f(n) = f(0) =

2
.
Bi ton 8.22. K hiu
u
n
=
_

0
cos
n
x cos nxdx, n Z.
Xc nh hm s f : Z R theo cng thc
f(n) = 2
n
u
n
, n Z.
8.2. Hm s xc nh trn tp cc s nguyn 181
Gii. t cos
n
x = u v cos nxdx = dv th theo cng thc tch phn tng phn,
ta thu c
u
n
=
1
n
cos
n
x sinx

0
+
_

0
cos
n1
x sinx sinnxdx
=
1
2
_

0
cos
n1
x[cos(n 1)x + cos(n + 1)x]dx
=
1
2
u
n1

1
2
u
n
+
1
2
_

0
cos
n1
x sinx + sinnxdx
=
1
2
u
n1

1
2
u
n
+
1
2
u
n
.
Vy nn
u
n
=
1
2
u
n1
=
1
4
u
n2
= =
1
2
n1
u
1
=
1
2
n1

2
.
Bi ton 8.23. Xc nh hm s u
n
c tnh theo cng thc
u
n
=
_
4
0
tan
2n
xdx
Gii. Ta vit u
n
di dng sau
u
n
=
_
4
0
tan
2n
xdx
=
_
4
0
tan
2n2
x[(tan
2
x + 1) 1]dx
=
_
4
0
tan
2n2
xd tanx u
n1
=
tan
2n1
x
2n 1

4
0
u
n1
=
1
2n 1
u
n1
.
Do vy
u
n
+u
n1
=
1
2n 1
,
u
n1
+ u
n2
=
1
2n 3
,

8.2. Hm s xc nh trn tp cc s nguyn 182
u
1
+u
0
=
1
2 1
.
Suy ra
u
n
= (1)
n
_

4
+
n

k=1
(1)
k
2k 1
_
.
Bi ton 8.24. Xc nh hm s f : Z R c tnh theo cng thc
f(n) =
1
_
0
x
n

1 xdx, n Z.
Gii. t x
n
= u,

1 xdx = dv th
f(n) =
1
_
0
x
n

1 xdx
=
_

2
3
x
n
(1 x)
3
2

1
0
+
2n
3
1
_
0
x
n1

1 x(1 x)dx
=
2n
3
1
_
0
(x
n1
x
n
)

1 xdx
=
2n
3
1
_
0
(x
n1

1 xdx
2n
3
1
_
0
x
n

1 xdx
2n
3
x
n1

2n
3
f(n).
Vy nn
f(n) =
2n
2n 3
x
n1
.
V f(0) =
2
3
nn ta c ngay
f(n) =
1
_
0
x
n

1 xdx = 2
(2n)!!
(2n + 3)!!
, n Z.
Bi ton 8.25. Xc nh hm f : Z Z tho mn cc iu kin
f(0) = 1, f(f(n) = f(f(n + 2) + 2) = n n Z.
8.2. Hm s xc nh trn tp cc s nguyn 183
Gii. Nhn xt rng f l nh x 1-1
f(m) = f(n) f(f(m)) = f(f(n)) m = n.
Vy nn
f(n) = f(n + 2) + 2 n Z.
Suy ra
f(n + 2) = f(n) 2, f(0) = 1, f(1) = f(f(0)) = 0.
Vy nn
f(2) = f(0) 2 = 1,
f(3) = f(1) 2 = 2,
f(n) = (n 1).
Tng t
f(1) = f(1) + 2 = 2,
f(2) = f(0) + 2 = 3,
f(3) = f(1) + 2 = 5,
f(n) = (n 1).
Bi ton 8.26. Cho gc vi 0 < < . Xc nh cp s a, b sao cho dy hm
P
n
(x) c tnh theo cng thc
P
n
(x) = x
n
sin x sin(n) + sin(n 1)
lun lun chia ht cho f(x) = x
2
+ax +b.
Gii. Vi n = 3 th
P
3
(x) = x
3
sin x sin(3) + sin 2 = sin (x + 2 cos )(x
2
2x cos + 1).
T suy ra vi f(x) = x
2
+ 2x cos + 1 th P
3
(x)
.
.
.f(x). Vi n 3 th
P
n+1
(x) = xP
n
(x) + (x
2
2x cos + 1) sinn.
Suy ra f(x) = x
2
+ 2x cos + 1.
8.3. Hm s xc nh trn tp cc s hu t 184
Bi tp
Bi 1. Xc nh hm s f : Z R nu bit: f(1) = a, f(m+n) = f(n) +f(m);
Bi 2. Xc nh hm s f : Z R nu bit: f(1) = a, f(mn) = f(n) +f(m)
(m, n, mn Z).
Bi 3. Xc nh hm s f : Z R nu bit f
_
m
n
_
= f(n)+f(m) (m, n,
m
n
Z).
Bi 4. Xc nh hm s f : Z R tho mn iu kin f
_
m
n
_
= f(m) f(n)
(m, n,
m
n
Z).
Bi 5. Xc nh hm s f : Z R tho mn iu kin f(m + n) = f(m)f(n)
(m, n Z).
Bi 6. Xc nh hm s f : Z R nu bit f(m+n) +f(mn) =
1
2
(f(2m) +
f(2n) , (m, n, mn Z).
Bi 7. Xc nh hm s f : Z R
+
tho mn iu kin f(m + n) =
f(m)
f(n)
(m, n Z).
Bi 8. Xc nh hm s f : Z R
+
tho mn iu kin f
_
m+n
2
_
=
_
f(m)f(n)
(m, n,
m+n
2
Z).
Bi 9 . Xc nh hm s f : Z R
+
tho mn iu kin xm+n
2
=
2f(m)f(n)
f(m)+f(n)
(m, n,
m+n
2
Z).
Bi 10 . Xc nh hm s f : Z R
+
tho mn iu kin f
_
m+n
2
_
=
_
f(m)
2
+f(n)
2
2
(m, n,
m+n
2
Z).
8.3 Hm s xc nh trn tp cc s hu t
Bi ton 8.27. Xc nh hm s f : Q R tho mn cc iu kin
f( +) = f() +f() + (, Q). (1)
Gii. T phng trnh (1) ta nhn c
f( + 1) = f(1) +f() +
hay
f( + 1) f() = a +, a = f(1). (2)
Phng trnh f( + 1) f() = a + l mt phng trnh sai phn tuyn tnh
khng thun nht cp 1. Do phng trnh c trng c nghim = 1 nn ta c
nghim tng qut ca phng trnh thun nht f( + 1) f() = 0 l

f() = c (3)
8.3. Hm s xc nh trn tp cc s hu t 185
Ta vit
=
1
2
( + 1)
2

1
2

1
2
.
Khi , nghim ring ca (2) c dng
f()

= (d + e).
Thay f()

vo (2) ta c
f()

=
_
a
1
2
_
. (4)
V f() =

f() +f()

nn t (3) v (4) ta c nghim ca (2) l


f() = c +
1
2

2
+
_
a
1
2
_
n. (5)
Do f(1) = a, t (5) ta c c = 0.
Thay c = 0 vo (5), ta thu c nghim ca (2)
f() =
1
2

2
+
_
a
1
2
_
. (6)
Th li ta thy nghim dng (6) tho mn iu kin ca u bi.
Bi ton 8.28. Tn ti hay khng tn ti mt hm s f : Q R tho mn
iu kin
f( + n) = f() + f() + + n, (, Q). (7)
Gii. Lp li cch gii nh i vi Bi ton trn, t phng trnh (l) ta suy ra
f( + 1) = f(1) + f() + + 1
hay
f( + 1) f() = a + vi a = f(1) + 1. (8)
Phng trnh f( + 1) f() = a + l mt phng trnh sai phn tuyn tnh
khng thun nht cp 1. Do phng trnh c trng c nghim = 1 nn ta c
nghim tng qut ca phng trnh thun nht f( + 1) f() = 0 l

f() = c (9)
Ta vit
=
1
2
( + 1)
2

1
2

1
2
.
8.3. Hm s xc nh trn tp cc s hu t 186
Khi , nghim ring ca (8) c dng x

n
= n(d + e). Thay f()

vo (8) ta
c
f()

=
1
2

2
+
_
a
1
2
_
. (10)
V f() =

f() +f()

nn t (9) v (10) ta c nghim ca (8) l


f() = c +
1
2

2
+
_
a
1
2
_
. (11)
Do f(1) = a 1, t (11) ta c c = 1.
Thay c = 1 vo (11), ta c nghim ca (8)
f() =
1
2

2
+
_
a
1
2
_
+ 1. (12)
Th li ta thy nghim dng (12) khng tho mn iu kin ca u bi. Vy
khng tn ti mt hm s f : Q R tho mn iu kin
f( +) = f(). +f() + + (, Q).
Bi ton 8.29. Xc nh f : Q R
+
tho mn iu kin
f() = f()f() (, Q).
Gii. Ta c x
1.
= f(1)f(). Suy ra f(1) = 1. Gi s = p l mt s nguyn
t. Khi f(p
k
) = (f(p))
k
(quy np) v nu = p

1
1
. . . p
s
s
th
f() = (f(p
1
))

1
(f(p
s
))
s
.
Vy f(p) c th nhn gi tr tu khi p l mt s nguyn t.
Kt lun:
f(p) c th nhn gi tr tu khi p l mt s nguyn t v
f() = (f(p
1
))

1
(f(p
s
))
s
khi = p

1
1
. . . tp
s
s
.
Bi ton 8.30. Xc nh dy f : Q R tho mn iu kin
F( +) + f( ) = f(3) (, Q, ).
Gii. Cho = 0, ta c 2f() = f(3). Suy ra f(0) = 0. t = ta c
f(2) = f(3). Suy ra, mt mt th
f(4) = f(6) = f(9)
8.3. Hm s xc nh trn tp cc s hu t 187
v mt khc th
f(4) +f(2) = f(9).
T suy ra
f() =
1
2
f(3) =
1
2
f(2) = 0
vi mi Q.
Tip theo, ta i tm nhng hm s thc hin php chuyn tip mt i lng
trung bnh ca cp ch s sang mt i lng trung bnh ca cp phn t tng
ng ca hm s. Cc bi ton ny lin quan cht ch n vic chuyn tip cc
cp s; n s m phng cc cp s tng qut, chng hn, ta c th chuyn mt
cp s cng sang mt cp s nhn, cp s iu ho,...
Di y ta xt mt s bi ton chuyn tip cc i lng trung bnh c bn
trong chng trnh ph thng.
1) Php chuyn cc i lng trung bnh cng
Bi ton 8.31. Xc nh hm s u(), sao cho
u
_
+
2
_
=
u() +u()
2
_
, ,
+
2
Q
_
Gii. t u(1) = , u(2) = . Ta c
u(2) = u
_
3 + 1
2
_
=
u(3) +u(1)
2
.
Suy ra
u(3) = 2u(2) u(1) = 2 .
Tip tc qu trnh nh vy, ta c
u(3) = u
_
4 + 2
2
_
=
u(4) +u(2)
2
.
Suy ra
u(4) = 2u(3) u(2) = 2(2 ) = 3 2.
Bng phng php quy np, ta thu c
u() = (n 1) (n 2), n.
Vy
_
u() = ( )n + 2 , n,
u(1) = , u(2) = .
t = a + b; = 2a + b, th a = v b = 2 .
Do , nghim ca phng trnh l u() = an +b; a, b tu .
2) Php chuyn i lng trung bnh cng sang trung bnh iu ho
8.3. Hm s xc nh trn tp cc s hu t 188
Bi ton 8.32. Xc nh hm s u() Q sao cho
u
_
+
2
_
=
2u()u()
u() +u()
_
, ,
+
2
Q
_
.
Gii. Ta c
u
_
+
2
_
=
2u()u()
u() +u()
u
_
+
2
_
=
2
1
u()
+
1
u()
.
t
1
u()
= v(), th phng trnh cho tng ng vi
v
_
+
2
_
=
v() + v()
2
.
Theo Bi ton 1, v() = a + b; a, b 0, a + b > 0.
Vy nghim ca phng trnh l
u() =
1
a + b
; a, b 0; a + b > 0.
3) Php chuyn i lng trung bnh cng sang trung buh nhn
Bi ton 8.33. Xc nh hm s u() sao cho
u
_
+
2
_
=
_
u()u();
_
, ,
+
2
_
Q.
Gii. Ta c
u() = u
_
+
2
_
=
_
u()u() =
_
[u()]
2
= [u()[ 0.
t u(1) = , u(2) = ( 0, 0).
a) Nu = 0 th
u() = u
_
1 + 2 1
2
_
=
_
u(1)u(2n 1) = 0, Q.
Vy u() 0 l nghim duy nht ca phng trnh .
b) Nu > 0 v = 0 th
u() = u
_
2 + 2 2
2
_
=
_
u(2)u(2n 2) = 0, 2.
8.3. Hm s xc nh trn tp cc s hu t 189
Suy ra
u() =
_
, nu = 1
0 nu 2
l nghim ca phng trnh.
c) Xt trng hp > 0 v > 0. Gi s tn ti n
0
3 sao cho u(n
0
) = 0.
Th th
u(n
0
1) = u
_
n
0
+n
0
2
2
_
=
_
u(n
0
)u(n
0
2) = 0.
Chn n
0
= 3 th u(n
0
1) = u(2) = 0, hay = 0, mu thun.
Do , c th gi thit rng u() > 0 vi mi Q. Ta c
u(2) = u
_
3 + 1
2
_
=
_
u(3)u(1).
Suy ra
u(3) =
u
2
(2)
u(1)
=

2

.
Mt khc
u(3) = u
_
4 + 2
2
_
=
_
u(4)u(2).
Suy ra
u(4) =
u
2
(3)
u(2)
=
_

_
2

=

3

2
.
Bng phng php quy np ton hc, ta chng minh c rng
u() =

1

2
, 3.
m

2
=
_

__

_
n
.
t
_
= ab,
= ab
2
(a > 0, b > 0).
Suy ra

= a,

= b.
8.3. Hm s xc nh trn tp cc s hu t 190
Vy nghim ca phng trnh l
u() =
_
nu = 1
0 nu 2
( 0)
hoc u() = a.b

(a > 0, b > 0).


4) Php chuyn i lng trung bnh cng sang trung bnh bc hai
Bi ton 8.34. Xc nh hm s u(), sao cho
u
_
+
2
_
=
_
u
2
() +u
2
()
2
_
, ,
+
2
Q
_
.
Gii. Ta c
u() = u
_
+
2
_
=
_
u
2
() +u
2
()
2
=
_
u
2
() = [u()[ 0, Q.
t u(1) = 0 ; u(2) = 0. Ta c
u(2) = u
_
3 + 1
2
_
=
_
u
2
(3) +u
2
(1)
2
.
Suy ra
u
2
(3) = 2u
2
(2) u
2
(1) = 2
2

2
u(3) =
_
2
2

2
(

2).
Tng t
u(3) = u
_
4 + 2
2
_
=
_
u
2
(4) +u
2
(2)
2
.
Suy ra
u
2
(4) = 2u
2
(3) u
2
(2) = 2(2
2

2
)
2
= 3
2
2
2
hay
u(4) =
_
3
2
2
2
_

_
3
2
_
.
Bng quy np ton hc, ta chng minh c h thc
u() =
_
(n 1)
2
(n 2)
2
, n 3.
Nhn xt rng, ta lun c
_
( 1)
2
( 2)
2
=
_
(
2

2
) + 2
2

2
.
8.4. Phng trnh trong hm s vi cp bin t do 191
t
_

2
= a +b

2
= 2a +b.
Suy ra
_
a =
2

2
b = 2
2

2
.
Vy nghim ca phng trnh l u() =

a +b ; a 0, a + b 0.
Nhn xt 8.2. Trong c bn bi ton nu trn, nu ta thay m bi (n + 1)
v n bi (n 1) th ta c th a chng c v cc phng trnh sai phn quen
bit.
8.4 Phng trnh trong hm s vi cp bin t do
Trong mc ny, ta i tm nhng hm s thc hin php chuyn tip mt biu
thc i s ca cp ch s sang mt i lng khc ca cp phn t tng ng
ca dy s. Cc bi ton ny lin quan cht ch n vic chuyn tip cc hm
s; n s m phng cc hm s c bit trong s hc, i s,...
Bi ton 8.35. Tm hm f : Q Q tha mn cc iu kin f(1) = a Q v
f( +n) + f(m) = 2f()f(), m, Q.
Gii. Cho m = n = 0 ta c f(0) = 0 hoc f(0) = 1. Nu f(0) = 0 th thay
n = 0 ta c 2f() = 0 vi mi m Q. Do vy f() 0 v ng vi a = 0.
Nu f(0) = 1, cho m = n = 1 ta thu c f(2) = 2a
2
1.
Tip tc thay m = 2; n = 1 vo iu kin bi ra ta c f(3) = 4a
3
3a. T
ta c d on f() = T
n
(a) vi mi 1.
D on c chng minh d dng bng phng php quy np.
mt khc, cho m = 0 ta c f()+f() = 2f(0)f() = 2f() nn f() =
f(). Vy f() l hm chn. Vy ta c
f() =
_

_
1 khi m = 0,
a khi m = 1,
T
|m|
(a) khi [m[ 1, m Q.
Bi ton 8.36. Tm hm f : Q R tha mn cc iu kin f(0) ,= 0, f(1) =
5
2
v
f( + n) +f(m) = f()f(), m, Q.
8.4. Phng trnh trong hm s vi cp bin t do 192
Gii. Cho m = n = 0 ta c, do f(0) ,= 0, f(0) = 2. Tip theo, theo quy np
ta c
f() = 2

+ 2

, Q.
Th li ta thy hm ny tho mn iu kin bi ra.
Bi ton 8.37. Tm hm f : Q [0, +) tha mn cc iu kin f(1) = 1 v
f( +n) + f(m ) =
1
2
[f(2m) + f(2n)], m, Q, m n.
Gii. Cho m = n = 0 ta c f(0) = 0. Cho m = 1, = 0 th
f(1) + f(1) =
1
2
[f(2) + f(0)].
Suy ra f(2) = 4.
Chng minh bng quy np ta c f() =
2
.
Tht vy, do f(k) + f(k) =
1
2
[f(2k) + f(0)] nn c ngay f(2k) = 4k
2
.
Cng vy, do f(k + 1) +f(k 1) =
1
2
[f(2k) + f(2)] nn ta c
f(k + 1) =
1
2
f(2k) + 2 f(k 1) = (k + 1)
2
.
Bi ton 8.38. Tm cc a thc hai bin P(m, n) (m, Q) tho mn iu
kin
a) P(am, an) = a
2
P(m, n) vi mi m, n, a Q,
b) P(b + c, a) +P(c +a, b) +P(a +b, c) = 0 vi mi a, b, c Q,
c) P(1, 0) = 1.
Gii. Trong b) t b = 1 a; c = 0 ta c
P(1 a, a) = 1 P(a, 1 a). (1)
Li t c = 1 a b v kt hp vi a) ta c
P(a +b, 1 a b) = P(a, 1 a) + P(b, 1 b) + 2. (2)
t f() = P(m, 1 ) + 2. Khi f(1) = P(1, 0) + 2 = 3 v (5) tr thnh
f(m+ n) = f() + f(). l phng trnh dy chuyn i php cng
_
f(m+n) = f() +f(),
f(1) = 3.
(3)
Phng trnh (3) c nghim duy nht f() = 3n. Vy nn
P(n, 1 ) = 3n 2. (4)
8.4. Phng trnh trong hm s vi cp bin t do 193
Bng phng php quy np ta s thu c
P(a, b) = (a +b)
2
_
3
a
a +b
2
_
= (a + b)(a 2b), a, b Q.
Tm li P(, ) = ( +n)
2
(m 2n).
Bi ton 8.39. Cho a thc Chebyshev T
n
(x) = cos( arccos x). Chng minh
rng vi m, n Q; m v x R th T
n
(x) l nghim ca phng trnh dy
sau
T
n+m
(x) +T
nm
(x) = 2T
n
(x)T
m
(x).
Gii. S ng nh ngha T
n
(x) v phng php quy np hoc s dng cc cng
thc
cos(n +m)x + cos(n m)x = 2 cos nx cos mx
v
cosh(n + m)x + cosh(n m)x = 2 cosh(nx) cosh(mx),
ta c ngay iu phi chng minh.
Bi ton 8.40. Tm hm f Q Q tha mn cc iu kin
Q : < f() < Q,
f( +n) + f(m) = 2f()f() m, Q. (1)
Gii. Cho m = n = 0 ta c f(0) 0, 1. Gi s f(0) = 0. Cho = 0 trong
(1) ta c 2f() = 2f()f(0) = 0 v f 0.
Gi s f(0) = 1. Cho m = 0 trong (1) ta thu c f() = f() vi mi
Q. Vy ch cn xt Q. Cho = 1 trong (1), ta c
f( + 1) = 2f()f(1) f(m 1)
v thu c cng thc truy hi theo f(1). Nu [f(1)[ 2 th t gi thit ta c
f(2n) = 2[f()]
2
1
tng v khng gii ni, tri vi gi thit. Vy f(1) 1, 0, 1.
Vi f(1) = 1 th f() = (1)
n
(quy np).
Vi f(1) = 1 th f() 1.
Vi f(1) = 0 ta c dy tun hon (quy np)
f(4m) = 1, f(4 + 1) = 0, f(4 + 2) = 1, f(4 + 3) = 0.
Suy ra f(2) = 4. Chng minh bng quy np ta c f() =
2
. Tht vy, do
f(k) +f(k) = (1/2)[f(2k) +f(0)] nn c ngay f(2k) = 4k
2
. Cng vy, do f(k +
1) +f(k 1) = (1/2)[f(2k) + f(2)] nn ta c
f(k + 1) =
1
2
f(2k) + 2 f(k 1) = (k + 1)
2
.
8.4. Phng trnh trong hm s vi cp bin t do 194
Bi ton 8.41. K hiu
u

=
_
2
0
sin
n
xdx, Q.
Xc nh hm s f : Q R theo cng thc
f() = ( + 1)u
n
g( + 1), Q.
Gii. S dng cng thc tch phn tng phn, ta thu c
u

= cos x sin
1
x

2
0
+
_
2
0
( 1) sin
2
x cos
2
xdx
=
_
2
0
( 1) sin
2
x(1 sin
2
x)dx
= ( 1)(u
1
u
n
).
T y suy ra
g( + 2) =
+ 1
+ 2
u
n
, Q. (1)
T (1) ta nhn c
f( + 1) = ( + 2)g( + 1)g( + 2)
= ( + 2)g( + 1)
+ 1
+ 2
u

= ( + 1)g( + 1)u

= f().
Vy nn
f() = f(0) =

2
.
Bi ton 8.42. K hiu
u

=
_

0
cos

x cos nxdx, Q.
Xc nh hm s f : Q R theo cng thc
f() = 2

, Q.
8.4. Phng trnh trong hm s vi cp bin t do 195
Gii. t cos
n
x = u v cos nxdx = dv th theo cng thc tch phn tng phn,
ta thu c
u

=
1

cos
n
x sinx

0
+
_

0
cos
1
x sinx sinnxdx
=
1
2
_

0
cos
1
x[cos( 1)x + cos( + 1)x]dx
=
1
2
u
1

1
2
u

+
1
2
_

0
cos
1
x sinx + sinnxdx
=
1
2
u
1

1
2
u

+
1
2
u

.
Vy nn
u

=
1
2
u
1
=
1
4
u
2
= =
1
2
1
u
1
=
1
2
1

2
.
Bi ton 8.43. 3. Xc nh hm s u

c tnh theo cng thc


u

=
_
4
0
tan
2
xdx
Gii. Ta vit u
n
di dng sau
u

=
_
4
0
tan
2
xdx
=
_
4
0
tan
22
x[(tan
2
x + 1) 1]dx
=
_
4
0
tan
22
xd tanx u
1
=
tan
21
x
2 1

4
0
u
1
=
1
2 1
u
1
.
Do vy
u

+u
1
=
1
2 1
,
u
1
+ u
2
=
1
2 3
,

8.4. Phng trnh trong hm s vi cp bin t do 196
u
1
+u
0
=
1
2 1
.
Suy ra
u

= (1)

4
+

k=1
(1)
k
2k 1
_
.
Bi ton 8.44. Xc nh hm s f : Q R c tnh theo cng thc
f() =
1
_
0
x

1 xdx, Q.
Gii. t x

= u,

1 xdx = dv th
f() =
1
_
0
x

1 xdx
=
_

2
3
x
n
(1 x)
3
2

1
0
+
2
3
1
_
0
x
1

1 x(1 x)dx
=
2
3
1
_
0
(x
1
x
n
)

1 xdx
=
2
3
1
_
0
(x
1

1 xdx
2
3
1
_
0
x

1 xdx
2
3
x
1

2
3
f().
Vy nn
f() =
2
2 3
x
1
.
V f(0) =
2
3
nn ta c ngay
f() =
1
_
0
x

1 xdx = 2
(2n)!!
(2 + 3)!!
, Q.
Bi ton 8.45. Xc nh hm f : Q Q tho mn cc iu kin
f(0) = 1, f(f() = f(f( + 2) + 2) = Q.
8.4. Phng trnh trong hm s vi cp bin t do 197
Gii. Nhn xt rng f l nh x 1-1
f() = f() f(f()) = f(f()) m = n.
Vy nn
f() = f( + 2) + 2 Q.
Suy ra
f( + 2) = f() 2, f(0) = 1, f(1) = f(f(0)) = 0.
Vy nn
f(2) = f(0) 2 = 1,
f(3) = f(1) 2 = 2,
f() = ( 1).
Tng t
f(1) = f(1) + 2 = 2,
f(2) = f(0) + 2 = 3,
f(3) = f(1) + 2 = 5,
f() = ( 1).
Bi ton 8.46. Cho gc vi 0 < < . Xc nh cp s a, b sao cho dy hm
P
n
(x) c tnh theo cng thc
P
n
(x) = x

sin x sin() + sin( 1)


lun lun chia ht cho f(x) = x
2
+ax +b.
Gii. Vi = 3 th
P
3
(x) = x
3
sin x sin(3) + sin 2 = sin (x + 2 cos )(x
2
2x cos + 1).
T suy ra vi f(x) = x
2
+ 2x cos + 1 th P
3
(x)
.
.
.f(x). Vi 3 th
P
+1
(x) = xP
n
(x) + (x
2
2x cos + 1) sin.
Suy ra f(x) = x
2
+ 2x cos + 1.
8.5. S dng gii hn gii phng trnh hm 198
Bi tp
Bi 1. Xc nh hm s f : Q R nu bit: f(1) = a, f( +n) = f() +f();
Bi 2. Xc nh hm s f : Q R nu bit: f(1) = a, f(m) = f() +f()
(, , m Q).
Bi 3. Xc nh hm s f : Q R nu bit f
_

_
= f() +f() (, ,
m

Q).
Bi 4. Xc nh hm s f : Q R tho mn iu kin f
_

_
= f() f()
(, ,
m

Q).
Bi 5. Xc nh hm s f : Q R tho mn iu kin f( + n) = f()f()
(m, Q).
Bi 6. Xc nh hm s f : Q R nu bit f(+n)+f(m) =
1
2
(f(2m)+f(2n)
, (, , m Q).
Bi 7. Xc nh hm s f : Q R
+
tho mn iu kin f( + ) =
f()
f()
(m, Q).
Bi 8. Xc nh hm s f : Q R
+
tho mn iu kin f
_
+
2
_
=
_
f()f()
(, ,
+
2
Q).
Bi 9 . Xc nh hm s f : Q R
+
tho mn iu kin x+
2
=
2f()f()
f()+f()
(, ,
+
2
Q).
Bi 10 . Xc nh hm s f : Q R
+
tho mn iu kin f
_
+
2
_
=
_
f()
2
+f()
2
2
(, ,
+
2
Q).
Bi 11. Xc nh cc hm f : Q Q tho mn cc iu kin f(1) = 2 v
f(xy) = f(x)f(y) f(x + y) + 1, x, y Q.
8.5 S dng gii hn gii phng trnh hm
Mt trong nhng tnh cht cn thit s dng gii hn l tnh lin tc ca
hm s. Khi s dng gii hn gii phng trnh hm ngi ta thng lm nh
sau.
1. Xy dng mt ng thc ng vi mi gi tr ca n sau ly gii hn hai
v nh s dng tnh cht lin tc ca hm s.
2. Tnh lin tc khng c tc dng i vi phng trnh hm trong tp hu t
Q. Tuy nhin nu bit chc chn l hm lin tuc, ta c th thit lp cng
thc cho hm trong Q v suy ra cng thc phi tm tng t trong tp R.
Bi ton 8.47. Tm tt c cc hm f : R R lin tc, tho mn iu kin
f(x + y) = f(x) +f(y), x, y R.
8.5. S dng gii hn gii phng trnh hm 199
Gii. Cho x = y = 0, suy ra f(0) = 2f(0, suy ra f(0) = 0. Cho x = y = 1, th
c f(2) = 2f(1). Cho x = 2, y = 1 th c f(3) = f(2) + f(1) = 3f(1). Quy
np ta c f(n) = nf(1), vi mi n N

.
K hiu f(1) = a, suy ra f(n) = na, vi mi n N

. Cho x = n, y = n
ta c 0 = f(0) = f(n) + f(n). Suy ra f(n) = f(n), f(n) = a(n), v
f(n) = an vi mi n Z.
t x = y, ta c f(2x) = 2f(x), f(3x) f(2x +x) = 2f(x) +f(x), f(3x) =
3f(x). Suy ra
f(mx) = mf(x), m N, x R.
T
an = f(n) = f
_
m
n
m
_
= mf
_
n
m
_
suy ra f(n/m) = an/m. Suy ra f(x) = ax, vi mi x Q. T suy ra vi mi
x R lun tn ti x
n

1
, x
n
Q sao cho lim
n
x
n
= x. Ta c
f(x
n
) = ax
n
.
Ly gii hn ta c
lim
n
f(x
n
) = lim
n
(ax
n
).
T f(x) = ax, vi mi x R.
Bi ton 8.48. Tm tt c cc hm lin tc f : R R tho mn iu kin
f(x + y) + f(x y) = 2f(x) + 2f(y).
Gii. Cho x = y = 0, suy ra f(0) = 0. Cho x = 0 suy ra f(y) +f(y) = 2f(y).
Do f(y) = f(y), tc l f(x) l hm s chn trn R.
K hiu f(1) = a. t x = y = 1, suy ra f(2) + f(0) = 4f(1) = 4a. T
f(2) = 4a. t x = 2, y = 1 suy ra f(3) + f(1) = 2f(2) + 2f(1), f(3) =
2f(2) + f(1) = 9a.
Ta chng minh quy np f(n) = an
2
. Ta gi s f(n) = an
2
, phi chng minh
f(n+1) = a(n+1)
2
. Cho x = n, y = 1, ta c f(n+1)+f(n1) = 2f(n)+2f(1).
Suy ra f(n + 1) = 2an
2
a(n 1)
2
+ 2a = a(2n
2
(n 1)
2
+ 2). Tip tc khai
trin cho ta
f(n + 1) = a(n + 1)
2
.
Do f l hm s chn nn f(n) = f(n) = an
2
= a(n)
2
. Ta c f(n) = an
2
vi
mi n Z.
By gi ta chng minh cng thc f(nx) = n
2
f(x), vi mi n N. Vi x = y
ta c f(2x) + f(0) = 2f(x) + 2f(x) = 2
2
f(x). Suy ra f(2x) = 2
2
f(x).
8.5. S dng gii hn gii phng trnh hm 200
Gi s f(nx) = n
2
f(x), ta phi chng minh f((n+1)x) = (n+1)
2
f(x). Tht
vy f((n + 1)x) + f((n 1)x) = 2f(nx) + 2f(x). Suy ra
f((n + 1)x) = (n 1)
2
f(x) + 2n
2
f(x) + 2f(x)
T y tip tc khai trin cho ta f((n + 1)x) = (n + 1)
2
f(x). Khi an
2
=
f(n) = f(m.
n
m
) = m
2
f(n/m). Suy ra f(n/m) = a(n/m)
2
, vi mi m, n N. Suy
ra ta c f(x) = ax
2
, vi mi x Q.
Vy vi mi x R lun tn ti x
n

n
, x
n
Q sao cho lim
n
x
n
= x. T
f(x
n
) = ax
2
n
, hay l
lim
n
f(x
n
) = lim
n
ax
2
n
.
T f(x) = ax
2
, vi mi x R.
Gii (2). Ta c f(0) = 0, cho x = y ta c f(2x) = 4f(x). Suy ra
f(2x)
(2x)
2
=
f(x)
x
2
, x R 0.
Suy ra
g(2x) = g(x), g(x) =
f(x)
x
2
. (8.1)
Suy ra
g(x) = g
_
x
2
_
= g
_
x
2
2
_
= = g
_
x
2
n
_
,
Do f lin tc nn g lin tc trn R0. Suy ra g(x) = lim
n
g(x/2
n
) = g(0) = a.
Suy ra
g(x) = a, x R 0.
Vy f(x) = ax
2
, vi mi x R 0. Do f(0) = 0 nn f(x) = ax
2
, vi mi
x R.
Bi ton 8.49. Gi s f : R R v tha mn cc iu kin
1. f(1) = 1,
2. f(x +y) = f(x) + f(y), x, y R,
3. f(x).f(1/x) = 1, x ,= 0.
Gii. Ta chng t rng hm cn tm l f(x) = x, vi mi x R. Theo bi ton
(8.47), t iu kin th nht v th hai ta suy ra f(x) = x, vi mi x Q.
By gi ta chng t f(x l hm lin tc. Cn chng minh lim
h
f(x+h) = f(x)
hay l lim
h0
(f(x) +f(h)) = f(x). Tc l cn chng t lim
h0
f(h) = 0.
8.5. S dng gii hn gii phng trnh hm 201
Nu a, b > 0 th [a+b[ = [a[ +[b[, t iu kin th ba suy ra rng vi x ,= 0 ta
c f(x) v f(1/x) cng du. Ch rng theo bt ng thc gia trung bnh cng
v trung bnh nhn, ta c

f
_
x +
1
x
_

f(x) +f
_
1
x
_

= [f(x)[ +

f
_
1
x
_

2
_
f(x).f
_
1
x
_
= 2.
Suy ra
f(y) 2 vi mi y 2. (8.2)
Vy nu [y[
1
2
hay [1/y[ 2 th theo (8.2) ta c f(1/y) 2.
T iu kin th ba ta c
1 = [f(y).f(1/y)[ [f(y)[.2.
Suy ra
[f(y)[
1
2
. (8.3)
Nu [y[
1
4
, suy ra [2y[
1
2
, suy ra [f(2y)[
1
2
. Do
[f(y)[
1
4
. (8.4)
Bng quy np, ta chng minh c
[f(y)[
1
2
n
, vi mi [y[
1
2
n1
.
Tht vy, gi s [f(y)[
1
2
n1
vi [y[ 1/2
n1
. Khi nu [y[ 1/2
n
th theo
gi thit quy np ta c
1
2
n1
[2y[.
Suy ra 1/2
n1
[f(2y)[, hay l

1
2
n1

2f(y).
Tc l [f(y)[ 1/2
n
vi [y[ 1/2
n
. Vy lim
h0
f(h) = 0, suy ra f(x) l hm lin
tc trn R, m f(x) = x, vi mi x Q. Vy
f(x) = x, vi mi x R.
8.5. S dng gii hn gii phng trnh hm 202
Bi ton 8.50. Cho f : R R lin tc tha mn iu kin f(1) = 1, f(x+y) +
f(xy) = f(x) +f(y) +f(x).f(y).
(a) Chng minh rng f(x) cng tnh: f(x + y) = f(x) +f(y).
(b) Hy tm tt c cc hm f tha mn iu kin
f(x +y) + f(xy) = f(x) + f(y) + f(x).f(y).
Gii. Cu (a). Cho y = 1, th ta c
f(x + 1) = f(x) + 1. (8.5)
Thay x bi x + 1, ta c
f(x + 1 +y) +f((x + 1)y) = f(x + 1) +f(y) +f(x + 1)f(y).
V f(x + 1) = f(x) + 1 nn ta c
f(x +y) + 1 + f(xy +y) = f(x) + 1 +f(y) +f(x + 1).f(y)
= f(x) +f(y) +f(x)f(y) + 1 + f(y)
= f(x + y) + f(xy) + f(y) + 1.
T suy ra
f(xy +y) = f(xy) + f(y) (8.6)
Vy vi u, v bt k, tn ti x sao cho u = vx, suy ra x =
u
v
,= 0. Suy ra
f(u +v) = f(vx +v) = f(vx) + f(v) = f(u) + f(v). (8.7)
Nu v = 0 th f(u) = f(u) + f(0), suy ra f(0) = 0. Thay x = y = 0 vo
f(x + y) + f(xy) = f(x) + f(y) + f(x).f(y),
ta c
f(0) +f(0) = f(0) +f(0) + f(0).f(0).
Suy ra
f(0) = 0 (8.8)
T (8.7) v (8.8) suy ra f cng tnh.
Cu (b). T cu (a) suy ra f(x + y) = f(x) + f(y), v f(xy) = f(x).f(y).
Tc l hm f(x) va cng tnh va nhn tnh. Suy ra f(x) = x, x =
m
n
, thnh
8.5. S dng gii hn gii phng trnh hm 203
th x Q. Suy ra vi mi x R, lun tn ti x
n
Q : x
n
x khi n . Do
fx) lin tc trn R nn lim
n
f(x
n
) = f(x), hay l x = f(x).
Gi s hm s khng gim. Cho x = y, ta c f(x
2
) = f
2
(x) 0, suy ra
f(u) 0 vi mi u 0. Nu x > y th f(x) = f(y +x y) = f(y) + f(x y)
f(y), v f(x y) 0 vi x y 0.
Vi mi x R Q, tn ti r
n
: r
n
Q sao cho
r
n
> x : lim
n
r
n
= x. (8.9)
Tn ti s
n
: s
n
Q sao cho
s
n
< x : lim
n
s
n
= x. (8.10)
Suy ra s
n
< x < r
n
. Do f l hm s khng gim nn f(s
n
) f(x) f(r
n
), hay
l
s
n
f(x) r
n
. (8.11)
Ly gii hn cho ta
lim
n
s
n
f(x) lim
n
r
n
.
T (8.9), (8.10) v (8.11), theo nguyn l kp ta c x f(x) x. Suy ra
f(x) = x. Cho x = y = 1, t ta c f(2) = 2. Thay x = x 1 v y = y 1 ta
c
f(x + y) + f((x y)(y + 1)) = f(x 1) + f(y + 1) +f(x 1).f(y + 1). (8.12)
Cho y = 1, ta c
f(x + 1) +f(x) = f(x) +f(1) +f(x).f(1).
ng thc ny tng ng vi
f(x + 1) = f(x) + 1. (8.13)
Cho x = 2, t (8.12) cho ta
f(2 + y) +f(y + 1) = f(1) +f(y + 1) +f(1).f(y + 1).
V (8.13) nn t y ta c
f(2 +y) = f(1)(1 + f(y + 1)) = 1 + f(y + 1),
hay
f(2 +y) = f(2) +f(y). (8.14)
8.5. S dng gii hn gii phng trnh hm 204
Cho x = 2, t ng thc
f(x + y) + f(xy) = f(x) + f(y) + f(x).f(y),
ta c
f(2 + y) +f(2y) = f(2) + f(y) + f(2).f(y).
Phng trnh ny tng ng vi
f(2y) = 2f(y), (8.15)
hay l
f(2y)
2y
=
f(y)
y
. (8.16)
t g(x) = f(x)/x, x ,= 0, ta c (8.16) tng ng vi
g(x) = g(2x).
Suy ra
g(x) = g
_
x
2
_
= g
_
x
4
_
= = g
_
x
2
n
_
. (8.17)
Suy ra tn ti (x
n
)

0
vi mi x R sao cho x
n
x, g(x) l hm s lin tc trn
R 0. Thnh ra, lim
n
g(x
n
) = g(x). Tc l
lim
n
g
_
1
2
n
_
= g(x).
V (8.17) nn t y ta c g(1) = g(x), hay
f(1)
1
= g(x), f(1) = g(x) = f(x)/x.
Cui cng ta c f(x) = x.
Suy ra f cng tnh.
Ch rng trong (b) nu cn xt ring x Q th ta c
f(n) = f(1 + 1 + + 1
. .
n s 1
) = nf(1) = n.
Ta c
f(n.
m
n
) = f(n).f(m/n)
tng ng vi mi
f(m) = n.f(n/m)
f(m) = nf(m/n)
Suy ra
f
_
m
n
_
=
f(m)
n
=
mf(1)
n
=
m
n
.
Vy f(m/n) = m/n, hay f(x) = x, vi mi x Q.
8.5. S dng gii hn gii phng trnh hm 205
Bi ton 8.51. Tm hm f(x) xc nh R

(0 x < +) tha mn cc
iu kin
1. f(x.f(y)).f(y) = f(x + y) x, y 0,
2. f(2) = 0
3. f(x) ,= 0 x [0, 2).
Gii. Cho y = 2 th t iu kin th nht ta c f(x.f(2)).f(2) = f(x + 2). T
iu kin th hai suy ra f(x + 2) = 0 vi mi x 0. V iu kin th ba nn
f(x) ,= 0 vi mi x [0, 2), suy ra t = x + 2 2, suy ra f(t) = 0 vi mi t 2.
Suy ra
f(x) = 0, x 2. (8.18)
Vy
f(x) =
_
0 nu x [2, +)
,= 0 nu x [0, 2)
(8.19)
Ta ch cn tm hm f(x) vi 0 x 2 th f((2 x).f(x)).f(x) = f(2 x+x) =
f(2) = 0.
Suy ra f((2 x).f(x)) = 0. Kt hp vi (8.18) ta c (2 x)f(x) 2. Suy ra
1/f(x) ,= 0 v f(x) 2/(2 x). Do
1
f(x)

2 x
2
. (8.20)
Mt khc, f((y x).f(x)) ,= 0, nn (y x).f(x) < 2 theo (8.19). Ta c nh x
v cho y 2, do tnh lin tc ca f nn (y x)f(x) (2 x)f(x) 2. Vi
2 x > 0 th f(x) 2/(2 x). Suy ra
1
f(x)

2 x
x
. (8.21)
T (8.20) v (8.21) suy ra
2 x
2

1
f(x)

2 x
x
.
Do f(x) = 2/(2 x). Tm li
f(x) =
_
2
2x
khi 0 x < 2
0 khi x 2
8.5. S dng gii hn gii phng trnh hm 206
Cch 2. Khi 0 x < 2, ta c 2 x > 0,
f((2 x).f(x)).f(x) = f(2 x + x) = f(2) = 0.
Do f(x) ,= 0 nn f((2 x)f(x)) = 0, suy ra (2 x)f(x) 2. Thnh th,
f(x)
2
2 x
(8.22)
Mt khc, f((y x)f(x)) ,= 0 nn (y x)f(x) < 2. Ta c nh x v cho y 2,
do tnh lin tc ca f nn (y x)f(x) (2 x)f(x) 2. Suy ra
f(x)
2
2 x
. (8.23)
T (8.22) v (8.23) suy ra f(x) =
2
2x
. Tm li
f(x) =
_
2
2x
khi 0 x < 2
0 khi x 2
Bi ton 8.52. Tm hm f(x) xc nh v lin tc vi mi x > 0 v tho mn
iu kin
1. f(uv) = f(v) + f(u), u, v > 0
2. lim
x1
f(x) = 0,
3. f(x) ,= 0, x > 0.
Gii. Gi s f(x) , 0, vi mi x > 0.
f(x) = f(1.x) = f(1) +f(x),
suy ra f(1) = 0. Ly x > 0, x
n
x
0
khi n . Suy ra x
n
/x
0
1 khi n .
Suy ra
limf
_
x
n
x
0
_
= 0.
Vy f(x
n
) = f(x
0
,
xn
x
0
) = f(x
0
) + f(
xn
x
0
) f(x
0
) + 0. Tc l
lim
xnx
0
f(x
n
) = f(x
0
).
f(uv) = f(u) + f(v), u, v > 0. (8.24)
8.5. S dng gii hn gii phng trnh hm 207
Suy ra f(x
n
) = nf(x), vi x > 0, n N

. Do f(1) = 0 nn f(x
n
) = nf(x)
ng khi n = 0. Hn na, vi mi n N

th 0 = f(1) = f(x
n
, x
n
) = f(x
n
) +
f(x
1
) = nf(x) + f(x
n
). Suy ra f(x
n
) = nf(x), v
f
_
x
m
n
_
= f
__
x
1
n
_
m
_
= mf(x
1/n
) = m
1
n
f(x) =
m
n
f(x).
Suy ra f(x
m/n
) =
m
n
f(x), m, n Z. Do
f(x
r
) = rf(x), x > 0 r Q.
c bit f(2
r
) = r.f(2) = r.A, r Q, A = f(2). Nu x > 0 th x = 2
log
2
x
suy
ra vi mi x > 0 th tn ti mt dy s hu t r
n

0
sao cho
lim
r
r
n
= log
2
x.
Suy ra
lim
r
2
rn
= 2
log
2
x
= x
dn n lim
n
f(2
rn
) = f(x), m f(2
rn
) = A.r
n
A. log
2
x khi n .
Suy ra
lim
n
f(2
rn
) = A. log
2
x = f(x).
Do f(x) , 0 vi x > 0 nn vi mi x > 0 ta c A ,= 0, t A = log
a
2, a > 0 v
a ,= 1, f(x) Alog
2
x = log
a
2. log
2
x = log
a
x. Vy
f(x) = log
a
x, x > 0, 0 < a ,= 1.
Bi ton 8.53. Tm f(x) xc nh v lin tc vi mi x > 0 v tho mn iu
kin
1. f(uv) = f(u).f(v), u, v > 0
2. lim
x1
f(x) = 1.
Gii. Ta c vi mi x > 0
f(x) = f(

x.

x) = f
2
(

x) 0.
Suy ra f(x) x, x > 0.
Nu tn ti x
0
> 0 f(x
0
) = 0 th vi mi x > 0 ta c f(x) = f(x
0
.
x
x
0
) =
f(x
0
).f
_
x
x
0
_
= 0. Theo gi thit ta cng c lim
x1
f(x) = 1. Vy f(x) > 0, vi mi
x > 0.
8.5. S dng gii hn gii phng trnh hm 208
Xt hm s g(x) = limf(x), vi mi x > 0 tho mn iu kin
g(u, v) = lnf(uv) = ln(f(u).f(v)) =
= lnf(u) + ln f(v)
= g(u) +g(v)
v lim
x1
g(x) = 0.
Vy g(x) tha mn iu kin bi ton 8.52 trn. Suy ra
g(x) = log
a
x, 0 < a ,= 1, x > 0.
Suy ra lnf(x) = log

x = log

e. lnx. Do
f(x) = x

vi = log
a
e, (0 < a ,= 1)
Bi ton 8.54. Cho f : (1, 1) R lin tc v
f(x) = f
_
2x
1 +x
2
_
, x [1, 1].
Chng minh rng f(x) l hm hng.
Gii. Ta c nh x, xt dy s (x
n
)

1
xc nh bi
x
0
= x > 0, x
n+1
=
2x
1 + x
2
. (8.25)
Dy (x
n
) l dy s tng. Suy ra x
n+1
x
n
, hay 2x
n
/(1 +x
2
n
) x
n
. T y ta c
1 x
n
1. Theo bt ng thc gia trung bnh cng v trung bnh nhn, ta c
x
n+1
=
2x
n
1 + x
2
n
1.
Do dy s (x
n
) tng v b chn di bi 1 nn tn ti gii hn lim
n
x
n
= l > 0.
Ta c (8.25) tng ng vi
l =
2l
1 + l
2
,
t ta c l = 1.
Dy s
f(x
n+1
) = f
_
2x
n
1 + x
2
n
_
= f(x
n
).
Ly f(x) = f(x
0
) = = f(x
n
). Do f(x) lin tc trn [1, 1] nn lim
n
f(x
n
)
= f(1), hay f(x) = f(1) = c, vi c l hng s.
(trng hp x
0
= x < 0 xt tng t)
8.5. S dng gii hn gii phng trnh hm 209
Bi ton 8.55. Tm tt c cc hm f(x) xc nh v lin tc trn R 0 v
tha mn iu kin sau
(f(x
2
) x
2
)(f(x) x) =
1
x
3
, x ,= 0. (8.26)
Gii. Phng trnh (8.26) tng ng vi
(x
2
f(x
2
) x
4
)(xf(x) x
2
) = 1.
t xf(x) x
2
= g(x), ta thu c
g(x
2
)g(x) = 1. (8.27)
Suy ra g(x) ,= 0, vi mi x R. Ta c g(0) = 1 v g(1) = 1. Thay x bi x
vo trong (8.27) ta thu c
g(x
2
)g(x) = 1 = g(x
2
)g(x).
Suy ra g(x) = g(x) trn tp i xng qua gc to R. Suy ra g(x) l hm s
chn trn R, nn ta ch cn xt g(x) trn tp x 0 l .
Xt 0 x 1, ta c
g(x) =
1
g(x
2
)
=
1
1
g(x
4
)
= g(x
4
).
Suy ra g(x) = g(x
4
).
Li c
g(x
4
) =
1
g((x
4
)
2
)
=
1
1
g((x
4
)
2
)
= g((x
4
)
4
) = g(x
4
2
).
Suy ra g(x
4
) = g(x
4
2
). Vy ta thu c
g(x) = g(x
4
) = g((x
4
)
4
) = = g(( (x
4
)
4
)
4
)
4
) = g(x
4
n
), n +.
Qua gii hn ta thu c
g(x) = lim
n+
g(x
(1/4)
n
),
do lim
0 x 1
x
4
n
= 0. Suy ra
g(x) = lim
n+
g(x
4
n
) = g(0),
m g(0) = 1. Suy ra g(x) = 1 = c.
8.5. S dng gii hn gii phng trnh hm 210
Xt x > 1, ta c
g(x) =
1
g(x
1
2
)
=
1
1
g(x
1
4 )
= g(x
1
4
) = = g(x
(
1
4
)
n
).
Qua gii hn, ta thu c g(x) = lim
n+
x
(
1
4
)
n
= g(1) v vi x > 1 th lim
n+
=
1, suy ra g(x) = g(1) = 1 = c.
Vy c = xf(x) x
2
, hay f(x) = c/x + x, c l mt hng s.
Bi ton 8.56. Cho f : (1, 1) R lin tc tho mn iu kin
f(x) = f
_
2x
1 +x
2
_
, x (1, 1). (8.28)
Chng minh rng f(x) l hm hng.
Gii. Xt 0 < x < 1. Ta c nh x, xt dy s (x
n
)
+
1
nh sau
x
0
= x, x
n+1
=
1
_
1 x
2
n
x
n
. (8.29)
Dy ny c suy ra t vic xt dy s
x
n
=
2x
n
+ 1
1 +x
2
n+1
.
Ta chng minh rng (x
n
)

1
xc nh vi mi n v
lim
n
x
n
= 0. (8.30)
T (8.28) suy ra f(x) = f(x
0
) = f(x
1
) = = f(x
n
). Do f(x) lin tc trn
(1, 1) nn f(x) = lim
n
f(x
n
) = f(0).
Ta chng minh dy s (x
n
)

1
b chn.
D thy (x
n
)

1
lun dng vi mi n N

. Ta chng minh x
n
1, vi mi
n N

.
Nu n = 0 th x
0
= x < 1, ng theo gi thit.
Gi s x
k
< 1, ta c
x
k+1
=
1
_
1 x
2
n
x
k
1.
Bt ng thc ny tng ng vi 1x
k

_
1 x
2
k
. T y ta c x
k
(x
k
1) 0,
iu ny lun ng vi x
k
< 1.
8.5. S dng gii hn gii phng trnh hm 211
Suy ra (x
n
)

1
l dy s gim.
By gi ta chng minh dy s (x
n
)

1
b chn bi s 0. Tht vt, v (x
n
)

1
l
dy s gim nn tn ti lim
n
x
n
= c (ta i chng minh c = 0).
Suy ra
lim
n
x
n
= lim
n
2x
n
+ 1
1 + x
2
n+1
= lim
n
2x
n
1 + x
2
n
.
T c = c/(1 + c
2
), suy ra c = 0 hoc c = 1. Do dy s gim nn c = 0, suy
ra lim
n
x
n
= 0. Vy dy s (x
n
)

1
gim v b chn bi 0 vy dy s (x
n
)

1
b
chn.
Ta c
f(x
n
) = f
_
2x
n+1
1 +x
2
n+1
_
= f(x
n+1
).
Suy ra f(x) = f(x
n+1
) = f(x
n
) = = f(x
1
) = f(0). Do f(x) lin tc trn
(1, 1) nn lim
n
f(x
n
) = f(0), hay l f(x) = f(0) = c vi c l mt hng s.
Trng hp 1 < x < 0. Ta chng minh dy s (x
n
)

1
n iu tng v b
chn bi s 0.
Nhn xt. Bi ton 8.54 v 8.56 khc nhau c bn iu kin nn bi 8.54
xt dy s x
n+1
=
2xn
1+x
2
n
, bi ton 8.56 xt dy s x
n
=
2xn+1
1+x
2
n
sao cho dy s
khng th bng 1 c, suy ra x
n
=
1

1x
2
n
xn
.
Bi ton 8.57. Cho f : R R lin tc, 0 < c <
1
4
. Gi s f tho mn iu
kin f(x) = f(x
2
+ c), x R. Chng minh rng f(x) l hm hng.
Gii. Nhn xt rng f l hm s chn trn R. Suy ra, ta ch cn xt x 0. Xt
x
2
+ c = x, tc vi 0 < c <
1
4
th tn ti hai nghim phn bit , ca phng
trnh
x
2
x +c = 0. (8.31)
Trng hp 1. Xt x [0, ]. C nh x
0
, xt dy s
x
0
= x, x
n+1
= x
2
n
+c. (8.32)
Ta c
f(x
n+1
) = f(x
2
n
+c) = f(x
n
), 0, 1, 2, ..., .
Suy ra
f(x) = f(x
1
) = f(x
2
) = = f(x
n
),
v f(x) lin tc trn R. Suy ra lim
n
f(x
n
) = f(x).
8.5. S dng gii hn gii phng trnh hm 212
Ta chng minh (x
n
)

1
: lim
n
x
n
= . Suy ra limf(x
n
) = f() v f(x) lin
tc trn R, hay l f(x) = f(), vi mi x [0, ], hay f(x) = c, vi c l mt
hng s.
a) Chng minh lim
n
= vi (x
n
)

1
xc nh bi (8.32).
Ta c (x
n
)

1
l dy s tng. Xt g(x) = x
2
+c, g

(x) = 2x > 0 vi x [0, ].


Suy ra g(x) ng bin trn [0, ]. Do , g(x
1
) > g(x
0
), tng t vi x
2
> x
1
ta
c g(x
2
) > g(x
1
). Vy (x
n
)

1
l dy s tng.
b) Chng minh (x
n
)

1
b chn bi (bng phng php quy np). Vi x
0
=
x < . Gi s (8.32) ng vi n = k: x
k
< . Suy ra x
k+1
= x
2
k
+c <
2
+c =
v l nghim ca (8.31). Suy ra (8.32) ng vi n = k + 1.
T a) v b) suy ra lim
n
x
n
= .
Trng hp 2. Xt x [, ], xt dy s
x
0
= x, x
n+1
= x
2
n
+c. (8.33)
Chng minh tng t nh trng hp 1, (x
n
)

1
l dy s gim., x
n
, suy ra
lim
n
x
n
= .
Suy ra
f(x) = f(), x [, ].
Trng hp 3. x [, +), xt dy s xc nh bi
x = x
0
, x
n+1
=

x
n
c vi x
n
= x
2
n+1
+ c. (8.34)
Chng minh rng lim
n
x
n
= .
Xt g(x) =

x c. Tnh o hm cho ta
g

(x) =
1
2

x c
> 0, vi x [, +).
Suy ra g(x) ng bin trn [, +).
Ta c x
1
=

x
0
c < x
0
, hay x
2
0
x
0
+ c > 0 lun ng do x
0
[, +).
Nu x
1
< x
0
th dy s (x
n
)

1
gim.
Ta chng minh (x
n
)

1
b chn di bi bng phng php quy np. Nu
x
0
= x , gi s x
k
> , x
k+1
=

x
k
+ c

c = . iu ny lun ng
v l mt nghim ca
2
+c = 0. Suy ra tn ti lim
n
x
n
= l (l ). T
lim
n
x
n
= . (8.35)
T dy s (8.32) suy ra f(x
n
) = f(x
2
n+1
+ c) = f(x
n+1
). Ly
f(x
0
) = f(x
1
) = = f(x
n
).
8.5. S dng gii hn gii phng trnh hm 213
f(x) lin tc trn [, +). Do
lim
n
f(x
n
) = f(),
hay l f(x) = f() = c, vi c l hng s.
Php chng minh hon tt.
Bi ton 8.58. Chng minh rng khng tn ti hm s f : (0, ) (0, +)
tho mn iu kin
1. f(x) l hm s tng: x > y > 0 th f(x) > f(y).
2. f
_
x +
1
f(x)
_
2f(x).
Gii. Gi s f(x) tn ti, ta c nh x
0
> 0. Xt dy s xc nh bi
x
0
= x (8.36)
x
n+1
= x
n
+
1
f(x
n
)
. (8.37)
iu kin th nht trong bi cho ta
f(x
n+1
) 2f(x
n
) 2(2f(x
n1
)) = 2
2
f(x
n1
) 2
2
.2f(x
n2
)
2
n
f(x
1
) 2
n+1
f(x
0
).
Vy ta c
f(x
k
) 2
k
f(x
0
) vi mi k = 0, 1, 2, ... (8.38)
x
1
= x
0
+
1
f(x
0
)
,
x
2
= x
1
+
1
f(x
1
)
,
x
3
= x
2
+
1
f(x
2
)
,
...
x
n
= x
n1
+
1
f(x
n1
)
,
Suy ra
x
n
= x
0
+
n

k=0
1
f(x
k
)
.
8.5. S dng gii hn gii phng trnh hm 214
T (8.38) suy ra
1
f(x
k
)

1
2
k
f(x
0
)
.
Suy ra
x
n
x
0
+
n

k=0
1
2
k
f(x
0
)
= x
0
+
1
f(x
0
)
.
n

k=0
1
2
k
= x
0
+
2
f(x
0
)
.
V
n

k=0
1
2
k
=
1
1
1
2
nn
x
n
a
_
= x
0
+
2
f(x
0
)
_
, n.
Do f l hm s tng nn 2
n
f(x
0
) f(x
0
) < f(a), vi mi a. T y v (8.38)
ta suy ra f(a) > 2
n
f(x
0
) vi mi n. Suy ra f(a) > , mu thun vi gi thit
rng f(x) tn ti. iu mu thun ny cho ta iu phi chng minh.
Bi ton 8.59. Tm tt c cc hm f : R R lin tc ti x = 0 v tho mn
iu kin nf(nx) = f(x) + nx, trong n > 1 l s t nhin c nh no .
Gii. Cho n = 0, t thay gi tr ny vo biu thc cho, ta c nf(0) =
f(0) + 0, hay (n 1)f(0) = 0. Suy ra f(0) = 0, v n > 1. Cng t biu thc
cho, thay x bi x/n th
nf
_
n
x
n
_
= f
_
x
n
_
+n
x
n
,
hay
nf(x) = f
_
x
n
_
+x.
Suy ra
f(x) =
1
n
.f
_
x
n
_
+
x
n
. (8.39)
Trong (8.39) thay x bi x/n, ta c
f
_
x
n
_
=
1
n
_
x
n
2
_
+
x
n
2
.
Suy ra
f(x) =
1
n
_
1
n
f
_
x
n
2
_
+
x
n
2
_
+
x
n
=
1
n
2
f
_
x
n
2
_
+
x
n
3
+
x
n
. (8.40)
8.5. S dng gii hn gii phng trnh hm 215
Trong (8.39) li thay x bi x/n
2
th ta c
f
_
x
n
2
_
=
1
n
f
_
x
n
3
_
+
x
n
3
. (8.41)
T (8.40) suy ra
f(x) =
1
n
3
f
_
x
n
3
_
+ fracxn
5
+
x
n
3
+
x
n
.
T , ta c th chng minh quy np theo k rng
f(x) =
1
n
k
f
_
x
n
k
_
+
x
n
2k1
+
x
n
2k3
+ +
x
n
. (8.42)
Ta c
S
k
=
x
n
2k1
+
x
n
2k3
+ +
x
n
l tng cp s nhn hu han. Suy ra
S
k
=
x.
1
n
1
1
n
2
=
nx
n
2
1
.
Suy ra
lim
k
S
k
=
nx
n
2
+ 1
,
v
lim
k
1
n
k
f
_
x
n
k
_
= 0.f(0),
v f(x) lin tc ti x = 0 suy ra f(x) = nx/(n
2
1).
Th li, ta c kt qu ng. Vy
f(x) =
nx
n
2
1
, n > 1, n N

, x R.
Bi tp
Bi 8.1. Tm tt c cc hm f xc nh v lin tc trn R tha mn iu kin
f(x
3
) x
2
f(x) =
1
x
3
x, x ,= 0.
Bi 8.2. Gi s f : R R lin tc v f(x+y).f(xy) = f
2
(x) vi mi x, y R.
Chng minh rng f = 0 hoc f khng c khng im.
8.5. S dng gii hn gii phng trnh hm 216
Bi 8.3. Gi s f : R R lin tc v f(x y).f(x + y) = f
2
(x).f
2
(y), vi mi
x, y R. Chng minh rng f(x) = 0 hoc f(x) khng c khng im.
Bi 8.4. Tm hm lin tc tho mn
f(x +y) = f(x) +f(y) +ax
2
+ bxy +cy
2
, x, y R.
Bi 8.5. Tm tt c cc hm f : R R lin tc v tha mn iu kin
f(x
2
) +f(x) = x
2
+ x, x R.
Bi 8.6. Gi s a R, f(x) lin tc trn [0.1] tho mn iu kin
1. f(0) = 0
2. f(1) = 1
3. f(
x+y
2
) = (1 a)f(x) + af(y), vi mi x, y [0, 1], x slanty.
Tm cc gi tr c th ca a.
Ti liu tham kho
[1] Nguyn Vn Mu, "a thc i s v phn thc hu t", NXB Gio dc
2002.
[2] Nguyn Vn Mu,"Phng trnh hm", NXB Gio Dc, 1996.
[3] B.J.Venkatachala, "Functional Equations - A problem Solving Approach",
PRISM 2002.
[4] Cc tp ch Kvant, Ton hc v tui tr, t liu Internet.
217

You might also like